Download as pdf or txt
Download as pdf or txt
You are on page 1of 173

DAMS CBT 2015 Test -1

1. Failure of the sixth aortic arch arteries to form would lead to loss of blood supply to which of the following
essential organs?

a. Right side of the heart

b. Face

c. Thyroid gland

d. Lungs

Solution. Ans 1: (d) Lungs


Ref– Read the text below
Sol:
- Branches of the arteries of the sixth aortic arches form the pulmonary arteries. In addition, the left sixth arch
artery forms the ductus arteriosus.
- The blood supply to the right side of the heart is primarily derived from the right and left coronary arteries
derived from the truncus arteriosus.
- The face and thyroid gland receive blood primarily from the facial and superior thyroid arteries,
respectively.
- These are branches of the common and external carotid arteries which, in turn, are derivatives of the second
and third aortic arch arteries.

Answer. d

2. The scapula has no direct attachment to the axial skeleton. During development, the scapula is formed by
which of the following?

a. Splanchnic lateral plate mesoderm

b. Neural crest cells

c. Axial mesoderm

d. Somatic lateral plate mesoderm

Solution. Ans 2: (d) Somatic lateral plate mesoderm


Ref– Read the text below
Sol:
- Somatic lateral plate mesoderm gives rise to the connective tissue, cartilage, and bones of the appendages,
including the shoulder and pelvis.
- The muscles of the appendages, however, originate from somitic mesoderm [myotome]. Neural crest cells
contribute to the connective tissue of the head but not the appendages.
- Axial mesoderm forms the notochord, whereas splanchnic lateral plate mesoderm forms the smooth muscle
and connective tissue associated with the viscera.

www.damsdelhi.com Email: info@damsdelhi.com


DAMS CBT 2015 Test -1

Answer. d

3. During development,the arytenoids cartilages arise from which of the following?

a. Second pharyngeal arch

b. Third pharyngeal arch

c. Fifth pharyngeal arch

d. Sixth pharyngeal arch

Solution. Ans 3: (d) Sixth pharyngeal arch


Reference – Read the text below
Sol:
- The arytenoid cartilages arise as swellings from the sixth pharyngeal (branchial) arch at 32 days of
gestation.
- They are located between the caudal end of the hypobranchial eminence and the cranial end of the
laryngotracheal tube.
- The swellings will grow cranially to form the arytenoids and corniculate cartilages along with the primitive
aryepiglottic folds.
- In laryngomalacia, the bulky arytenoid cartilages prolapsed anteromedially on inspiration, resulting in
stridor.
- The first ,second, and third pharyngeal arches do not participate in the development of laryngeal cartilages.
- The fifth pharyngeal arch remains rudimentary and does not give rise to any adult structure.

Answer. d

4. 4 Occlusion of which of the following vessels affects the entire dorsolateral part of the rostral medulla
(level of the restiform body) and produces the lateral medullary (Wallenberg) syndrome?

a. Anterior inferior cerebellar artery

b. Anterior spinal artery

c. Posterior inferior cerebellar artery

d. Posterior spinal artery

Solution. Ans 4: (c) Posterior inferior cerebellar artery


Reference – Read the text below
Sol:
- The posterior inferior cerebellar artery supplies the rostral, dorsolateral medulla.
- The posterior spinal and anterior spinal arteries supply dorsal and ventral portions, respectively, of the
caudal medulla.

www.damsdelhi.com Email: info@damsdelhi.com


DAMS CBT 2015 Test -1
- The anterior inferior cerebellar and superior cerebellar arteries supply portions of the pons and
mesencephalon.

Answer. c

5. Transitional epithelium is found in all of the following except;

a. Ureter

b. Membranous urethra

c. Major calyx

d. Bladder

Solution. Ans 5: (b) Membranous urethra


Ref– Human Histology I. Bir Singh 4th 262
Sol:

www.damsdelhi.com Email: info@damsdelhi.com


DAMS CBT 2015 Test -1

Answer. b

6. An 18-year-old boy involved in an automobile accident presents with arm that cannot abduct. His paralysis
is caused by damage to which of the following nerves?

a. Suprascapular and axillary

b. Thoracodorsal and upper subscapular

c. Axillary and musculocutaneous

d. Radial and lower subscapular

Solution. Ans 6: (a) Suprascapular and axillary


Reference – Read the text below
Sol:
- The abductors of the arm are the deltoid and supraspinatus muscles, which are innervated by the axillary and
www.damsdelhi.com Email: info@damsdelhi.com
DAMS CBT 2015 Test -1
suprascapular nerves, respectively.

- The thoracodorsal nerve supplies the latissimus dorsi, which can adduct, extend, and rotate the arm
medially.
- The upper and lower subscapular nerves supply the subscapularis, and the lower subscapular nerve also
supplies the teres major; both of these structures can adduct and rotate the arm medially.
- The musculocutaneous nerve supplies the flexors of the arm, and the radial nerve supplies the extensors of
the arm.
- The dorsal scapular nerve supplies the levator scapulae and rhomboid muscles; these muscles elevate and
adduct the scapula, respectively.

Answer. a

7. A 17-year-old boy with a stab wound received multiple injuries on the upper part of the arm and required
surgery. If the brachial artery were ligated at its origin, which of the following arteries would supply blood to
the profunda brachii artery?

a. Lateral thoracic

b. Subscapular

c. Posterior humeral circumflex

d. Superior ulnar collateral

Solution. Ans 7: (c) Posterior humeral circumflex


Reference – Read the text below
Sol:
- The posterior humeral circumflex artery anastomoses with an ascending branch of the profunda brachii
artery, whereas the lateral thoracic and subscapular arteries do not.
- The superior ulnar collateral and radial recurrent arteries arise inferior to the origin of the profunda brachii
artery.

Answer. c

8. A 31-year-old patient complains of sensory loss over the anterior and posterior surfaces of the medial third
of the hand and the medial one and one-half fingers. He is diagnosed by a physician as having funny bones
symptoms. Which of the following nerves is injured?

a. Axillary

b. Radial

c. Median

d. Ulnar

Solution. Ans 8: (d) Ulnar


www.damsdelhi.com Email: info@damsdelhi.com
DAMS CBT 2015 Test -1
Reference – Read the text below
Sol:
- The ulnar nerve supplies sensory fibers to the skin over the palmar and dorsal surfaces of the medial third of
the hand and the medial one and one-half fingers.
- The median nerve innervates the skin of the lateral side of the palm; the palmar side of the lateral three and
one-half fingers; and the dorsal side of the index finger, the middle finger, and one-half of the ring finger.
- The radial nerve innervates the skin of the radial side of the hand and the radial two and onehalf digits over
the proximal phalanx.

Answer. d

9. The mammary glands are modified versions of which type of gland?

a. Sebaceous gland

b. Lymph gland

c. Sweat gland

d. Endocrine gland

Solution. Ans 9: (c) Sweat gland


Reference – Read the text below
Sol:
- The mammary glands are modified sweat glands.

Answer. c

10. The intercostobrachial nerve communicate with:

a. Medial brachial cutaneous nerve

b. Thoracodorsal nerve

c. Long thoracic nerve

d. First intercostal nerve

Solution. Ans 10: (a) Medial brachial cutaneous nerve


Reference – Read the text below
Sol:
- The intercostobrachial nerve communicates with the medial brachial cutaneous nerve.

Answer. a

www.damsdelhi.com Email: info@damsdelhi.com


DAMS CBT 2015 Test -1
11. A 54-year-old man comes to a hospital with abdominal pain, jaundice, loss of appetite, and weight loss. On
examination of his radiograms and computed tomography (CT) scans, there is slowly growing tumor in the
uncinate process of the pancreas. Which of the following structures is most likely compressed by this tumor?

a. Main pancreatic duct

b. Portal vein

c. Superior mesenteric artery

d. Superior pancreaticoduodenal artery

Solution. Ans 11: (c) Superior mesenteric artery


Reference – Read the text below

Sol:
- The uncinate process of the pancreas is a projection of the lower part of the head to the left behind the
superior mesenteric vessels.
- The superior pancreaticoduodenal artery runs between the duodenum and the head of the pancreas. The main
pancreatic duct runs transversely through the entire pancreas superior to the uncinate process.
- The splenic artery runs along the superior border of the pancreas. The portal vein runs behind the neck of
the pancreas.

Answer. c

12. The root of the mesentery crosses all of the following structures except:

a. Abdominal aorta

b. Inferior vena cava

c. Right ureter

d. Splenic artery

Solution. Ans 12: (d) Splenic artery


Reference – Read the text below
Sol:
- The root of the mesentery crosses (successively) the ascending and horizontal parts of the duodenum,
abdominal aorta, inferior vena cava, right ureter, right psoas major, and right testicular or ovarian vessels.

Answer. d

13. The woman undergoes hip surgery. If all of the arteries that are part of the cruciate anastomosis of the
upper thigh are ligated, which of the following arteries maintains blood flow?

www.damsdelhi.com Email: info@damsdelhi.com


DAMS CBT 2015 Test -1
a. Medial femoral circumflex

b. Lateral femoral circumflex

c. Superior gluteal

d. Inferior gluteal

Solution. Ans 13: (c) Superior gluteal


Reference – Read the text below
Sol:
* The superior gluteal artery does not participate in the cruciate anastomosis of the thigh. The inferior gluteal
artery, transverse branches of the medial and lateral femoral circumflex arteries, and an ascending branch of
the first perforating artery form the cruciate anastomosis of the thigh.

Answer. c

14. A 20-year-old patient cannot flex and medially rotate the thigh while running and climbing. Which of the
following muscles is most likely damaged?

a. Semimembranosus

b. Rectus femoris

c. Vastus intermedius

d. Tensor fasciae latae

Solution. Ans 14: (d) Tensor fasciae latae


Reference – Read the text below
Sol:
- The tensor fasciae latae can flex and medially rotate the thigh, so this is the muscle most likely damaged.
- The hamstring muscles (semitendinosus, semimembranosus, and biceps femoris) can extend the thigh and
flex the leg.
- The sartorius can flex the thigh and leg. The rectus femoris can flex the thigh and extend the leg.
- The vastus intermedius can extend the leg.

Answer. d

15. Regarding the sphenoid bone:

a. The sphenoid air sinuses in the body of the sphenoid are symmetrical structures.

b. The anterior clinoid process is part of the greater wing of sphenoid bone.

c. The posterior clinoid process is part of the lesser wing of sphenoid bone.

www.damsdelhi.com Email: info@damsdelhi.com


DAMS CBT 2015 Test -1
d. The posterior part of the floor of the anterior cranial fossa is formed by the lesser wing of sphenoid.

Solution. Ans 15: (d) The posterior part of the floor of the anterior cranial fossa is formed by the lesser wing
of
sphenoid.
Reference – Read the text below
Sol:
- The sphenoid air sinuses in the body of the sphenoid are usually asymmetrical structures.
- The anterior clinoid process it is part of the lesser wing of sphenoid bone.
- The pterygoid fossa and posterior clinoid are borne on the superior surface of the body of sphenoid.
- The posterior border of the lesser wing is the sphenoid ridge, meningiomas of skull base arise in this
location.

Answer. d

16. Type II pneumocytes have all of the following characteristics except

a. They elaborate pulmonary surfactant

b. They exhibit surface microvilli

c. They make up most of the alveolar surface area

d. They contain osmiophilic lamellar bodies

Solution. Ans-16: (c) They make up most of the alveolar surface area
Ref: Read the text below

Sol :
- Type II pneumonocytes cover less than 5% of the alveolar surface, but they form a reserve for replacement
of damaged type I pneumonocytes
- These multilamellar bodies are the source of the phospholipid-containing pulmonary surfactant.
- Defects in these cells contribute to infant and adult respiratory distress.

Answer. c

17. The oxygen transported by 100 mL of blood in a person breathing 100% oxygen at 4 atm is –

a. 9 mL

b. 6 mL

c. 3 mL

d. 0.3 mL

Solution. Ans-17: (a) 9 mL


Ref: Read the text below
www.damsdelhi.com Email: info@damsdelhi.com
DAMS CBT 2015 Test -1
Sol :
This can be solved with the help of Alveolar Gas Equation. A simplifies form of the alveolar gas equation
is as given below:-
PAO2 = PIO2 - PACO2
R
Where
- PAO2 is the alveolar O 2;
- PIO2 is the partial pressure of oxygen in the inspired air;
- PACO2 is the alveolar CO 2; PACO2 is 45 mms of Hg (same as in the venous blood)
- R is the respiratory quotient- we can take this as 1
PIO2 = (PB – PH2O ) X Fractional concentration of O 2
Where
- PB is the barometric pressure- in this case since the person is breathing at 4 atm pressure, the
value of PB is 4 X 760 mms of Hg = 3040 mms of Hg
- PH2O is the partial pressure of water vapour added to the inspired air- this is 47 mms of Hg,
irrespective of altitude
- Fractional concentration of O 2 in this case is 100/100 = 1
The PIO2 is
= {(4 X 760) – 47} X 1
= 3040 – 47
= 2993 mms of Hg
Going back to the alveolar gas equation
PAO2 = PIO2 - PACO2
= 2993 – 45
= 2948 mms of Hg
To calculate the dissolved oxygen we have to remember the following:
Amount of dissolved oxygen = .003mL/ dL/ mm of Hg i.e., for PAO2 of 1 mm of Hg, the dissolved oxygen is .
003 mL per 100 mL of blood. Therefore, when the PAO2 is 2948 mms of Hg, the dissolved oxygen is
approximately 9 mL per 100 mL of blood.

Answer. a

18. Sympathetic stimulation of the heart results in

a. An increase in the activity of the SR calcium pump

b. An increase in the duration of systole

c. An increase in the duration of diastole

d. A decrease in the affinity of troponin for calcium

Solution. Ans-18: (a) An increase in the activity of the SR calcium pump


Ref: Read the text below
Sol :
- Sympathetic stimulation of the heart activates a G protein, which, by activating adenylyl cyclase, increases
the intracellular concentration of cAMP. cAMP activates protein kinase A, which phosphorylates a number of
www.damsdelhi.com Email: info@damsdelhi.com
DAMS CBT 2015 Test -1
proteins, including phospholamban. Unphosphorylated phospholamban inhibits the SR Ca2+ pump.
- Inhibition of the SR Ca2+ pump is removed when phospholamban is phosphorylated, and, therefore, its
activity is increased.
- The increased activity of the SR Ca2+ pump removes Ca2+ from the cytoplasm rapidly and therefore
reduces the duration of systole. Sympathetic stimulation of the heart also increases heart rate.
- Therefore, the total duration of the cardiac cycle is decreased, resulting in a decrease in systole and diastole.
Because the duration of systole is reduced disproportionately by the phosphorylation of phospholamban, the
decrease in diastole isn’t as great as might be predicted by the increase in heart rate, and, therefore, diastolic
filling is not compromised.
- Sympathetic stimulation increases the amount of Ca2+ released from the SR during systole and decreases the
affinity of troponin for Ca2+. The increase in systolic Ca2+ concentration overcomes the deceased affinity of
troponin for Ca2+, resulting in an increased contractility.

Answer. a

19. Energy expenditure in resting state depends on

a. Lean body mass

b. Adipose tissue

c. Resting heart rate

d. Exercise

Solution. Ans-19: (a) Lean body mass


Ref: Read the text below
Sol :
- More the lean body mass more is the energy expenditure in the resting state.

Answer. a

20. Metabolic alkalosis will be observed in a patient with

a. Hyperaldosteronism

b. Hyperventilation

c. Persistent diarrhea

d. Renal failure

Solution. Ans-20: (a) Hyperaldosteronism


Ref: Read the text below
Sol :
- Metabolic alkalosis is caused by the loss of nonvolatile, or fixed, acid from the body. An increased
aldosterone secretion will cause an increase in H+ secretion from the distal nephron, resulting in metabolic
alkalosis.
www.damsdelhi.com Email: info@damsdelhi.com
DAMS CBT 2015 Test -1
- Persistent diarrhea will cause the loss of bicarbonate from the body resulting in metabolic acidosis. Renal
failure is often accompanied by metabolic acidosis because of the inability to excrete H+.
- Diabetes also causes metabolic acidosis because of the accumulation of keto acids. Hyperventilation results
in a respiratory alkalosis, which is compensated for by a decreased excretion of acid by the kidneys, resulting
in a metabolic acidosis.

Answer. a

21. Which one of the following substances causes renal blood flow to decrease?

a. Nitric oxide

b. Bradykinin

c. Prostaglandins

d. Adenosine

Solution. Ans-21: (d) Adenosine


Ref: Read the text below
Sol :
- Blood flow through the kidney is controlled by a myriad of humoral agents. Adenosine acts via A2 receptors
to increase afferent arteriolar resistance and thereby decrease renal blood flow.
- Nitric oxide dilates the afferent arteriole and constricts the efferent arteriole, producing a rise in glomerular
capillary pressure (and glomerular filtration) without having much of an effect on renal blood
flow.Prostaglandins, bradykinin, and dopamine all increase renal blood flow.
- Cyclooxygenase inhibitors, such as aspirin, that decrease prostaglandin synthesis may impair renal blood
flow sufficiently to exacerbate the effects of renal failure.

Answer. d

22. The mean electrical axis during the ventricular depolarization recorded in the three leads shown below
would be closest to

www.damsdelhi.com Email: info@damsdelhi.com


DAMS CBT 2015 Test -1

a. 30 degrees

b. 90 degrees

c. 150 degrees

d. 210 degrees

Solution. Ans-22: (a) 30 degrees


Ref: Read the text below
Sol :
- The mean electrical axis (MEA) represents the average direction traveled by the ventricular muscle action
potentials as they propagate through the heart.
- The propagation path and the mass of tissue through which the action potentials travel influence the
direction.
- The MEA is approximately perpendicular to the axis of the limb lead with the smallest QRS wave
magnitude.
- In this case, the smallest deflection is in lead III. Therefore, the MEA lies along lead aVR. As shown in the
diagram, the MEA could be either +30° or -150°. Since the deflection is upright in leads I and II, the MEA is
+30°.

Answer. a

23. The primary direct stimulus for excitation of central chemoreceptors is

a. In H+ Increase

b. Increase in CO 2

c. Increase in O 2

d. Decrease in CO 2

Solution. Ans-23: (a) In H+ Increase

www.damsdelhi.com Email: info@damsdelhi.com


DAMS CBT 2015 Test -1
Ref: Read the text below
Sol :
- Central or the medullary chemoreceptors are located on the ventral surface of the medulla. The most potent
stimulus for the central chemoreceptors is H+ . The central chemoreceptors monitor the H+ concentration in
the CSF, including the brain interstitial fluid. CO2 can readily penetrate membranes and can cross the blood
brain barrier whereas H+ penetrates slowly. Therefore, an increase in H+ in the blood does not have any
effect on respiration.
- CO2 that enters the brain and CSF is promptly hydrated. The H2CO3 dissociates, so that local concentration
of H+ rises and this in turn stimulates respiration.
- Thus, the effects of CO2 are mainly due to its movement into the CSF and brain interstitial fluid, where it
increases the local H+ concentration and stimulates the central chemoreceptors.

Answer. a

24. Which of the following statements regarding coronary artery blood flow in a healthy person is true ?

a. During systole, coronary artery blood flow is uniform from subendocardial to epicardial regions of the left
ventricle

b. Myocardial oxygen extraction, not coronary artery blood flow, increases during exercise

c. Coronary artery blood flow is directly proportional to arterial blood pressure over a range of pressures
within 20-30 mm Hg of normal

d. Coronary artery blood flow is proportional to myocardial oxygen demands

Solution. Ans-24: (d) Coronary artery blood flow is proportional to myocardial oxygen demands
Ref: Read the text below
Sol :
- Coronary blood flow closely matches myocardial work (or oxygen consumption).
- Myocardial oxygen extraction is near maximal at rest and does not increase appreciably, even during
exercise.
- Coronary blood flow is maximal during diastole, in which ventricular compression of the capillaries is
minimal. In addition, there is significant heterogeneity across the ventricular wal during systole, such that
subendocardial blood flow is reduced, and blood flow is shifted to the epicardial vessels.
- Autoregulation is normally observed in the myocardium, such that blood flow does not change over a large
range of perfusion pressures.

Answer. d

25. All of the following statements concerning the major determinants of glomerular filtration rate (GFR),
which are renal blood flow (RBF) and glomerular hydrostatic pressures, are correct except

a. Constriction of the afferent arteriole decreases both RBF and GFR

b. An increase in RBF, even with little change in glomerular pressure, increases GFR
www.damsdelhi.com Email: info@damsdelhi.com
DAMS CBT 2015 Test -1
c. In a normal kidney, an increase in systemic arterial pressure from 100 to 150 mm Hg increases GFR
severalfold

d. Constriction of the efferent arteriole decreases RBF and slightly increases GFR

Solution. Ans-25: (c) In a normal kidney, an increase in systemic arterial pressure from 100 to 150 mm Hg
increases GFR severalfold
Ref: Read the text below
Sol :
- Blood enters the glomerulus via an afferent arteriole and leaves via an efferent arteriole.
- A decrease in renal blood flow (RBF) or a decrease in glomerular hydrostatic pressure tends to decrease the
glomerular filtration rate (GFR).
- Accordingly, constriction of the afferent arteriole generally has this effect.
- An increase in RBF that increases hydrostatic pressure increases GFR. This effect of RBF persists even
without an increase in hydrostatic pressure because of a subtle oncotic effect.
- Although raising systemic pressure would theoretically increase hydrostatic pressure and GFR, the effect is
greatly minimized by normal autoregulation in the kidney.
- Thus, RBF and hydrostatic pressure are maintained by afferent arteriolar constriction in the presence of this
increase over normal systemic pressures. Constriction of the efferent arteriole increases hydrostatic pressure
(and GFR), but this effect is also offset by the above-mentioned decrease in RBF; thus, only a modest increase
in GFR is normally observed.

Answer. c

26. All of the following match important vasodilators with corresponding tissues except

a. Adenosine–heart

b. carbon dioxide-brain

c. Low oxygen-lung

d. Increased body temperature – skin

Solution. Ans-26: (c) Low oxygen-lung


Ref: Read the text below
Sol :
- Compared with other tissue, the lung is relatively unique in having a vasoconstrictor response to hypoxia
rather than a vasodilator response.
- Although the mechanism remains obscure, the rationale seems to be to diver blood flow from poorly
ventilated regions of the lung, thus improving the matching of ventilation and perfusion.
- The vascular bed in the heart, like those in many other organs, dilates to adenosine, and this vasodilator
mechanism may be common in its matching of blood flow to local tissue metabolism.
- The brain has a very well-described and important vasodilator response to carbon dioxide.

Answer. c

27. Which one of the following is the putative inhibitory neurotransmitter responsible for relaxation of

www.damsdelhi.com Email: info@damsdelhi.com


DAMS CBT 2015 Test -1
gastrointestinal smooth muscle?

a. Dopamine

b. Vasoactive intestinal peptide

c. Somatostatin

d. Substance P

Solution. Ans-27: (b) Vasoactive intestinal peptide


Ref: Read the text below
Sol :
- Important inhibitory neurotransmitters in the gastrointestinal tract include vasoactive intestinal peptide and
nitric oxide.
- Relaxation of gastrointestinal smooth muscle occurs following activation of nonadrenergic, noncholinergic
(NANC) enteric nerve fibers. Acetylcholine, substance P, and dopamine are excitatory neurotransmitters.
- Somatostatin is a paracrine secretory product with multiple effects on gastrointestinal function.

Answer. b

28. Cholera toxin causes diarrhea by inhibiting

a. Neutral NaCl absorption in the small intestine

b. Electrogenic Na absorption from the small intestine

c. Na-glucose coupled absorption from the small intestine

d. Na/H exchange in the small intestine

Solution. Ans-28: (a) Neutral NaCl absorption in the small intestine


Ref: Read the text below
Sol :
- Diarrhea is defined as the excretion of 200 g or more of water in the stools of an adult during a 24-h period.
- Although Na is absorbed from the small intestine by several mechanisms, cholera toxin specifically inhibits
neutral NaCl absorption.
- In addition, cholera, the most severe form of diarrhea, produces its effect by increasing salt and water
secretion by intestinal crypt cells.

Answer. a

29. The highest blood flow per gram of left ventricular myocardium would occur

a. When aortic pressure is highest

www.damsdelhi.com Email: info@damsdelhi.com


DAMS CBT 2015 Test -1
b. When left ventricular pressure is highest

c. At the beginning of isovolumic contraction

d. At the beginning of diastole

Solution. Ans-29: (d) At the beginning of diastole


Ref: Read the text below
Sol :
- Blood flow through the coronary vessels of the left ventricle is determined by the ratio of perfusion pressure
to vascular resistance.
- The perfusion pressure is directly related to the aortic pressure at the ostia of the coronaries. Myocardial
vascular resistance is significantly influenced by the contractile activity of the ventricle.
- During systole, when the ventricle is contracting, vascular resistance increases substantially.
- Flow is highest just at the beginning of diastole because, during this phase of the cardiac cycle, aortic
pressure is still relatively high and vascular resistance is low due to the fact that the coronary vessels are no
longer being squeezed by the contracting myocardium.

Answer. d

30. Central venous pressure is increased by

a. Decreasing blood volume

b. Increasing venous compliance

c. Increasing total peripheral resistance

d. Decreasing heart rate

Solution. Ans-30: (d) Decreasing heart rate


Ref: Read the text below
Sol :
- Central venous pressure is the hydrostatic pressure in the great veins at their entrance to the right atrium.
- Increasing venous compliance would decrease pressure in the venous vessels and therefore would decrease
central venous pressure.
- Decreasing blood volume would have the same effect. Reducing the plasma concentration of aldosterone
would result in a decrease in blood volume.
- Increasing total peripheral resistance would tend to shift volume from the venous side of the circulation to
the arterial side, resulting in a decrease in venous pressure.

- Increasing cardiac output tends to lower central venous pressure, whereas lowering cardiac output tends to
increase central venous pressure. A reduction in heart rate would tend to lower cardiac output and therefore
increase central venous pressure.

Answer. d

31. Prokaryotic promoter of transcription situated 35 bp upstream is:

www.damsdelhi.com Email: info@damsdelhi.com


DAMS CBT 2015 Test -1
a. TATA box

b. Goldberg hogness box

c. CAAT box

d. TGG box

Solution. Ans-31: (d) TGG box


Ref.: Read the text below
Sol :
. Bacterial promoter situated 35 bp upstream to the starting site of transcription is TGG box
. Promoters are short conserved sequences in the coding strand of DNA that specifies the start
of transcription
Bacterial promoters :
. TATA box of pribnow box
Situated 10 bp upstream
. TGG box
Situated 35 bp upstream
. Eukaryotic promoters :
. Goldberg hogness box
Situated 25 bp upstream
. CAAT box
Situated 25 bp upstream
. CAAT box
Situated 70 bp upstream
GC rich region
. The DNA strand that is transcribed into a RNA molecule is : Template/sense strand/n on
coding/’+’ strand
Opposite strand is : Coding/antisense/non template/ ‘-‘ strand Sigma subunit of RNA polymerase
recognizes promoter site

Answer. d

32. All are true about fibrous proteins except:

a. They have mainly structural functions

b. They have an axial ratio of < 3

c. Elastin is a fibrous protein

d. They have negligible water solubility

Solution. Ans-32: (b) They have an axial ratio of < 3


Ref.: Read the text below
Sol :

. Axial ratio is the ratio of length to breadth.


. Fibrous proteins possess axial ratio of 10 or more.
www.damsdelhi.com Email: info@damsdelhi.com
DAMS CBT 2015 Test -1
Fibrous Proteins :
. Structural proteins
. Minimum water solubility
. Axial ratio > 10
. Eg. Collagen, Keratin, elastin
Globular proteins
. Spherical shape
. Axial ratio < 3
. Dynamic functions
. Eg. Albumin, globulin, most enzymes

Answer. b

33. Calcium absorption is increased in the GUT by :

a. 1,25 dihydroxycholecalciferol

b. Intrinsic factor

c. Calcitonin

d. PTH

Solution. Ans-33: (a) 1,25 dihydroxycholecalciferol


Ref.: Read the text below
Sol :
. In liver, Vitamin – D is hydroxylated at the 25th position, and in kidney further hydroxylation is
affected at the 1st position to produce dihydroxy-cholecalciferol or calcitriol.
. The calcitriol induces a carrier protein in the intestinal mucosa, which increases the absorption
of calcium. Hence blood calcium level tends to be elevated.
. Vitamin D is acting independently on bone. Vitamin D increases the number and activity of
osteoblasts, the bone forming cells.
. Secretion of alkaline phosphatase by osteoblasts is increased by vitamin D

Answer. a

34. LDL lipoprotein is associated with :

a. Apoprotein A2

b. Apoprotein B100

c. Apoprotein C1

d. Apoprotein E

Solution. Ans-34: (b) Apoprotein B100


Ref.: Read the text below

www.damsdelhi.com Email: info@damsdelhi.com


DAMS CBT 2015 Test -1
Sol :
. Low-density lipoprotein (LDL) is one of the five major groups of lipoproteins, which in order of

size, largest to smallest, are chylomicrons, VLDL, IDL, LDL, and HDL, that enable transport of
multiple different fat molecules, including cholesterol, within the water around cells and within
the water-based bloodstream
. Each native LDL particle contains a single apolipoprotein B-100 molecule (Apo B-100, a protein
that has 4536 amino acid residues and a mass of 514 kDa), which circulates the fatty acids,

keeping them soluble in the aqueous environment.[citation needed] In addition, LDL has a highly
hydrophobic core consisting of polyunsaturated fatty acid known as linoleate and about 1500
esterified cholesterol molecules.

Answer. b

35. cAMP is required for the activation of :

a. Phosphofructokinase I

b. Protein kinase

c. Phosphoglucomutase

d. Hexokinase

Solution. Ans-35: (b) Protein kinase


Ref.: Read the text below
Sol :
Some actions of cyclic AMP are summarized below :
. It activates “PROTEIN KINASE”.
. Cyclic AMP helps in conversion of cholesterol in pregnenolone.
. It stimulates 11-beta – hydroxylase for steroidogenesis.
. It stimulates tyrosine hydroxylase for steroidogenesis.
. It stimulates tyrosine hydroxylase for catecholamine synthesis.

Answer. b

36. After a 72-hour fast the substance likely to the involved in the formation of energy would be :

a. Muscle glycogen

b. Liver glycogen

c. Aminoacid

d. Acetoacetate

Solution. Ans-36: (d) Acetoacetate


www.damsdelhi.com Email: info@damsdelhi.com
DAMS CBT 2015 Test -1
Ref.: Read the text below
Sol :
. The ketone bodies, acetoacetate and 13-hydroxy butyrate are acids.
. In starvation, the dietary supply of glucose is decreased. The increased rate of lipolyssis is to
provide alternate source of fuel.
. The excess acetyl CoA is converted to ketone bodies.
. The high glucagon – insulin ratio prevailing under conditions of starvation favours ketogenesis.
. The brain derives 75% of energy from ketone bodies under conditions of fasting.
. Hyper-emesis in early pregnancy and prolonged labour are other causes for ketosis in clinical
practice.

Answer. d

37. Which among the following is a feature of non competitive inhibition?

a. Increased Vmax

b. Decreased Vmax

c. Increased Km

d. Decreased Km

Solution. Ans-37: (b) Decreased Vmax

Ref.: Read the text below

Sol :

A non competitive inhibitor has no effect on Km. but decrease V max


V max : maximum velocity
Km : the substrate concentration at which the enzyme attains half of the V max

Competitive inhibition Non competitive inhibition


Acting on Active site May or may not
Structure of inhibitor Competitive inhibition Non competitive inhibition
Inhibition is Active site May or may not
Excess substrate Substrate analogue Unrelated
Km Reversible Generally irreversible
V max Inhibition relieved No effect
Significance Increased No change
No change Decreased
Drug action Toxicological

www.damsdelhi.com Email: info@damsdelhi.com


DAMS CBT 2015 Test -1

Answer. b

38. Acute phase reactants are all except

a. Ferritin

b. Heparin

c. Plasminogen

d. Serum amyloid A protein

Solution. Ans-38: (b) Heparin


Ref.: Read the text below
Sol :
. ACUTE PHASE REACTANTS

Acute phase reactants Acute phase proteins Negative acute phase proteins
§ C-reactive protein : reacts
against C-polysaccharide of
pneumococcal capsule (Q), most
sensitive marker of coronary
artery disease
§ Levels are decreased in
response to inflammation
§ Ceruloplasmin
§ Albumin
§ Ferritin
§ Transthyretin
§ ESR § Alpha-1 antitrypsin & alpha-2
macroglobulin
§ Retinol binding protein
§ Viscosity of blood
§ Factor VIII, fibrinogen &
§ Transferrin
plasminogen
§ Iron
§ Serum amyloid A protein
§ Fetuin
§ Haptoglobulin

§ Complement proteins

§ Lipoprotein A

Answer. b

www.damsdelhi.com Email: info@damsdelhi.com


DAMS CBT 2015 Test -1
39. Encircle the material which is not involved in protein translation in eukaryotes?

a. RNA polymerase

b. Aminoacylt – RNA

c. Ribosomes

d. Peptidyl transferase

Solution. Ans-39: (a) RNA polymerase


Ref.: Read the text below
Sol :
. RNA polymerase enzyme is involved in transcription process, not in translation process.
. Aminoacyl t RNA is needed during the process of translation to carry the amino acid at the
surface of the ribosome for the attachment in the elongating polypeptide chain.
. Ribosome is the organell on which the protein synthesis takes place.

Answer. a

40. The 40 nm gap in between adjacent tropocollagen molecule in collagen which serve as the site of bone
formation is occupied by which of the following moiety?

a. Carbohydrate

b. Ligand moiety

c. Ca++
d. Fe +++

Solution. Ans-40: (c) Ca++

Ref.:Read the text below

Sol :

“The mechanisms involved in mineralization are not fully understood, but several factors have been
implicated.
Alkaline phosphatase contributes to mineralization, but in itself is not sufficient.
Small vesicles (matrix vesicles) containing calcium and phosphate have been described at sites of
mineralization, but their role is not clear.
Type I collagen appears to be necessary, with mineralization being first evident in the gaps between
successive molecules.”

Answer. c

41. If consecutive four nucleotides code for an amino acid, how many amino acid can be theoretically coded by
www.damsdelhi.com Email: info@damsdelhi.com
DAMS CBT 2015 Test -1
nucleic acid?

a. 4

b. 64

c. 16

d. 256

Solution. Ans-41: (d) 256


Ref.: Read the text below
Sol :
- If there would have been two nucleotide in each codon the chance of their varied
combination will be 4 2 i.e. 16, and three nucleotide in each codon means there will occur 4 3 i.e. 64 codons
are possible.
- According to the above calculation, if there occurs 4 nucleotide in a codon, the chance of there varied
combination will be 4 4 i.e. 256.

Answer. d

42. NH4 released by :

a. Arginase

b. Glutamate dehydrogenase

c. Glutaminase

d. All of the above

Solution. Ans-42: (c) Glutaminase


Ref.: Read the text below

Sol :

The glutaminase present in the tubular cells can hydrolyze glutamine to ammonia and glutamic acid. The
NH3 diffuses into the luminal fluid and combines with H+ to form NH4- + is excreted as NH4+ in
acidosis.
PS : Glutamate dehydrogenase causes the synthesis of ammonia from nitrogen in nitrogen metabolism,
but not ammonium ion.

Answer. c

43. Normal role of micro RNA is?

www.damsdelhi.com Email: info@damsdelhi.com


DAMS CBT 2015 Test -1

a. Gene regulation

b. RNA splicing

c. Translation initiation

d. Conformational change of DNA

Solution. Ans-43: (a) Gene regulation


Ref.: Read the text below
Sol :
- Micro RNA and si RNA are both involved in negative gene regulation
- Difference between miRNA and si RNA

miRNA Si RNA
§ Involved in negative gene Si RNA
regulation § Involved in negative gene
regulation

§ Small size (21-25 nucleotide large) § Small size (21-25 nucleotide large)
§ Small size (21-25 nucleotide large)
§ Derived from single stranded
RNA § Derived from double stranded
RNA
§ miRNA binds the 3’ untranslated
region (UTR) of the mRNA and thus § Si RNA bind nonspecific area of
prevents the expression of the mRNA the mRNA and this duplex is
degraded in the P bodies

www.damsdelhi.com Email: info@damsdelhi.com


DAMS CBT 2015 Test -1

Answer. a

44. For single carbon transfer, which co-enzyme is responsible?

a. Acetyl Coenzyme A

b. Biotin

c. THFA

d. Pyridoxine

Solution. Ans-44: (c) THFA


Ref.: Read the text below

Sol :
Folic Acid

Coenzyme from: Tetrahydro-folic acid (THFA).

Function : Transporter for single carbon units (-CH3, - Ch2, COOH) for purine, thymidylate synthesis and
methionine synthesis from homocysteine – requires B 12.

Deficiency :

Most common vitamin deficiency in humans


Hematopoietic tissue – anemia
Epithelial cells – nutrient absorpt*ion *impaired.

Answer. c

45. In eukaryotes, initiation of translation of m-RNA requires :

a. Aminoacyl t-RNA synthetase

b. GTP

c. elF

d. Ribosomes 40s

Solution. Ans-45: (c) elF


Ref.: Read the text below
Sol :
- The prefix eIF denotes a eukaryotic initiation factor. For example, eIF – 4E is a protein that binds directly to
the 7-methylguanosine cap, whereas eIF-4A is a helicase.
www.damsdelhi.com Email: info@damsdelhi.com
DAMS CBT 2015 Test -1
- The difference in initiation mechanism between prokaryotes and eukaryotes is, in part, a consequence of the
difference in RNA processing.
- The 5 end of mRNA is readily available to ribosomes immediately after transcription in prokaryotes.

Answer. c

46. Subendothelial deposits in glomerulus are seen in which of the following glomerulonephropathies?

a. MPGN

b. PSGN

c. Minimal change disease

d. FSGS

Solution. Ans-46: (a) MPGN

Ref.:Read the text below

Sol :

MEMBRANOPROLIFERATIVE GLOMERULONEPHROPATHY

The glomeruli are large and hypercellular due to proliferation of cells in the mesangium and
endocapillary proliferation involving capillary endothelium and infiltrating leukocytes
Crescents are present in many cases
The glomerular capillary wall shows a ‘double-contour” or “tram – track” appearance (GBM splitting)
Type 1 MPGN: Characterized by the presence of discrete subendothelial electron-dense deposits.
By immunofluorescence, C3 is deposited in a granular pattern, and IgG and early complement
components (C1q and C4) are often also present.

Answer. a

47. Congo red staining for amyloid under polarized light shows :

a. Silver birefringence

b. Golden birefringence

c. Blue birefringence

d. Green birefringence

Solution. Ans-47: (d) Green birefringence


Ref: Robbin’s Pathology, 8th ed., p-254
Sol :
www.damsdelhi.com Email: info@damsdelhi.com
DAMS CBT 2015 Test -1
- The traditional way of identifying amyloid in tissue sections has been staining with Congo red and
demonstration of green birefringence under crossed polarizers.

Answer. d

48. A 10-year-old child has had recurrent otitis media for the past 8 years. On physical examination, there is
hepatosplenomegaly. Laboratory findings include anemia and leucopenia . A bone marrow biopsy is
performed, and high magnification of the sample shows the findings depicted in the figure. An inherited
deficiency of which of the following enzymes is most likely to produce these findings?

a. Glucocerebrosidase

b. Acid maltase

c. Sphingomyelinase

d. Hexosaminidase A

Solution. Ans 48: (a) Glucocerebrosidase


Ref:Read the text below
Sol:
- This child has one of the forms of Gaucher disease.
- Type1, seen in the child, accounts for 99% of cases and does not involve the central nervous system (CNS) .
It is caused by deficiency of glucocerbrosidase.

Answer. a

49. Metastatic calcification seen in all except:

a. Multiple myeloma

www.damsdelhi.com Email: info@damsdelhi.com


DAMS CBT 2015 Test -1
b. Breast cancer

c. Atherosclerosis

d. Renal failure

Solution. Ans-49: (c) Atherosclerosis


Ref.: Read the text below
Sol :
Metastatic calcification almost always results secondary to disturbances in calcium metabolism.
Hypercalcemia also accentuates dystrophic calcification. It is seen in tissues which have propensity to lose
acid.
It affects.
- Alveolar walls of lungs (most common site)
- Gastric mucosa
- Pulmonary veins
- Systemic arteries
- Kidneys
Conditions associated with metastatic calcification :
- Primary tumors of bone marrow
- Skeletal metastasis
- Leukemia
- Paget’s disease
- Sarcoidosis
- William’s syndrome
- Renal failure

Answer. c

50. Bronchiolitis obliterans is pathognomonic of which type of lung transplant rejection?

a. Acute

b. Hyperacute

c. Chronic

d. Transfusion related acute lung injury (TRALI)

Solution. Ans-50: (c) Chronic


Ref.: Read the text below
Sol :
- The transplanted lung is subject to two major complications: infection and rejection
- Acute rejection of the lung occurs to some degree in all patients despite routine immune suppression.
- It often occurs during the early weeks to months after surgery but may occur years later whenever immune
suppression is decreased.
- Chronic rejection is a significant problem in at least half of all lung transplant patients by 3 to 5 years.

Answer. c

www.damsdelhi.com Email: info@damsdelhi.com


DAMS CBT 2015 Test -1

51. Massive splenomegaly is not seen in :

a. Malaria

b. Chronic myelogenous leukemia

c. Syphilis

d. Sickle cell anemia

Solution. Ans-51: (d) Sickle cell anemia


Ref: Robbin’s - 633
Sol :
THE CAUSES OF MASSIVE SPLENOMEGALY INCLUDES :
(i) Thalassemia
(ii) Visceral leishmaniasis (Kala Azar)
(iii) Schistosomiasis
(iv) Chronic myelogenous leukemia
(v) Chronic lymphocytic leukemia
(vi) Lymphomas
(vii) Hairy cell leukemia
(viii) Myelofibrosis
(ix) Polycythemia vera
(x) Gauchers disease
(xi) Niemann Pick disease
(xii) Sarcoidosis
(xiii) Autoimmune hemolytic anemia
(xiv) Malaria

Answer. d

52. The commonest site for extragonadal germ cell tumour is

a. Retroperitoneum

b. Sacrococcygeal region

c. Pineal gland

d. Mediastinum

Solution. Ans-52: (d) Mediastinum


Ref.: Ackerman’s 9th/e p. 485
Sol :
Extragonadal germinal cell syndromes are rare tumors that predominantly affect young males. Literature
suggests that the only known risk factor for extragonadal germ cell tumors (EGCTs) is Klinefelter syndrome
(47XXY), which is associated with mediastinal nonseminomatous germ cell tumors. They are characterized
www.damsdelhi.com Email: info@damsdelhi.com
DAMS CBT 2015 Test -1
by their location on the midline from the pineal gland to the coccyx. In extragonadal germ cell tumors, no
evidence of a primary malignancy is present in either the testes or ovaries by radiologic imaging or physical
examination. Extragonadal germ cell tumors produce a rich symptomatology and may reach large volumes if
they arise in silent areas. Histologically, they mirror their gonadal counterparts with which they share the
same chemosensitivity and radiosensitivity. Modern approaches to diagnosis and treatment can result in high
rates of long-term survival and even cure.
- The most common site of extragonadal germ cell tumors (EGGCTs) is the mediastinum (50- 70%) followed
by the retroperitoneum (30-40%), the pineal gland (5%), and the sacrococcygeal area (less than 5%).

Answer. d

53. A 50 year- old man has experienced midabdominal pain for several weeks. Stool is positive for occult
blood. An upper gastrointestinal endoscopy is performed and biopsies taken. Microscopic examination of a
biopsy specimen of a duodenal lesion is shown. Which of the following terms best describes this lesion?

a. Abscess

b. Caseating granuloma

c. Chronic inflammation

d. Ulceration

Solution. Ans 53: (d) Ulceration


Ref: Read the text below
Sol:
- Inflammation involving an epithelial surface may cause such extensive necrosis that the surface becomes
eroded, forming an ulcer.

Answer. d

www.damsdelhi.com Email: info@damsdelhi.com


DAMS CBT 2015 Test -1

54. Growth factor oncogen is

a. Myc

b. Fos

c. Sis

d. Jun

Solution. Ans-54: (c) Sis


Ref.: Robbins 8th/e p. 281
Sol :
- Sis oncogene is a growth factor.
- Myc, jun and fos oncogenes are nuclear regulatory proteins

Answer. c

55. Sezary syndrome is included in category of

a. T cell leukemia

b. Lymphoma

c. B cell leukemia

d. Pigmented disorder of skin

Solution. Ans-55: (a) T cell leukemia


Ref.: Robbins 8th/e p. 616
Sol :
- Mycosis fungoides & sezary syndrome appear to be a different manifestations of a single neoplastic entity.
- It is an indolent disorder of peripheral CD4T cells that is characterized by the involvement of skin &
therefore belong to the group of cutaneous T cell lymphoid neoplasms.

Answer. a

56. Life of neutrophil in circulation is normally

a. 4 to 8 hours

b. 4 to 8 mins

c. 24 to 28 hours

www.damsdelhi.com Email: info@damsdelhi.com


DAMS CBT 2015 Test -1
d. 24 to 28 mins

Solution. Ans-56: (a) 4 to 8 hours


Ref.: Guyton’ 366
Sol :
- Life of granulocyte (Neutrophil) in circulation is normally 4 to 8 hours and another 4 to 5 days in tissues.
- Lymphocytes may be
o Short live → Life span 2 weeks
o Long lived → Life span upto 3 years
- Monocytes in Circulation → 1 – 3 days.
- Platelets → 7 – 10 days.

Answer. a

57. A 23-year-old female sought medical help because of a painless asymmetrical enlargement of the lower
neck. The patient had no history of dyspnea, dysphagia, hoarseness, or previous radiation exposure. On
physical examination, besides the enlarged asymmetrical thyroid gland, there was also a palpable
lymphadenopathy. A lymph node biopsy (see Figure) was performed. Hematoxilin and eosin (H&E) stained
slide shows the lesion. What is the most appropriate diagnosis?

www.damsdelhi.com Email: info@damsdelhi.com


DAMS CBT 2015 Test -1

a. Medullary carcinoma of the thyroid

b. Follicular carcinoma

c. Papillary carcinoma

d. Anaplastic carcinoma

Solution. Ans-57: (c) Papillary carcinoma


Ref: Read the text below
Sol :
- Papillary carcinoma of the thyroid is the most common form of thyroid cancer. Most cases are seen between
the second and third decade of life and are associated with previous radiation therapy.
- Many times the first manifestation is a metastasis to the regional neck nodes. The histologic characteristics
of papillary carcinoma are branching papillae with single or multiple layers of cuboidal to columnar cells.
- The characteristic appearance of the nucleus is rather clear, ground-glass (orphan Annie) nuclei.
- Characteristic intracytoplasmic inclusions, and occasional grooves, are seen. Psammoma bodies are often
present in the papillae.
- The most common variant of papillary carcinoma is the follicular variant, in which the tumor cells form
follicular architecture; however, the nuclear changes, as well as focal areas of papillary structures, are enough
to make the differential diagnosis from follicular carcinoma.

Answer. c

58. Which of the following changes associated with shock may not revert back to normal?

www.damsdelhi.com Email: info@damsdelhi.com


DAMS CBT 2015 Test -1
a. Diffuse alveolar damage

b. Adrenal cortical lipid depletion

c. Acute tubular necrosis

d. Neuronal damage

Solution. Ans-58: (d) Neuronal damage


Ref.: Read the text below
Sol :
Three Major Types of shock

Answer. d

59. Most common primary tumor of heart is:

a. Myxoma

www.damsdelhi.com Email: info@damsdelhi.com


DAMS CBT 2015 Test -1
b. Rhabdomyoma

c. Fibroma

d. Lipoma

Solution. Ans-59: (a) Myxoma


Ref.: Read the text below
Sol :
- The most common primary cardiac tumors, in descending order of frequency (overall, including adults and
children), are :
- Myxomas, fibromas, lipomas, papillary fibroelastomas, rhabdomyomas, angiosarcomas, and other sarcomas
- 90% are located in the atria (atrial myxomas), with a left-to-right ratio of approximately 4:1
- The region of the fossa ovalis in the atrial septum is the favored site of origin
- Myxomas elaborate IL-6 leading to constitutional symptoms like fever, weight loss, cachexia, malaise and
arthralgias
- 90% are sporadic, 10% familial with autosomal dominant transmission, which form part of carney complex

Answer. a

60. Irreversible injury in MI occurs within:

a. 40 min

b. 10 min

c. 60 min

d. 90 min

Solution. Ans-60: (a) 40 min


Ref.: Read the text below
Sol :
- Irreversible injury occurs within 20-40 min
- Approximate Time of Onset of Key Events in Ischemic cardiac Myocytes

www.damsdelhi.com Email: info@damsdelhi.com


DAMS CBT 2015 Test -1

Answer. a

61. All are true about extrinsic asthma except?

a. Presents in childhood

b. Family history of allergy/asthma present

c. IgE levels increased

d. Samter’s triad seen

Solution. Ans-61: (d) Samter’s triad seen


Ref: Read the text below
Sol :
- Samter’s triad (asthma, aspirin sensitivity, and nasal polyps ) is seen with intrinsic asthma

Answer. d

www.damsdelhi.com Email: info@damsdelhi.com


DAMS CBT 2015 Test -1

62. PAS positive diastase resistant granules in intestine are seen in:

a. Pseudomembranous colitis

b. Whipple’s disease

c. Tropical sprue

d. Small bowel carcinoid

Solution. Ans-62: (b) Whipple’s disease


Ref.: Read the text below
Sol :
WHIPPLE’S DISEASE
- Chronic multisystem disease caused by a Gram-positive actinomycetes,Tropheryma whipplei
- The disease is more common in middle aged white men Clinical features : The onset is insidious and is
characterized by diarrhea, steatorrhea, abdominal pain, weight loss, migratory large joint arthropathy, fever,
ophthalmic and CNS symptoms
- The development of dementia is a relatively late symptom and an extremely poor prognostic sign
- Culture negative endocarditis may be seen
- PAS positive diastase resistant granules in intestine are seen in Whipple’s disease. -
It is associated with presence of rod shaped bacteria.

Answer. b

63. All are features of tumor lysis syndrome except:

a. Hypocalcemia

b. Hyperkalemia

c. Hypophosphatemia

d. Hyperuricemia

Solution. Ans-63: (c) Hypophosphatemia


Ref.: Read the text below
Sol :
TUMOR LYSIS SYNDROME
Includes :
- Hyperuricemia
- Hyperkalemia
- Hyperphosphatemia
- Hypocalcemia
- Lactic acidosis

www.damsdelhi.com Email: info@damsdelhi.com


DAMS CBT 2015 Test -1
It is associated with :
- Hodgkin lymphoma
- All (most common)
- CLL
- Less commonly with solid tumors

Answer. c

64. All are true regarding polycythemia vera except:

a. JAK2 mutation seen

b. Can cause venous thrombosis

c. Serum urea levels low

d. Normal erythropoietin levels

Solution. Ans-64: (c) Serum urea levels low


Ref.: Read the text below
Sol :
POLYCYTHEMIA VERA
- Serum urea levels are high due to high cell turn over in PV.
- Symptomatic gout is seen in 5-10% individuals
- Polycythemia vera is associated with activating point mutation in the typrosine kinase JAK2 Clinical features
include :
- Mostly discovered incidentally on routine hemogram
- Headache, dizziness
- Thrombosis: DVT, MI, mesenteric/hepatic/dural venous thrombosis
- Bleeding : due to defect in platelet function (can be minor or major bleeds)
- Hypertension
- Aquagenic pruritus
- GI ulcers (due to histamine release as a result of basophilia)
- Hyperuricemia/gout
- Massive splenomegaly

Answer. c

65. Carcinoma, which has predilection for metastasis to hands and feet bones is :

a. Prostate

b. Bronchus

c. Pelvis

www.damsdelhi.com Email: info@damsdelhi.com


DAMS CBT 2015 Test -1
d. Breast

Solution. Ans-65: (b) Bronchus


Ref: Read the text below
Sol :
- In adults, the metastatic lesions generally occur in the axial skeleton and other sites with residual red
marrow, although the lesions may be found anywhere in the skeletal system.
- Common sites for metastases are the vertebrae, pelvis, proximal parts of the femur, ribs, proximal part of the
humerus, and skull. Certain carcinomas may have a predilection for particular skeletal sites. For example,
metastases to the bones of the hands and feet are rare, but 50% of hand metastases originate from lung
neoplasm.

Answer. b

66. Which of the following antirheumatic drugs can be administered intravenously?

a. Adalimumab

b. Etanercept

c. Infliximab

d. Anakinra

Solution. Ans 66: (c) Infliximab


Ref– Read
Sol:

Answer. c

67. All of the following decrease iron absorption except –

a. Tetracyclines

b. Phosphates

www.damsdelhi.com Email: info@damsdelhi.com


DAMS CBT 2015 Test -1
c. Phytates

d. Ascorbic acid

Solution. Ans 67: (d) Ascorbic acid


Ref– Read
Sol:

Factors increasing iron absorption Factors decreasing iron absorption


Factors decreasing iron absorption
∙ Alkalies / Antacids (by
∙ Acid (by favouring dissolution rendering iron insoluble and opposing
and reduction of ferric iron) its reduction)
∙ Reducing substances like ∙ Phosphates (by complexing iron)
ascorbic acid and amino acids
containing sulfhydryl radical ∙ Phytates (by complexing iron)
∙ Meat (by increasing HCl ∙ Tetracyclines (by complexing
secretion and providing heme iron iron)

∙ Presence of other foods in the


stomach

www.damsdelhi.com Email: info@damsdelhi.com


DAMS CBT 2015 Test -1

Answer. d

68. Of the many types of adrenergic receptors found throughout the body, which is most likely responsible for
the cardiac stimulation that is observed following an intravenous injection of epinephrine?

a. α1-adrenergic receptors

b. α2 -adrenergic receptors

c. β1-adrenergic receptors

d. β2-adrenergic receptors

Solution. Ans 68: (c) β1-adrenergic receptors.


Reference – Read the text below
Sol:
- Stimulation of both the contractile and rhythmic effects of epinephrine on the heart is
mediated through activation of postsynaptic β 1-adrenergic receptors.
- These receptor sites mediate an epinephrine-induced increased firing rate of the SA
node,increased conduction velocity through the AV node and the His-Purkinje system, and
increased contractility and conduction velocity of atrial and ventricular muscle.
-Epinephrine activation of α adrenoceptors does not affect cardiac function. β 2-adrenergic
receptors play a minor role in cardiac stimulation.
- They are more important in the relaxation of tracheobronchial smooth muscle, relaxation of the
detrusor of the urinary bladder, dilation of arterioles that serve skeletal muscles, and increased
secretion of insulin by the pancreas.
- Lipolysis in fat cells and melatonin secretion by the pineal gland appear to involve stimulation of
β 3-adrenergic receptors.

Answer. c

69. A nucleophilic attack on deoxyribonucleic acid (DNA) that causes the disruption of base pairing occurs as
a result of the administration of

a. Cyclophosphamide

b. 5-FU

c. Methotrexate

d. Prednisone

Solution. Ans 69: (a) Cyclophosphamide


Reference – Read the text below
Sol:

www.damsdelhi.com Email: info@damsdelhi.com


DAMS CBT 2015 Test -1
- Cyclophosphamide, an alkylating agent, reacts with purine and pyrimidine bases of DNA to
form bridges and dimers.
- These products interfere with DNA replication. 5-FU,methotrexate, and 6-thioguanine are
antimetabolites, and the steroid prednisone has some tumor-suppressive effects.

Answer. a

70. Morphine may be characterized best by which of the following statements?

a. It is classified as a mixed agonist-antagonist drug

b. It is used medically to inhibit withdrawal symptoms in persons who are dependent on heroin

c. At high doses, it causes death by respiratory depression

d. It has an addiction potential equal to that of codeine

Solution. Ans 70: (c) At high doses, it causes death by respiratory depression.
Reference – Read the text below
Sol:
- Morphine is a pure agonist opioid drug with agonist activity toward all the opioid subtype
receptor sites.
- In high doses, deaths associated with morphine are related to the depression of the respiratory
center in the medulla.
- Morphine has a high addiction potential related to the activity of heroin or dihydromorphine.
- Codeine has a significantly lower addiction potential.

Answer. c

71. Haloperidol may best be characterized by which of the following statements?

a. It is classified as a phenothiazine

b. It is a selective D2 receptor agonist

c. Its mechanism of action is completely different from that of chlorpromazine

d. It is more potent as an antipsychotic drug than is chlorpromazine

Solution. Ans 71: (d) It is more potent as an antipsychotic drug than is chlorpromazine.
Reference – Read the text below
Sol:
- Haloperidol is a butyrophenone derivative with the same mechanism of action as the
phenothiazines, that is, blockade of dopaminergic receptors.
- It is more selective for D2 receptors.
- Haloperidol is more potent on a weight basis than the phenothiazines, but produces a higher
incidence of extrapyramidal reactions than does chlorpromazine
www.damsdelhi.com Email: info@damsdelhi.com
DAMS CBT 2015 Test -1

Answer. d

72. Which of the following antimalarials can be combined with arterolane in ACT regimen for uncomplicated
falciparum malaria?

a. Lumefantrine

b. Piperaquine

c. Amodiaquine

d. Pyronaridine

Solution. Ans 72: (b) Piperaquine


Ref– Read

Sol:

ACT regimens for uncomplicated falciparum malaria


∙ Artesunate + Amodiaquine

∙ Artesunate + Mefloquine

∙ Artesunate + Pyronaridine

∙ Artesunate + Sulfadoxine / Pyrimethamine

∙ Artemether + Lumefantrine

∙ Arterolane + Piperaquine

∙ Dihydroartemisinin + Piperaquine

Answer. b

73. Rosiglitazone has been withdrawn due to an increased risk of –

a. Carcinoma of gallbladder

b. Carcinoma of urinary bladder

c. Congestive cardiac failure

d. Hepatotoxicity

Solution. Ans 73: (c) Congestive cardiac failure


www.damsdelhi.com Email: info@damsdelhi.com
DAMS CBT 2015 Test -1
Ref– Read
Sol:
- Troglitazone has been withdrawn due to hepatotoxicity
- Pioglitazone has been withdrawn in many Western countries due to carcinoma of urinary
bladder

Answer. c

74. The highly sedative antihistaminic drug is –

a. Hydroxyzine

b. Cinnarizine

c. Pheniramine

d. Chlorpheniramine

Solution. Ans 74: (a) Hydroxyzine


Ref– Read
Sol:

Classification of 1st generation H1 antihistaminics:

∙ Diphenhydramine

∙ Dimenhydrinate
Highly sedative Dimenhydrinate

Promethazine

www.damsdelhi.com Email: info@damsdelhi.com


DAMS CBT 2015 Test -1

∙ Hydroxyzine
∙ Pheniramine

∙ Cyproheptadine

Moderately sedative ∙ Meclizine

∙ Buclizine

∙ Cinnarizine
∙ Chlorpheniramine

∙ Dexchlorpheniramine

Mildly sedative ∙ Triprolidine

∙ Clemastine

∙ Cyclizine

Answer. a

75. Which of the following best describes diltiazem’s effect on digoxin?

a. It decreases digoxin metabolism

b. It decreases digoxin renal excretion

c. It decreases digoxin plasma protein binding

d. It decreases digoxin intestinal absorption

Solution. Ans-75: (b) It decreases digoxin renal excretion


Ref: Read the text below.
Sol:
- Digoxin levels rise with concomitant administration of diltiazem by an unknown mechanism
that reduces renal clearance.

Answer. b

76. A 75-year-old male, postprostatectomy for carcinoma of the prostate with local metastasis found during
surgery, would best be treated with which of the following?

a. Mifepristone

b. Spironolactone

www.damsdelhi.com Email: info@damsdelhi.com


DAMS CBT 2015 Test -1
c. Aminoglutethimide

d. Leuprolide

Solution. Ans 76: (d) Leuprolide.


Reference – Read the text below
Sol:
- Leuprolide is a peptide that is related to GnRH or luteinizing hormone–releasing hormone
(LHRH).
-This agent is used to treat metastatic prostate carcinoma. A hypogonadal state is produced in
the patient from the continuous administration of leuprolide, by its capacity to inhibit
gonadotropin release.
- Testosterone levels in the body become significantly reduced.

Answer. d

77. Which of the following is a stimulant laxative?

a. Lactulose

b. Calcium polycarbophil

c. Bisacodyl

d. Milk of Magnesia

Solution. Ans 77: (c) Bisacodyl


Ref– Read
Sol:
Drugs for constipation:

∙ Bran

∙ Psyllium Bulk-
forming laxatives
∙ Methylcellulose

∙ Calcium polycarbophil

∙ Isapghol
∙ Docusates

Surfactant laxatives ∙ Poloxamers

∙ Lactulose

www.damsdelhi.com Email: info@damsdelhi.com


DAMS CBT 2015 Test -1
∙ Phenolphthalein

∙ Bisacodyl Stimulant
laxatives
∙ Sdium picosulfate

∙ Oxyphenisatin
∙ Magnesium sulfate

∙ Magnesium citrate

∙ Magnesium hydroxide

∙ Milk of Magnesia
Osmotic laxatives
∙ Sodium sulfate

∙ Sodium phosphate

∙ Soidum potassium tartarate

∙ Castor oil
∙ Senna
Anthraquinone derivatives
∙ Cascara sagrada

Answer. c

78. All of the following drugs administered to breastfeeding mothers are safe in therapeutic doses to infants
except –

a. Low dose aspirin

b. Ciprofloxacin

c. Warfarin

d. Methyldopa

Solution. Ans 78: (b) Ciprofloxacin


Ref– Read
Sol:
- Ciprofloxacin achieves high concentration in breast milk and has a theoretical risk of causing
arthropathy in infants.
- Hence, it is to be preferably avoided in breastfeeding mothers

Answer. b

79. The most serious side effect of thalidomide is –


www.damsdelhi.com Email: info@damsdelhi.com
DAMS CBT 2015 Test -1

a. Sedation

b. Constipation

c. Peripheral neuropathy

d. Teratogenecity

Solution. Ans 79: (c) Peripheral neuropathy


Ref– Read
Sol:
- The most serious side effect of thalidomide is peripheral sensory neuropathy.
- It occurs in 10-30% of patients with multiple myeloma or other malignancies in a dose- and
time-dependent manner.
 It is asymmetrical, painful, peripheral paresthesia with sensory loss, commonly presenting with
numbness of toes and feet, muscle cramps, weakness, signs of pyramidal tract involvement and

carpal tunnel syndrome.


- The incidence increases with higher cumulative doses of thalidomide, especially in elderly
patients.

Although symptoms improve upon discontunation, long-standing sensory loss may not reverse.

Answer. c

80. Fat-soluble vitamins generally have a greater potential toxicity compared with water-soluble vitamins
because they are

a. More essential to vital metabolic processes

b. Metabolically faster

c. Avidly stored by the body

d. Administered in larger doses

Solution. Ans 80: (c) Avidly stored by the body


Reference – Read the text below
Sol:
- Fat-soluble vitamins,especially vitamins A and D, can be stored in massive amounts and,hence,
have a potential for serious toxicities.
- Water-soluble vitamins are easily excreted by the kidney and toxic accumulation rarely occurs.

Answer. c

81. Which of the following is a stool softener that has no effect on absorption of fat-soluble vitamins?

www.damsdelhi.com Email: info@damsdelhi.com


DAMS CBT 2015 Test -1

a. Mineral oil

b. Castor oil

c. Docusate sodium

d. Phenolphthalein

Solution. Ans 81: (c) Docusate sodium


Reference – Read the text below
Sol:
- Dioctyl sodium sulfosuccinate (docusate) is a detergent that, when given orally, softens the
stool and prevents straining.
- Mineral oil also softens the stool, but it tends to inhibit the absorption of fat-soluble vitamins
and other nutrients.
- Castor oil, phenolphthalein,and cascara sagrada are strong laxatives and cause watery stools.

Answer. c

82. The anticoagulant activity of warfarin can be potentiated by all of the following except:

a. Rifampin.

b. Aspirin.

c. Phenylbutazone.

d. Cimetidine.

Solution. Ans 82: (a) Rifampin.


Reference: Read the text below
Sol:
- Rifampin induces the hepatic mixed function oxidases that metabolize warfarin.
- Platelet inhibitors, such as aspirin,increase the anticoagulant effect of warfarin.
- Phenylbutazone can transiently increase the level of free warfarin by displacing it from the
plasma albumin binding site.
-Cimetidine inhibits warfarin metabolism and causes potentiation of the anticoagulant.
Disulfiram inhibits warfarin metabolism.

Answer. a

83. Drug having greater activity than ciprofloxacin against pneumococci is

a. Nalidixic acid

www.damsdelhi.com Email: info@damsdelhi.com


DAMS CBT 2015 Test -1
b. Norfloxacin

c. Levofloxacin

d. Ofloxacin

Solution. Ans 83: (c) Levofloxacin.


Reference – Read the text below
Sol:
- Both levofloxacin and ciprofloxacin are quinolones.
- Levofloxacin has greater activity against pneumococci than ciprofloxacin.

Answer. c

84. Which one of the following is the commonest reported side effect of gold therapy?

a. Mouth ulcers

b. Diarrhoea

c. Peripheral neuropathy

d. Aplastic anaemia

Solution. Ans 84: (b) Diarrhoea.


Reference – Read the text below
Sol:
- The most common side effect of oral gold therapy is diarrhea, which may occur with or
without abdominal pain.
- Severe adverse reactions occur in up to 5% of patients and include aplastic anaemia,

proteinuria, pulmonary fibrosis, alopecia, mouth ulcers, peripheral neuritis and cholestatic
jaundice.

Answer. b

85. The longest acting 5-HT3 antagonist is –

a. Ondansetron

b. Palonosetron

c. Granisetron

d. Ramosteron

Solution. Ans 85: (b) Palonosetron


Ref– Read
www.damsdelhi.com Email: info@damsdelhi.com
DAMS CBT 2015 Test -1
Sol:
- Palonosteron is the longest acting 5-HT3 antagonist having highest affinity to the receptor

Answer. b

86. Elevated IgG and IgM antibody titers to parvovirus suggest which of the following diagnosis?

a. Fifth disease

b. Susceptibility to chicken pox

c. Possible subacute sclerosing panencephalitis (SSPE)

d. Possible hepatitis B infection

Solution. Ans 86 : (a) Fifth disease


Ref– Read the text below
Sol:
- Fifth disease is a viral exanthem commonly seen in children 8 to 12 years old.
Children are ill for a few days but recover without incident.
- Unfortunately, if a pregnant female acquires the disease in the first trimester of
pregnancy, the fetus is at risk.
- The causative agent is thought to be a parvovirus (parvovirus B 19).

Answer. a

87. Which of the following tests depends on the presence of protein A on certain strains of Staphylococcus
aureus?

a. Enzyme-linked immunosorbent assay (ELISA)

b. Enzyme multiplied immunoassay test (EMIT)

c. Counterimmunoelectrophoresis (CIE)

d. Coagglutination (COA)

Solution. Ans 87 : (d) Coagglutination (COA)


Ref– Read the text below
Sol:
- Of the many methods available for antigen and antibody detection, LA, ELISA, EMIT, CIE, and COA are the
most widely used.
- Coagglutination (COA), also an agglutination test, is slightly less sensitive than LA but less susceptible to
changes in environment (e.g., temperature).
- Most strains of coagulase-positive staphylococci have protein A in their cell wall. Protein A binds the Fc
fragment of microbial antigens in body fluids.
- COA has also been used to rapidly type or group bacterial isolates.

www.damsdelhi.com Email: info@damsdelhi.com


DAMS CBT 2015 Test -1
Answer. d

88. A male patient presented with granulomatous penile ulcer. On Wright-geimsa stain tiny organisms of 2
microns within macrophages seen. What is the causative organism?

a. LGV

b. Calymmatobacterium granulomatis

c. Neisseria

d. Staph aureus

Solution. Ans-88: (b) Calymmatobacterium granulomatis


Ref: Read the text below
Sol:
- Donovanosis is caused by Klebsiella granulomatis (formerly known as Calymmatobacterium granulomatis),
an intracellular, gram-negative, pleomorphic, encapsulated (when mature) bacterium measuring 1.5 by 0.7
microm.
- K. granulomatis shares many morphologic and serologic characteristics and >99% homology at the
nucleotide level with Klebsiella species that are pathogenic to humans.
- Polymerase chain reaction (PCR) amplification of the phoE gene shows it to be closely related to that in
Klebsiella pneumoniae, K. rhinoscleromatis, and K. ozaenae.
- Electron microscopy shows typical gram-negative morphology and a large capsule but no flagella.
- Filiform or vesicular protrusions occur on a corrugated cell wall.

Answer. b

89. Maltese cross seen on polarizing microscopy in

a. Cryptococcus neoformans

b. Penicillium marneffi

c. Blastomyces

d. Candida albicans

Solution. Ans-89: (a) Cryptococcus neoformans


Ref: Read the text below
Sol:

CAUSES OF MALTESE CROSS ON POLARIZING MICROSCOPY:


- Crptococcus neoformans
- Fabry’s ds
- Paracoccidiodes
- Cholesterol pericarditis

www.damsdelhi.com Email: info@damsdelhi.com


DAMS CBT 2015 Test -1
Answer. a

90. Diagnostic of Rabies

a. Guaneri bodies

b. Negri bodies

c. Cowdry A body

d. Cowdry B body

Solution. Ans-90: (b) Negri bodies


Ref: Read the text below
Sol:
Rabies Antigens or Nucleic Acids
- Tissues infected with rabies virus are currently identified most rapidly and accurately by means of
immunofluorescence or immunoperoxidase staining using antirabies monoclonal antibodies.
- A biopsy specimen is usually taken from the skin of the neck at the hairline. Impression preparations of
brain or cornea tissue may be used.
- A definitive pathologic diagnosis of rabies can be based on the finding of Negri bodies in the brain or the
spinal cord.
- They are sharply demarcated, more or less spherical, and 2–10 micro m in diameter, and they have a
distinctive internal structure with basophilic granules in an eosinophilic matrix.
- Negri bodies contain rabies virus antigens and can be demonstrated by immunofluorescence. Both Negri
bodies and rabies antigen can usually be found in animals or humans infected with rabies, but they are rarely
found in bats.
- Reverse transcription-polymerase chain reaction testing can be used to amplify parts of a rabies virus
genome from fixed or unfixed brain tissue. Although unusual as a diagnostic test, sequencing of amplified
products allows identification of the infecting virus strain.

Answer. b

91. HIV pt with malabsoption, fever, chronic diarrohea, with acid fast positive organism. What is the causative
agent?

a. Giardia

b. E.coli

c. Isospora

d. E.histolytica

Solution. Ans-91: (c) Isospora


Ref: Read the text below
Sol:
Isosporiasis

www.damsdelhi.com Email: info@damsdelhi.com


DAMS CBT 2015 Test -1
- The coccidian parasite Isospora belli causes human intestinal disease. Infection is acquired by the
consumption of oocysts, after which the parasite invades intestinal epithelial cells and undergoes both sexual
and asexual cycles of development.
- Oocysts excreted in stool are not immediately infectious but must undergo further maturation.
- Although I. belli infects many animals, little is known about the epidemiology or prevalence of this parasite
in humans.
- It appears to be most common in tropical and subtropical countries. Acute infections can begin abruptly with
fever, abdominal pain, and watery nonbloody diarrhea and can last for weeks or months.
- In patients who have AIDS or are immunocompromised for other reasons, infections often are not self-
limited but rather resemble cryptosporidiosis, with chronic, profuse watery diarrhea.
- Eosinophilia, which is not found in other enteric protozoan infections, may be detectable.
- The diagnosis is usually made by detection of the large (~25-micro m) oocysts in stool by modified acid-fast
staining.
- Oocyst excretion may be low-level and intermittent; if repeated stool examinations are unrevealing,
sampling of duodenal contents by aspiration or small-bowel biopsy (often with electron-microscopic
examination) may be necessary.

Answer. c

92. Which of the following congenital immunodeficiencies involves only a T-cell deficiency?

a. Thymic aplasia (DiGeogre’s syndrome)

b. Ataxia-telangiectasia

c. X-Linked hypogammaglobulinemia (Brution’s agammaglobulinemia)

d. Chronic granulomatous disease

Solution. Ans-92: (a) Thymic aplasia (DiGeogre’s syndrome)


Ref:Read the text below
Sol:
- Thymic aplasia (DiGeorge’s syndrome) involves a T-cell deficiency caused by failure of both the tymus and
parathyroids to develop properly because of a defect in the third and fourth pharyngeal pouches.
- Ataxia – telangiectasia is an autosomal recessive disease that appears by 2 years of age and involves a
combined B- and T-cell deficiency.
- X-linked hypgammaglobulinemia (Brution’s agammaglobulinemia) is a B-cell deficiency caused by a
mutation in the gene encoding a tyrosine kinase.
- Hereditary angioedema is a complement deficiency caused by an uncommon autosomal dominant disease that
results from a deficiency of C1 esterase inhibitor.
- Chronic granulomatous disease is a phagocyte deficiency caused by an X-linked disease in most cases (in
some patients the disease is autosomal) that appears by the age of 2 years.

Answer. a

93. Which of the following congenital variable immunodeficiencies involves a combined B- and Tcell
deficiency?

www.damsdelhi.com Email: info@damsdelhi.com


DAMS CBT 2015 Test -1
a. Hyper-IgM syndrome

b. Wiskott-Aldrich syndrome

c. Chronic mucocutaneous candidiasis

d. Chediak-Higashi syndrome

Solution. Ans-93: (b) Wiskott-Aldrich syndrome


Ref:Read the text below
Sol:
- Hyper-IgM syndrome and chronic mucocutaneous candidiasis also involve T-cell deficiency. Hyper-IgM
syndrome results from a defect inhelper T cells in a surface protein that interacts with the CD40 antigen on
the B-cell surface.
- Chronic mucocutaneous candidiasis results from a T-cell deficiency specifically for candida albicans; other
T- and B-cell functions are normal.
- Wiskott-Aldrich syndrome also involves a combined B- and T-cell deficiency where B cell numbers are
normal but antibody responses to polysaccharide antigens are absent and T-cell deficiencies are variable. -
Chediak-Higashi syndrome and leukocyte adhesion deficiency syndrome are phagocyte deficiencies.
- Chediak-Higashi syndrome is an autosomal recessive disease while leukocyte adhesion deficiency
syndrome is caused by a defective adhesion (LFA-1) protein on the surface of their phagocytes.

Answer. b

94. All of the following are important virulence factors in meningococci except

a. Polysacharide capsule

b. M Protein

c. Endotoxin

d. IgA protease

Solution. Ans-94: (b) M Protein


Ref:Read the text below
Sol:
- M protein is an antiphagocytic surface component that is not produced by meningoccoci.
- Virulence factors produced by meningococci include the polysaccharide capsule, which enables the
organism to resist phagocytosis by leukocytes; endotoxin, which causes fever, shock, and other
pathophysiologic changes; and IgA protease, which cleaves secretory IgA, allowing the bacteria to attach to
the membranes of the upper respiratory tract.

Answer. b

95. Which of the following infectious diseases is caused by a spirochete?

www.damsdelhi.com Email: info@damsdelhi.com


DAMS CBT 2015 Test -1
a. Lyme disease

b. Yellow fever

c. Typhus

d. Dengue fever.

Solution. Ans-95: (a) Lyme disease


Ref:Read the text below
Sol:
- Three genera of spirochetes can cause human infections: Treponema, which causes syphilis; Leptospira,
which causes leptospirosis; and Borrelia, which causes a number of tick-borne disease.

- Lyme disease is caused by the Borrelia burgdorferi spirochete. Upon Giemsa stain, B. burgdorferi appear as
large, loosely coiled organisms.
- Lyme disease is disease is treated with tetracycline or amoxicillin for acute infections, and penicillin for
chronic infections.
- Dengue fever is caused by the dengue virus, and yellow fever is caused by the yellow fever virus; both of
these viruses are transmitted by the Aedes aegypti mosquito.
- Typhus is caused by the bacteria Rickettsia typhi and is transmitted by flea bites.

Answer. a

96. Coxsackie group A commonly causes:

a. Conjunctivits

b. Aseptic meningitis

c. Hepatitis

d. Myocarditis

Solution. Ans-96: (b) Aseptic meningitis


Ref: Read the text below
Sol:
- Herpangina (Vesicular pharyngitis) – Group A coxsackie virus.
- Aseptic meningitis – Group A and all group B coxsackie virus -
Myocarditis and pericarditis – Group B.
- Juvenile Diabetes – Coxsackie B
- Epidemic pleurodynia or Bornholm disease a Group B virus.

Answer. b

97. Which of the following is false regarding sporotrichosis?

a. It spreads by lymphatics
www.damsdelhi.com Email: info@damsdelhi.com
DAMS CBT 2015 Test -1

b. Caused by S. schenkii

c. Medler bodies

d. Potassium iodide is drug of choice

Solution. Ans-97: (c) Medler bodies


Ref.: Ananthanarayan, Paniker CJ. Textbook of Microbiology. 6th Ed.
Sol :
- Medlar bodies are seen in chromoblastomycosis.
- They are globe-shaped, cigar – colored, thick-walled structures that are 4-12 um in diameter.
- The cells are characteristically divided in several planes of division by thick septa.
- Medlar bodies are also known as sclerotic bodies, copper penny bodies or muriform bodies.

Answer. c

98. Primary amoebic encephalitis is caused by :

a. Nageleria fowleri

b. Acanthameba spp.

c. Entamoeba hartmanni

d. Balmuthia mandrillaris

Solution. Ans-98 : (a) Nageleria fowleri


Ref.: Ananthanarayan- 6th Ed.
Sol :
- Naeglaria fowleri causes Primary Amebic Mening-oencephalitis (PAM). Infection is acquired by
swimming or diving in warm water contaminated with trophozoite and cysts.
- The trophozoites are neurotrophic → It ingests RBCs and brain tissue by phagocytosis → Production of
amoebostome with cytopathic enzymes → Acute hemorrhagic
meningoencephalitis.
Primary Amoebic meningoencephalitis
- PAM mimics acute bacterial meningitis
- Abrupt in onset with acute fulminant course
- Condition mostly fatal

Answer. a

99. All of the following nephritides are associated with hypocomplementemia except

a. Immunoglobulin A (IgA) nephropathy

www.damsdelhi.com Email: info@damsdelhi.com


DAMS CBT 2015 Test -1
b. Mesangioproliferative Glomerulonephropathy

c. Serum sickness

d. Systemic lupus erythematosus (SLE)

Solution. Ans-99: (a) Immunoglobulin A (IgA) nephropathy


Ref:Read the text below
Sol:
- Complement levels are normal in immunoglobulin A (IgA) nephropathy and diffuse proliferative
glomerulonephritis (poststreptococcal glomerulonephritis).
- Nephritides associated with hypocomplementemia include cryoglobulinemia,
membranoproliferative glomerulonephropathy, and a variety of visceral infections, includeing infections of
peritoneal and central nervous system (CNS) shunts (“shunt” nephritis).

Answer. a

100. All of the following statements describe the genetic code except

a. It is nearly identical for all organisms

b. It is composed of nucleotides containing three nucleotide code letters

c. It represents all of the nucleotide sequence information within a transcription unit


d. It contains transcription start and stop sequences

Solution. Ans-100: (c) It represents all of the nucleotide sequence information within a transcription unit
Ref:Read the text below
Sol:
- Three nucleotides are required to specify the insertion of an amino acid into a polypeptide chain.
- These groups of three nucleotides comprise a codon that is represented in the 5’ to 3’ direction. -
Because there are four different bases in RNA, the maximum number of codons is sixty-four.
- Sixty-one of these codons specify the twenty amino acids; some amino acids have more than one.
- The triplet AUG serves as a start signal, and three triplets that do not code for any amino acid serve as stop
signals.
- The genetic code is virtually universal; all organisms use the same codons to translate their genomes into
proteins.
- A transcription unit can be influenced by promoter and enhancer elements as well as methylation of
nucleotides.

Answer. c

101. Proof the presence of active disease caused by Mycobacterium tuberculosis is provided by which one of
the following diagnostic measures?

a. The tuberculin test

b. Clinical findings (e.g. weight loss, night sweats, cough, low-grade fever)
www.damsdelhi.com Email: info@damsdelhi.com
DAMS CBT 2015 Test -1

c. Finding acid-fast organisms in sputum

d. Isolation of M. tuberculosis

Solution. Ans-101: (d) Isolation of M. tuberculosis


Ref:Read the text below
Sol:
- Isolation of Mycobacterium tuberculosis is diagnostic of active tuberculosis.
- The tuberculin test can be positive in the absence of active disease.
- The clinical findings are not specifically pathognomonic for tuberculosis, nor is demonstration of acid-fast
organisms.

Answer. d

102. Infection associated with skin penetration includes all except:

a. Strongloides

b. Nectar americans

c. Ancylostoma

d. Dracunculus

Solution. Ans-102: (d) Dracunculus


Ref: Harrison 17th Ed. 1329
Sol:
- Dracunculus medinensis (Serpent worm/Dragon worm) causes Dracunculosis,
- Human acquire this infection when they ingest water containing infective larvae derived from Cyclops.

Soil to skin transmission


1. Ankylostoma duodenale (Hook worm)
2. Necator Americans
3. Strongyloides stercoralis

Answer. d

103. Rat flea is vector for all the following except:

a. Endemic typhus

b. Plague

c. Leptospirosis

d. Epidemic typhus

www.damsdelhi.com Email: info@damsdelhi.com


DAMS CBT 2015 Test -1
Solution. Ans-103: (d) Epidemic typhus
Ref: Park PSM, 19th ed. 562.
Sol:
- Rat flea is an arthropod responsible as a vector for transmission of diseases like:
- Bubonic plague
- Endemic typhus
- Leptospirosis.

Answer. d

104. Tunica reaction is seen in:

a. H. influenza

b. Plague

c. R. mooseri

d. B. anthrax

Solution. Ans-104: (c) R. mooseri


Ref: Textbook of Microbiology Ananthanarayan -- 383)

Sol:
- R. mooseri (R. typhi) gives positive Neill-Mooseri/tunica reaction. R. prowazekii gives negative reaction.
- When male guinea pigs are inoculated intraperitoneally with blood from a case of endemic typhus or with a
culture of R. typhi, they develop fever and characteristic scrotal inflammation.
- The scrotum becomes enlarged and testes cannot be pushed back into the abdomen because of inflammatory
adhesions between layers of tunica vaginalis.

Answer. c

105. Madura mycosis produces:

a. Brown to black granules

b. White to yellow granules

c. Red granules

d. No granules

Solution. Ans-105: (a) Brown to black granules

Ref:Textbook of Microbiology Ananthanarayan– 571)

Sol:

www.damsdelhi.com Email: info@damsdelhi.com


DAMS CBT 2015 Test -1
- Madura mycosis/Mycetoma/Madura foot is chronic slowly progressive infection of
subcutaneous tissue usually of foot. Actinomycetes and filamentous fungi may cause it.
- It is characterized by swelling of foot with multiple discharging sinuses.
- Granules and seropurulent fluid comes out of sinuses.
- In actinomycotic mycetoma (Actinomycetes, Nocardia) the grains contains thin filaments and are yellow in
color.
- In mycotic lesions filaments are broader and show septae and chlamydospores. Granules are brown to black
color.
- Actinomycotic lesions may respond to sulphonamides but mycotic lesions are resistant and may require
amputation.

Answer. a

106. Acute arsenic poisoning is confused with?

a. Ureteric colic

b. Acute gastroenteritis

c. Malaria

d. Diptheria

Solution. Ans-106: (b) Acute gastroenteritis


Ref.: Read the text below
Sol :
Acute arsenic poisoning
- Symptoms usually start within 30 minutes to 2 hours.
- Acute arsenic ingestion is typically followed by a severe gastroenteritis, garlic odour and hypersalivation.
- There is a characteristic sequence of multi-organ failure, with: neurological symptoms (within hours) and
cardiac features, succeeded by adult respiratory distress syndrome and renal/liver dysfunction.
- Marrow suppression develops after a few days to weeks in survivors, as does alopecia and an ascending
motor neuropathy.

Answer. b

107. Grievous injury comes under which section of IPC?

a. Section 319

b. Section 320

c. Section 324

d. Section 326

Solution. Ans-107: (b) Section 320

www.damsdelhi.com Email: info@damsdelhi.com


DAMS CBT 2015 Test -1
Ref.: Read the text below
Sol :
The differences between hurt and grievous hurt can be better shown by the tabular form :-

Answer. b

108. Kunkel’s test is done to demonstrate the presence of

a. Lead

b. CuS04

c. CO

d. Dhatura

Solution. Ans-108: (c) CO


Ref.: Read the text below
Sol :

www.damsdelhi.com Email: info@damsdelhi.com


DAMS CBT 2015 Test -1

Answer. c

109. “Ewing’s postulates” refer to?

a. Accidents as a cause by birth

b. Complication resulting from trauma

c. Circulation, respiration and brain functions

d. Circulation, respiration and excretion

Solution. Ans-109: (b) Complication resulting from trauma


Ref.: Read the text below
Sol :
- Ewing's postulate showing relationship between trauma and new growth
- The pathophysiology of burn scar neoplasms is clearly distinct from that of regular skin cancer. Ewing's
postulate requires: (1) evidence of a burn scar; (2) a tumor within the boundaries of the scar; (3) no previous
tumor in that location; (4) tumor histology that is compatible with the cell types found in the skin and the scar;
and (5) an adequate interval between the burn injury and tumor development

Answer. b

110. Bicycle spoke injuries are most commonly seen in:

www.damsdelhi.com Email: info@damsdelhi.com


DAMS CBT 2015 Test -1

a. Eyes

b. Hands

c. Foot

d. Anywhere

Solution. Ans-110: (c) Foot


Ref: Read the text below
Sol :
- Bicycle spoke injuries occur when the foot of a passenger (usually a child) is caught in the spokes of a
rotating wheel. (1)Cause –The dangling foot gets caught accidentally between the moving spokes of the
bicycle.
- (2)Usual injuries are: (1)abrasions (2)lacerations and (3)crushing of the foot. (4)Fractures - Supramalleolar
fracture. Greenstick fracture of the distal end of the tibia and fibula.

Answer. c

111. Which of the following facts is true about Subpoena duces tecum ?

a. It compels a witness to give evidence and also bring with him certain documents in his possession

b. It means the medical witness can send reply by post

c. It means the medical witness can be examined via telephone, this being used only for very busy medical
practitioners

d. The court officially personally accompanies medical witness to the court

Solution. Ans-111: (a) It compels a witness to give evidence and also bring with him certain documents in his
possession
Ref: Read the text below
Sol :
- Subpoena is of two types: (i) Subpoena ad testificandum: It compels a witness to attend and give evidence and
(ii) Subpoena duces tecum: It compels a witness to give evidence and also bring with him certain documents in
his possession (usually the post-mortem or the medicolegal report) specified in the subpoena.
- In Indian law, the service of Subpoena ad testificandum is guided by section 61 till section 69 of Criminal
Procedure Code (Cr.P.C.), while the service of Subpoena duces tecum is guided by section 91 of Cr.P.C.

Answer. a

112. Dry wine is

www.damsdelhi.com Email: info@damsdelhi.com


DAMS CBT 2015 Test -1
a. Methylated spirit

b. Methyl alcohol

c. Opium

d. Chloral Hydrate

Solution. Ans-112: (d) Chloral Hydrate


Ref: Read the text below
Sol :
Chloral hydrate is an unapproved sedative and hypnotic drug as well as a chemical reagent and precursor.
Long-term use of chloral hydrate is associated with a rapid development of tolerance to its effects and
possible addiction as well as adverse effects including rashes, gastric discomfort and severe renal, cardiac

and hepatic failure

Answer. d

113. Ashley’s rule deals with:

a. Sexing of skull

b. Sexing of sterum

c. Determination of height from long bones

d. Determination of race from skull

Solution. Ans-113: (b) Sexing of sterum


Ref: Read the text below
Sol :
- For sexing the European sterna Ashley formulated the “149 rule” according to which a male sternum
exceeded 149 mm in length, whereas the female sternum was less than 149 mm.

Answer. b

114. Venom of sea snake is mostly:

a. Neruotoxic

b. Hemolytic

c. Myotoxic

d. Hepatotoxic

Solution. Ans- 114: (c) Myotoxic


Ref: Read the text below
www.damsdelhi.com Email: info@damsdelhi.com
DAMS CBT 2015 Test -1
Sol :
The venom of colubrine snakes is mainly neurotoxic, that of viperine snakes mainly hemolytic and that of sea
snakes mainly myotoxic.
Nature of poison and main symptoms produced by various classes of snakes

In general, while hemotoxic bites are more painful, neurotoxic venom are more deadly.

Answer. c

115. Which poisonous acid is present in rhubarb leaves, sorrel, spinach and tomato?

a. Phenol

b. Oxalic acid

c. Citric acid

d. Muriatic acid

Solution. Ans- 115: (b) Oxalic acid


Ref: Read the text below
Sol :
- It is present in the form of calcium oxalate.
- Oxalates may also be produced by several common molds.
- Penicillium and Aspergillus molds can convert sugar into calcium oxalate at very high yields.

Answer. b

116. Identify this scientist-

www.damsdelhi.com Email: info@damsdelhi.com


DAMS CBT 2015 Test -1

a. John snow

b. Louis Pasteur

c. Robert Koch

d. James lind

Solution. Ans-116: (c) Robert Koch


Ref: Read the text below
Sol:
- Robert Koch was a celebrated German physician and pioneering microbiologist.
- The founder of modern bacteriology, he is known for his role in identifying the specific causative agents of
tuberculosis, cholera, and anthrax and for giving experimental support for the concept of infectious disease.
- In addition to his trail-blazing studies on these diseases, Koch created and improved laboratory technologies
and techniques in the field of microbiology, and made key discoveries in public health. [5] His research led to
the creation of Koch’s postulates, a series of four generalized principles linking specific microorganisms to
specific diseases that remain today the "gold standard" in medical icrobiology.[5] As a result of his
groundbreaking research on tuberculosis, Koch received the Nobel Prize in Physiology or Medicine in 1905

Answer. c

117. JSY stands for:

a. Janani suraksha yojna

b. Jeevan swastha yojna

c. Jan sewa yojna

d. Jan suraksha yojna

Solution. Ans-117: (a) Janani suraksha yojna


Ref: Read the text below

www.damsdelhi.com Email: info@damsdelhi.com


DAMS CBT 2015 Test -1
Sol:
- JSY stands for Janani Suraksha Yojna
- 100% centrally sponsored scheme offering maternity benefit
- Focuses on reducing IMR and MMR and promoting institutional deliveries

Answer. a

118. Which of the following is not an essential component of primary health care:

a. Provision of essential drugs

b. Cost effectiveness

c. Immunization against major vaccine preventable disease

d. Health education

Solution. Ans-118: (b) Cost effectiveness


Ref: Read the text below
Sol:
Essential components of primary health care include:
i. Education concerning prevailing health problems and methods for preventing and
controlling them
ii. Promotion of food supply and proper nutrition
iii. Adequate safe water supply and basic sanitation
iv. Maternal and child health including family planning
v. Immunization against vaccine preventable disease
vi. Prevention and control of locally endemic disease
vii. Appropriate treatment of common diseases
viii. Provision of essential drugs

Answer. b

119. The prevalence of Candida glabrata infection is found to be 80% in a population of 100. If the test is
repeated for 95% confidence levels, then what will be the range of prevalence for the Candida glabrata
infection?

a. 70 to 90

b. 72 to 88

c. 65 to 95

d. 4 to 100

Solution. Ans-119: (b) 72 to 88


Ref: Read the text below

www.damsdelhi.com Email: info@damsdelhi.com


DAMS CBT 2015 Test -1
Sol:
The range obtained by repeating a test is given by the formula:
p ± 2 √p (100-p) / n
where,
p = prevalence of the study as established from previous studies
n = sample size used for the present study

In this particular example,


p = 80
n = 100.

Hence, the range estimated would be =


80 ± 2 √80 (100-80) / 100
= 80 ± 2 x 4
= 80 ± 8
= 72 to 88

Answer. b

120. The prevalence of disease in a population is expected to be 50%. An investigator is interested in


assessing the prevalence in the range of 45 to 55% with 95% confidence. The minimum sample size required
for the study is –

a. 100

b. 200

c. 300

d. 400

Solution. Ans-120: (d) 400


Ref: Read the text below
Sol:
The formula to calculate sample size is –
N = 4pq/d2,
where
N = required sample size
p = prevalence of the disease as estimated from previous studies
q = 100-p
d = Error allowed in the study = (100-confidence interval)

In this example,
p = 50%

d = 100-95 = 5 Therefore,
q = 100-50 = 50%
www.damsdelhi.com Email: info@damsdelhi.com
DAMS CBT 2015 Test -1

5
0

5
0

5
2
= 400

www.damsdelhi.com Email: info@damsdelhi.com


DAMS CBT 2015 Test -1

Answer. d

121. In a district in India with a population of 10 lakhs, 50% of the population is under 16 years of age. If the
prevalence of blindness is 0.8/1000 in under 16-years population in the district, then how many blind people
are there in the district?

a. 400

b. 720

c. 2400

d. 7000

Solution. Ans-121: (a) 400


Ref: Read the text below
Sol:
Total population is 10 lakhs, out of which 50% is under 16 years of age.
Hence, the population of under-16 years age group in the district is –
50 x 10,00,000 = 5,00,000
100
Now, the prevalence of blindness in the district is 0.8/1000.

Therefore, the actual population of blind people in the district is –


0.8 x 5,00,000 = 400
1000

Answer. a

122. In a population of 5000, on 1st January, total cases of myopia were 500. Till 31st December, 90 new
cases were found. Calculate the incidence of myopia PER 100.

a. 1.8

b. 2

c. 5

d. 18

Solution. Ans-122: (b) 2


Ref: Park’s ‘Textbook of Preventive and Social Medicine’; 21/e, pg 57
Sol:
Incidence = Number of new cases of a specified disease during a
given time period x 100
Population at risk during that period

www.damsdelhi.com Email: info@damsdelhi.com


DAMS CBT 2015 Test -1
Population at risk= 5000-500= 4500
Incidence of myopia during the year in the population would be –
90 x 100 = 2 per 100.
4500

Answer. b

123. In a certain population, there were 4050 births in the last one year. There were 50 still births. 50 infants
died within 7 days where as 150 died within 28 days. What is the Neonatal mortality rate?

a. 50

b. 62.5

c. 12.5

d. 49.4

Solution. Ans-123: (a) 50


Ref: Read the text below
Sol:
Total number of births in one year = 4050
No. of still births in that year = 50
Hence, total number of live births in that year would be
= 4050 – 50
= 4000
Total number of infants dying in the first 7 days = 50
Total number of infants dying in the next 21 days = 150
Hence, total number of infants dying in the first 28 days of birth
= 50 + 150
= 200
Now,
Neonatal mortality rate = Number of deaths of children under
28 days of age in a year x 1000
Total number of live births in the
same year
= 200 x 1000
4000
= 50

Answer. a

124. The post-operative quality of life (QOL) scores of 200 prostate cancer patients has a mean of 60 and a
standard deviation of 10. How many patients are expected to have a QOL score between 40 and 80?

a. 190

b. 136
www.damsdelhi.com Email: info@damsdelhi.com
DAMS CBT 2015 Test -1

c. 120

d. 140

Solution. Ans-124: (a) 190

Ref: Read the text below


Sol:
The values enclosed in a normal distribution curve are:

In the above question,


Mean = 60
SD = 10

Hence, the range of values from 40 to 80 i.e. 60 ± 20 (Mean ± 2SD) would include 95% of the readings.
Therefore, total number of readings in the given range =
95x 200 = 190
100
Hence, 190 patients are expected to have a QOL score between 40 and 80.

Answer. a

125. Chandlers index is 225,what is the interpretation

a. Potential danger to community

b. No danger

c. Minor public health problem

d. Major public health problem

Solution. Ans 125: (a) Potential danger to community


Ref– Read the text below
Sol:
Morbidity and mortality from hookworm infection depend much on the worm load. Chandler worked out an
index on the basis of an average number of hookworm eggs per gram of feces for the entire community, which
is as below:
Average number of eggs per gram of stool

Chandler’s index is still used in epidemiological studies of hookworm disease. By this index, worm loads in
different population groups can be compared and also the degree of reduction of egg output after mass

www.damsdelhi.com Email: info@damsdelhi.com


DAMS CBT 2015 Test -1

treatment

Answer. a

126. Prepatent period’ in lymphatic filariasis is defined as the time interval between incoculation of infective
larvac and

a. Blockage of lymphatics

b. First appearance of detectable microfilaria

c. Development of lymphoedema

d. Development of adult worm

Solution. Ans 126: (b) First appearance of detectable microfilaria


Ref– Read the text below
Sol:
- Mosquitoes, which bite infected individuals, can take up these circulating microfilariae. Within the mosquito,
these embryonic larvae develop into second then third stage larvae over a period of 10 to 14 days. The
mosquito is then ready to bite and infect a new human host, thereby completing the life cycle.
- The interval between acquisition of infective larvae from a mosquito bite and detection of microfilariae in
the blood is known as the prepatent period. This interval is usually approximately 12 months in duration.

Answer. b

127. With statement to the epidemiology of severe Acute Respiratory syndrome (SARS), which one of the
following statements is not correct

a. Patient is most infectious during the first five days of illness

b. Morbidity and mortality rates are greater in elderly patients

c. SARS-CoV infection has no predilection for any age group

d. Both sexes are equally affected.

Solution. Ans 127: (a) Patient is most infectious during the first five days of illness
Ref– Read the text below
Sol:
- Severe acute respiratory syndrome (SARS) is a serious, potentially life-threatening viral infection caused by
www.damsdelhi.com Email: info@damsdelhi.com
DAMS CBT 2015 Test -1
a previously unrecognized virus from the Coronaviridae family. This virus has been named the SARS-
associated coronavirus (SARS-CoV)
- Anyone who has close personal contact with a person with known or suspected SARS within 10 days of
symptom onset (eg, health care workers, family members, caregivers) is at high risk of SARS-CoV infection.
- Both sexes are equally affected.
- SARS-CoV infection has no predilection for any age group; however, as stated above, morbidity and
mortality rates are greater in elderly patients.

Answer. a

128. HIV sentinel surveillance provides data –

a. To monitor disease trends

b. To identify high risk population

c. To estimate disease incidence

d. To assess the quality of related services

Solution. Ans-128: (a) To monitor disease trends


Ref: Park’s ‘Textbook of Preventive and Social Medicine’; 21/e, pg 398
Sol:
- After the establishment of the fact that the HIV infection is present in wide geographic areas, the objective
of the sentinel surveillance was redefined to monitor the trends of HIV infection.
- The objective of the surveillance is best achieved by annual cross-sectional survey of the risk group in the
same place over a few years by unlinked anonymous serological testing procedures by two ERS (i.e. when
HIV testing is carried out without identification of name of samples collected for other purposes such as
VDRL in STD clinics.
- The number of samples to be screened must represent the risk group under study and the sample size is
determined accordingly.
- Although the HIV sentinel surveillance data has been primarily used for monitoring the trends, i.e. to assess
how rapidly HIV infection increases or decreases over the time in different groups and areas, it can also
provide an estimate of the total burden of HIV infection in the country.

Answer. a

129. 10% of total deaths in a city is due to accidents. There are 500 total deaths reported. What is true?

a. 10% is case fatality rate.

b. Out of total deaths 50 are due to accidents.

c. Both of these.

d. None of these.

www.damsdelhi.com Email: info@damsdelhi.com


DAMS CBT 2015 Test -1
Solution. Ans-129: (b) Out of total deaths 50 are due to accidents
Ref: Read the text below
Sol:
- Proportional Mortality Rate ( PMR)
Proportion mortality rate (TB) = No. of death due to TB
TOTAL NO. OF DEATHS
Indicate Burden of a disease in terms of mortality.
- Most common cause of death is given by PMR.
Ex- 10% of total deaths is due to accidents. So out of 500 deaths 50 are due to accidents.

Answer. b

130. Out of 100 cases of Japanese encephalitis 30 will die. What is false?

a. 30% is case fatality rate.

b. Case fatality rate will measure virulence of a disease.

c. 70% will be the survival rate.

d. None of these.

Solution. Ans-130: (d) None of these.


Ref: Read the text below
Sol:
- CASE FATALITY RATE ( CFR) -
‘ killing power of a disease

- virulence of organism
- CFR= deaths x 100
cases
- PROPORTION
- CFR =1- Survival Rate
- Used in acute infections_

Answer. d

131. "Country health programming" approach is related with :-

a. UNICEF

b. SIDA

c. Ford foundation

d. Junior red cross

Solution. Ans 131: (a) UNICEF


www.damsdelhi.com Email: info@damsdelhi.com
DAMS CBT 2015 Test -1
Ref– Read the text below
Sol:
UNICEF
- Greater attention is being given to the concept of the "Whole child" meaning that assistance should hence
forward be geared not only to health & nutrition, as before, which are of immediate benefit to children, but
also to their long-term personnel development and to the development of the countries in which they live.
- This approach is also known as 'Country health programming'.

Answer. a

132. Bagassosis is most likely caused due to the inhalation of the dust of :-

a. Free silica

b. Coal

c. Sugar cane

d. Cotton fibre

Solution. Ans 132: (c) Sugar cane


Ref– Read the text below
Sol:
- A self-limited lung disease caused by an allergic response to bagasse, the fungi-laden, dusty debris left after
the syrup has been extracted from sugarcane.
- It is characterized by fever, dyspnea, and malaise.

Answer. c

133. Under Insecticide Treated Bed Nets Programme (ITN), insecticide used is:

a. Malathion

b. Fenitrothion

c. Deltamethrin

d. Benzene hexachloride (BHC)

Solution. Ans 133: (c) Deltamethrin


Ref– Read the text below
Sol:
Insecticide Treated Bed Nets Programme (ITN)
- Non-severe malaria episodes cause flu-like symptoms, where severe episodes can lead to death. Malaria is
one of the leading causes of mortality for children under five in the developing world .
- Insecticide-treated nets (including bednets and curtains) can kill and repel mosquitoes, reducing risk of
infection.
- Nets: There are two types of nets used: insecticide-treated nets (ITNs), which require retreatment every 6 to
www.damsdelhi.com Email: info@damsdelhi.com
DAMS CBT 2015 Test -1
12 months,and long-lasting insecticidal nets (LLINs), which are designed to last for 4-5 years.
- Under Insecticide Treated Bed Nets Programme (ITN), insecticide used is Deltamethrin

Answer. c

134. With reference to meningococcal meningitis, which one of the following statement is not correct.

a. Fatality in untreated cases is 90 percent

b. Disease spreads mainly by droplet infection epidemiological pattern of disease

c. Mass chemoprophylaxis causes immediate drop in the incidence rate of cases.

d. Geographic distribution and epidemic capabilities differ according to the serogroup.

Solution. Ans 134: (a) Fatality in untreated cases is 90 percent


Ref– Read the text below
Sol:
- Meningococcal meningitis is a bacterial form of meningitis, a serious
infection of the meninges that affects the brain membrane. It can cause severe brain damage and is fatal in
50% of cases if untreated.
- Several different bacteria can cause meningitis. Neisseria
meningitidis is the one with the potential to cause large epidemics.
- Disease spreads mainly by droplet infection epidemiological pattern of
disease
- Twelve serogroups of N. meningitidis have been identified, five of
which (A, B, C, W135, and X) can cause epidemics.
- Geographic distribution and epidemic capabilities differ according to
the serogroup.

Answer. a

135. Standardized mortality rate is

a. Standardized for age

b. Standardized for disease

c. Standardized for regions

d. Standardized for a particular time period

Solution. Ans 135: (a) Standardized for age


Ref– Read the text below
Sol:
- The standardized mortality ratio or SMR in epidemiology is the ratio of
observed deaths to expected deaths according to a specific health outcome in a population and serves as an
indirect means of adjusting a rate.

www.damsdelhi.com Email: info@damsdelhi.com


DAMS CBT 2015 Test -1
- The figure for observed deaths is usually obtained for a particular
sample of a population. The figure for expected deaths reflects the number of deaths for the larger population
from which the study sample has been taken e.g. national level of mortality attributed to a particular health
outcome.
- The calculation used to determine the SMR is simply: number of
observed deaths/number of expected deaths.

Answer. a

136. A patient presents with fever, sore throat and difficulty to swallow. His radiograph shows thumb sign.
What is the most likely diagnosis?

a. TB larynx

b. Croup

c. Acute Epiglottitis

d. Laryngomalacia

Solution. Ans 136: (c) Acute Epiglottitis


Ref– Read the text below
Sol:

∙ In acute epiglottitis, the epiglottis is swollen and this finding on lateral soft tissue X Ray is known as
“thumb sign”.

∙ Absence of a deep well defined vallecula, known as vallecula sign is also seen in acute epiglottitis.

Answer. c

137. Perilymph is the fluid between the membranous and bony labyrinth. Perilymph is rich in which ion?

a. Sodium

b. Potassium

c. Calcium

d. Magnesium

Solution. Ans 137: (a) Sodium


Ref– Read the text below
Sol:

The two main fluids in inner ear are perilymph and endolymph
PERILYMPH
- .Resembles ECF, rich in sodium ions

www.damsdelhi.com Email: info@damsdelhi.com


DAMS CBT 2015 Test -1
.Fills the space between bony and memnranous labyrinth
. Communicates with CSF (subarachnoid space) through aqueduct of cochlea, Which
opens into scala tympani near the round window
. Is an ultrafiltrate of blood serum
ENDOLYMPH
- Resembles ICF, rich in potassium ions
- Secreted by secretory cells of stria vascularis of cochlea and by the dark cells in utricle and near
the ampulla of semicircular canal.

Answer. a

138. Identify the instrument:

a. Luc’s forceps

b. Mollison’s mastoid retractor

c. Eve’s Tonsil Snare

d. Walsham forceps

Solution. Ans 138: (b) Mollison’s mastoid retractor


Ref– Read the text below
Sol:
- Mollison’s self Retaining Hemostatic Mastoid Retractor
- Used to retract soft tissues after incision and elevation of flaps, in mastroid surgeries
- Advantages are:
1. Retracts the soft tissue
2. Allows both hands to be free
3. Stops bleeding.

Answer. b

139. Pure tone audiometry is a subjective method for assessment of hearing. In a pure tone audiogram,
Carhart’s notch in audiogram is seen in:

www.damsdelhi.com Email: info@damsdelhi.com


DAMS CBT 2015 Test -1

a. Otosclerosis

b. Menier’s disease

c. Otitis media

d. Carcinoma middle ear

Solution. Ans 139: (a) Otosclerosis


Ref– Read the text below
Sol:
- Carhart’s notch is a dip in bone conduction at 2000HZ. It is seen in otosclerosis.
OTOSCLEROSIS
- Autosomal dominant pattern of inheritance (50% of cases have positive family history)
- Commoner in whites and Indians.
- More common in females
- Age group:20-30 years
- Usually bilateral(unilateral in 15% cases)
- Most common site affected is fissula ante fenestrum(anterior to oval window)
- There is a suspected etiological association with measles virus.

Answer. a

140. Most common complication following FESS is:

a. Synechiae

b. Diplopia

c. Meningitis

d. CSF rhinorrhea

Solution. Ans 140: (a) Synechiae


Ref– Read the text below
Sol:
- Synechiae are considered the most common complication overall and occur in upto 8% of the
individuals following FESS.
- Complication rates of FESS are similar to that of conventional surgeries.
- Most of the complications are minor and include synechiae, orbital injury, emphysema, minor hemorrhages
- The most common major complication of FESS is CSF leak (1.4%), followed by major hemorrhages,
nasolacrimal duct injury, meningitis and blindness.

Answer. a

141. Most common cause of orbital cellulitis is:

www.damsdelhi.com Email: info@damsdelhi.com


DAMS CBT 2015 Test -1
a. Penetrating injuries

b. Sinusitis

c. Dacryocystitis

d. Periorbital surgeries

Solution. Ans 141: (b) Sinusitis


Ref– Read the text below
Sol:
Sinusitis remains the most common cause of orbital cellulitis. Of orbital infections, 60-70% arise
secondary to sinus infections.

ORBITAL CELLULITIS
- Source of infection is mostly Ethmoidal sinus > Maxillary sinus

Answer. b

142. Most common site of sinonasal melanoma is:

a. Frontal sinus

b. Maxillary sinus

c. Ethmoid sinus

d. Nasal septum

Solution. Ans 142: (d) Nasal septum


Ref– Read the text below
Sol:
Most sinonasal melanomas occur on the anterior nasal septum and middle & inferior turbinates.
SINONASAL MELANOMA
- More common in women, more in elderly (5 th to 8th decade)
- Most common site of sinonasal melanoma is: anterior nasal septum> middle and inferior
turbinates, followed by oral cavity usually the palatine mucosa.
- May present as a sessile or polypoidal mass or ulceration, may not be pigmented always.
- Cervical LN mets are less frequent, but more often metastasise to lungs and brain.

Answer. d

143. Which of the following is the most important risk factor for Laryngeal carcinoma?

a. Smoking

b. Alcohol

www.damsdelhi.com Email: info@damsdelhi.com


DAMS CBT 2015 Test -1
c. Occupational

d. Laryngopharyngeal reflux.

Solution. Ans 143: (a) Smoking


Ref– Read the text below
Sol:
- Tobacco has been identified as the main causative agent in Ca larynx, with upto 98% of the patients being
smokers
- All the given options are risk factors for Ca larynx, most important being smoking.
- Occupational risks are mainly – asbestos, mustard gas, wood dust, cement dust, tar, and petroleum products.
- Other risk factors include alcohol, previous radiation, genetic factors, HPV (16& 18),
Laryngopharyngeal reflux

Answer. a

144. All are true about exostosis of the auditory meatus except?

a. Often multiple and bilatera

b. Mostly malignant

c. Often seen in persons frequently exposed to cold water

d. Most common in males

Solution. Ans 144: (b) Mostly malignant


Ref– Read the text below
Sol:
Exostoses and Osteomas are mostly asymptomatic. These lesions have no malignant potential and their
growth rate can be exceedingly slow.
EXOSTOSIS OF EXTERNAL EAR
- It is the most common benign tumour of the external auditory meatus.
- Are multiple, progressive, B/L
- Arise from the compact bone. (Whereas osteomas arise from the cancellous bone, osteomas
usually are single smooth swellings)
- Risk factors: swimmers and divers
- Three times more common in males
Treatment: Not needed if asymptomatic. Surgery is the treatment of choice and done when the swelling
is large, causing hearing impairment or retention of wax and debris.

Answer. b

145. Through which of the following cranial nerves is cough reflex on cleaning the ear mediated?

a. 5 th nerve

www.damsdelhi.com Email: info@damsdelhi.com


DAMS CBT 2015 Test -1
b. 7th nerve

c. 9th nerve

d. 10th nerve

Solution. Ans 145: (d) 10th nerve


Ref– Read the text below
Sol:

- Cough reflex on cleaning the ear is mediated through Arnold’s nerve(auricular branch of vagus)

Answer. d

146. High molecular weight protein in nuclear cataract

a. HM1

b. HM2

c. HM3

d. HM4

Solution. Ans-146: (d) HM4


Ref: Read the text below
Sol:
HM4 is exclusively seen in nuclear cataracts.

Answer. d

147. Which stain is used for the diagnosis of macular dystrophy

a. Masson trichome

b. Colloidal iron

c. Congo red

d. Rose Bengal

Solution. Ans-147: (b) Colloidal iron


Ref: Read the text below
Sol:
- In macular dystrophy there is deposition of glycosaminoglycans which are stained by colloidal iron.

Answer. b
www.damsdelhi.com Email: info@damsdelhi.com
DAMS CBT 2015 Test -1

148. Which anti fungal can be given intravitreally

a. Fluconazole

b. Voriconazole

c. Ketoconazole

d. Cotrimazole

Solution. Ans-148: (b) Voriconazole


Ref: Read the text below
Sol:
- Voriconazole and amphotericin-B can be given intravitreally.

Answer. b

149. When compare to blood , vitreous humor has high concentration of

a. Ascorbate

b. Glutathione

c. Glucose

d. Sodium

Solution. Ans-149: (a) Ascorbate

Ref: Read the text below


Sol:
- The vitreous :plasma ratio is 9:1
It is due to active transport by ciliary body epithelium .
- This is so due to the ability of the ascorbate to absorb ultraviolet rays and act as a free radical scavenger .

Answer. a

150. Investigation of choice in retinoblastoma is :

a. CT SCAN

b. MRI

c. X-ray

www.damsdelhi.com Email: info@damsdelhi.com


DAMS CBT 2015 Test -1
d. USG

Solution. Ans-150: (b) MRI


Ref: Read the text below
Sol:
- The choice of investigation is MRI , though commonly the first diagnosis is made by USG -B scan

Answer. b

151. Hollenhorst plaques are seen in :

a. CRAO

b. CRVO

c. CME

d. CSR

Solution. Ans-151: (a) CRAO


Ref: Read the text below
Sol:
- Hollenhorst plaques are cholesterol emboli seen in retinal arterioles. Most common cause of CRAO is
embolism .

Answer. a

152. Field of indirect ophthalmoscope is

a. 2DD

b. 4DD

c. 6DD

d. 8DD

Solution. Ans-152: (d) 8DD


Ref: Read the text below
Sol:
- The field is 8DD and magnification is 5 times . Hence it is used to see the periphery of the retina.

Answer. d

153. Which of the following topical agents cause heterochromia iridis

www.damsdelhi.com Email: info@damsdelhi.com


DAMS CBT 2015 Test -1

a. Latanoprost

b. Prednisolone

c. Timolol

d. Olopatadine

Solution. Ans-153: (a) Latanoprost


Ref: Read the text below
Sol:
- Latanoprost causes iris pigmentation and hence can lead to heterochromia iridis.

Answer. a

154. Appropriate treatment for presumed optic neuritis is

a. Oral prednisone

b. Intravenous methylprednisolone

c. Cyclophosphamide

d. Plasma exchange

Solution. Ans 154: (b) Intravenous methylprednisolone


Reference – Read the text below
Sol:
 Clinical trials have shown that intravenous methylprednisolone for
an attack of optic neuritis reduces the likelihood of development of multiple sclerosis (MS) over
2 years from 16.7% to 7.5%.
- It also is associated with a better outcome than oral prednisone.
- Intravenous methylprednisolone is thus recommended by most
experts as appropriate therapy for acute exacerbations of multiple sclerosis involving more than sensory
manifestations alone.

Answer. b

155. A patient with eye discharge, mild photophobia, a normal papillary reaction to light, and normal
intraocular pressure most likely has which of the following disorders?

a. Acute glaucoma

b. Optic neuritis

c. Acute conjunctivitis

www.damsdelhi.com Email: info@damsdelhi.com


DAMS CBT 2015 Test -1
d. Acute anterior uveitis

Solution. Ans 155: (c) Acute conjunctivitis


Reference – Read the text below
Sol:
- In acute conjunctivitis, this patient has a purulent eye discharge with matting of the eyelashes.
- There is mild photophobia, a normal papillary reaction to light, and normal intraocular pressure.
- It is most commonly secondary to Staphylococcus aureus, Streptococcus pneumoniae, or Haemophilus aegypti
(pink eye).
- Topical polymyxin B, neomycin, and bacitracin are effective therapies.

Answer. c

156. Following a gastric bypass, which form of calcium should be used as a supplement?

a. Calcium sulphate

b. Calcium chloride

c. Calcium gluconate

d. Calcium citrate

Solution. Ans-156: (d) Calcium citrate


Ref: (Schwartz 9th edition)
Sol:
- Calcium, iron and vitamin B 12 deficiency is a common complication following bariatric surgery procedures.
- The usual form of calcium supplementation is in the form calcium carbonate, which is cheap, but in such
patients with low acid exposure, calcium citrate is better absorbed.

Answer. d

157. The most common cause of acute mesenteric ischaemia is

a. Arterial embolus

b. Arterial thrombus

c. Non occlusive mesenteric ischaemia

d. Venous thrombosis

Solution. Ans-157: (a) Arterial embolus


Ref: Read the text below
Sol:

www.damsdelhi.com Email: info@damsdelhi.com


DAMS CBT 2015 Test -1

Answer. a

158. An 18 year old healthy woman is incidentally detected with a 2 cm splenic aneurysm of the mid portion
of the splenic artery. Which of the following is the treatment of choice for this patient?

a. Observation only

b. Embolization

c. Ligation or resection of anuerysm

d. Splenectomy

Solution. Ans-158: (c) Ligation or resection of anuerysm


Ref: Read the text below
Sol:
Splenic Artery aneurysm
- Splenic artery aneurysm is the most common visceral artery aneurysm.
- F:M – 4:1
- Significantly higher rates in patient with portal hypertension
- Indications for treatment include – pregnancy, women of child bearing age,presence of
symptoms, presence of pseudoaneurysms associated with inflammatory processes
www.damsdelhi.com Email: info@damsdelhi.com
DAMS CBT 2015 Test -1
- Aneurysm resection or ligation alone is acceptable for amenable lesions in the mid-splenic
artery, but distal lesions in close proximity to the splenic hilum should be treated with
concomitant splenectomy.

Answer. c

159. Which one of the following orifices cannot be used for NOTES?

a. Bladder

b. Vagina

c. Oral cavity

d. Umblicus

Solution. Ans-159: (d) Umblicus


Ref: Read the text below
Sol:
NOTES stands for natural orifice transluminal endoscopic surgery and it is a modification of laparoscopic
surgery. In this technique the peritoneal cavity is entered through a natural orifice and the surgery is
performed without leaving a scar on the abdominal wall. The various natural orifices which
can be used are:
1. Bladder
2. Vagina/ Cervix/ Uterus
3. Rectum
4. Oral cavity

Answer. d

160. A “sports hernia” is best described as

a. A direct inguinal hernia in an athlete

b. A stress related groin hernia in an athlete

c. A small tear or weakness in the posterior inguinal canal

d. Pubis osteitis

Solution. Ans-160: (c) A small tear or weakness in the posterior inguinal canal
Ref: Read the text below
Sol:
- Affects young men involved in active sports
- Famous cricketer who suffered from this condition - Sachin Tendulkar
- Patient presents with tenderness over the inguinal region
- Usually no hernial swelling is felt
www.damsdelhi.com Email: info@damsdelhi.com
DAMS CBT 2015 Test -1
- Mechanism - muscle tearing or stretching of the posterior inguinal wall (Gilmore's groin)
- MRI is helpful in diagnosis
- - Other musculo-skeletal causes should be ruled out before performing surgery

Answer. c

161. The most common non-continent urinary diversion procedure after radical cystectomy is:

a. Orthoptic neobladder (ileal pouch)

b. Ileal conduit

c. Appendiceal conduit

d. Ureterostomy

Solution. Ans-161: (b) Ileal conduit


Ref: Read the text below
Sol:
- The most common non continent urinary diversion is ileal conduit, in which a segment of distal ileum is
isolated and one end is brought out through the abdominal wall was a urostomy.
- The ureters are implanted into the conduit.
- Orthoptic neobladder is the most common continent urinary diversion where the pouch is
anastomosed with the urethra.

Answer. b

162. The most common mediastinal mass in children is

a. Lymphoma

b. Neurogenic tumor

c. Congenital cyst

d. Germ cell tumor

Solution. Ans-162: (b) Neurogenic tumor


Ref: Read the text below
Sol:

www.damsdelhi.com Email: info@damsdelhi.com


DAMS CBT 2015 Test -1

Answer. b

163. All predisposes to oral cancer except:

a. Erythroplakia

b. Leucoplakia

c. Submucosal fibrosis

d. Lichen planus

Solution. Ans-163: (d) Lichen planus


Ref.: Bailey - 735
Sol :
- Out of the given options all except lichen planus are definitely associated with oral cancer.
- “If there is an association of oral cancer and lichen planus, the relationship exist only with atrophic or
erosive lichen planus”.
- All patients with erosive or atrophic lichen planus should be observed and erosive disease should be treated
with topical steroids or systemic steroids in severe cases.

Answer. d

164. Gum tumor with 2 contra lateral mobile lymph node in cheek comes under all of the following except:

a. T3 N2 M0

b. T2 N2 M0

c. T4 N2 M0

d. T3 N3 M0

Solution. Ans-164: (d) T3 N3 M0


Ref.: CSDT - 264
Sol :
- As the size of tumor is not mentioned so it may be any T
- Since 2 contra lateral mobile lymph node present it will be N2 stage
- So stage can be : T3 N2 M0
T2 N2 M0
T4 N2 M0
www.damsdelhi.com Email: info@damsdelhi.com
DAMS CBT 2015 Test -1

Answer. d

165. Thyroglossal fistula develops to –

a. Developmental anomaly

b. Injury

c. Incomplete removal of thyroglossal cyst

d. Inflammatory disorder

Solution. Ans-165: (c) Incomplete removal of thyroglossal cyst


Ref.: Read the text below
Sol :
Thyroglossal fistula
- It is never congenital
- It follows infection as inadequate removal of thyroglossal cyst.
- Treatment :
As thyroglossal tract is closely related to the body of hyoid bone, the central part must be excised together
with the cyst or fistula (Sistrunk operation).

Answer. c

166. All of the following are associated with thyroid storm, except:

a. Surgery for thyroiditis

b. Surgery for thyrotoxicosis

c. Stressful illness in thyrotoxicosis

d. I131 therapy for thyrotoxicosis

Solution. Ans-166: (a) Surgery for thyroiditis


Ref.: Bailey - 791
Sol :
Thyroid storm
- Acute exacerbation of hyperthyroidism
Precipitated by :
- Acute illness
- Radioiodine treatment in partially treated hyperthyroidism
- Thyroid surgery in inadequately treated hyperthyroid patient
Symptoms
- Fever Delirium
- Seizures Vomiting

www.damsdelhi.com Email: info@damsdelhi.com


DAMS CBT 2015 Test -1
- Coma Diarrhea
- Jaundice
Treatment
- Propylthiouracil (Drug of choice)
- Propanolol (To reduce tachycardia)
- Iodides (Given after propylthiouracil)
- Glucocorticoids

Answer. a

167. Which suture material is used to carry out micro-vascular anastomosis for free flaps?

a. Silk

b. Vicryl

c. Prolene

d. Ethibond

Solution. Ans-167: (c) Prolene


Ref: Read the text below
Sol:

- Prolene – 4-0 till 9-0 prolene is used to carry out vascular repair or anastomosis and in vascular repairs, the
knots are kept outside the lumen.

Answer. c

168. Most common complication associated with ‘Total Parenteral Nutrition’ is :

www.damsdelhi.com Email: info@damsdelhi.com


DAMS CBT 2015 Test -1
a. Catheter related infection

b. Hyperglycemia

c. Pneumothorax

d. Hyperkalemia

Solution. Ans-168: (a) Catheter related infecton


Ref.: Read the text below
Sol :
Most common complication associated with TPN is Catheter related infection
- Total Parenteral Nutrition :
- For providing nutritional support
- All nutritional requirements given only through IV route
- TPN is given through central vein and not through a peripheral vein
- Can also use Peripherally Inserted Central Catheter (PICC)
- Preferred site for central vein infusion is the Superior vena cava
- Preferred access sites are Subclavian > Jugular > Femoral vein
- Total volume of PN should be 2000-300 ml/day

Answer. a

169. What is the indication for surgery in asymptomatic patients with abdominal aortic aneurysm?

a. Transverse diameter > 3 cm

b. Transverse diameter > 5.5 cm

c. Transverse diameter > 7.5 cm

d. No need for surgery unless symptomatic

Solution. Ans-169: (b) Transverse diameter > 5.5 cm


Ref.: Read the text below
Sol :
- Most precise test for imaging aortic aneurysm : CT scan
- 5-year risk of rupture for aneurysm : <5cm – 1 to 2% and > 5 cm-20 to 40%
- For asymptomatic abdominal aortic aneurysms, operation is indicated if the diameter is > 5.5 cm
- In elective surgery of AAA, the operative mortality rate is 1 to 2% whereas after acute rupture the mortality
of emergency operation is >50%
- MC complications after repair are Cardiac (ischemia and infarction) & Respiratory (atelectasis & lower lobe
consolidation)
- MC non-cardiac complication after repair of AAA is Renal failure.

Answer. b

170. What is the most common cause of osteoblastic bone metastasis in females?

www.damsdelhi.com Email: info@damsdelhi.com


DAMS CBT 2015 Test -1

a. Ca breast

b. Ca cervix

c. Ca thyroid

d. Osteosarcoma

Solution. Ans-170: (a) Ca breast


Ref.: Read the text below
Sol :
- Commonest cause of osteoblastic metastasis in women : Ca breast
- Bone metastasis in Ca breast is of mixed type, i.e. both osteolytic & osteoblastic
- But is predominantly osteolytic.
Bone Metastasis :
- Most common site for secondaries is Bone
- Skeletal metastasis may be osteolytic or osteoblastic
- Majority of metastasis are mixed type; predominantly osteolytic.

Answer. a

171. Hyperthyroidism associated with exophthalmos is most likely caused by

a. Toxic multinodular goiter

b. Subacute thyroiditis

c. Hashimoto’s thyroiditis

d. Graves’ disease

Solution. Ans-171: (d) Graves’ disease


Ref.: Read the text below
Sol :
- Each of these choices may cause thyrotoxicosis.
- Technically, exogenous thyroid hormone can cause thyrotoxicosis but not hyperthyroidism, since the excess
thyroid hormone does not come from the thyroid gland.
- Only graves’ disease is associated with exophthalmos.

Answer. d

172. Which of the following is not done in case of pheochromocytoma?

a. FNAC

www.damsdelhi.com Email: info@damsdelhi.com


DAMS CBT 2015 Test -1
b. MRI

c. CI

d. MIBG

Solution. Ans-172: (a) FNAC


Ref.: Harrison - 2151
Sol :
- Percutaneous fine-needle aspiration of chromaffin tumors is contraindicated; indeed, pheochromocytoma
should be excluded before adrenal lesions are aspirated.

Answer. a

173. Dissection of which artery is seen in pregnancy :

a. Carotid artery

b. Aorta

c. Coronary A

d. Femoral artery

Solution. Ans-173: (b) Aorta


Ref.: Harrison - 1565
Sol :
- The most common artery dissection seen in pregnancy is aortic dissection, mainly in 3rd trimester of
pregnancy.
www.damsdelhi.com Email: info@damsdelhi.com
DAMS CBT 2015 Test -1
- As such aortic dissection is more common in males than females.
- MC site of dissection is right lateral wall of ascending aorta, followed by descending aorta just below
ligmentum venosum.

Answer. b

174. OPSI is related to?

a. Kidney

b. Brain

c. Lung

d. Spleen

Solution. Ans-174: (d) Spleen


Ref.: Read the text below

Sol :
- This disease, overwhelming postsplenctomy infection (OPSI), also referred to as post splenectomy sepsis
(PSS), is one of a group of infectious disease processes, such as bacterial meningitis and meningococcemia,
for which diagnosis and therapeutic intervention are required immediately to minimize the disease impact.

Answer. d

175. Regarding gynaecomastia false statement is:

a. May regress spontaneously

b. May regress after discontinuing the offending drug

c. Subcutaneous mastectomy is indicated in all cases

d. Associated with liver failure

Solution. Ans-175: (c) Subcutaneous mastectomy is indicated in all cases


Ref.: Devita - 1437
Sol :

- Subcutaneous mastectomy is indicated only when gyanecomastia causes severe discomfort or


embarrassment.
Gynecomastia
- The male breast is normally vestigial but under certain hormonal and drug influences it may develop into a
significant breast mass, this is called as gynecomastia.
- Gynecomastia is the term used to define breast enlargement due to growth of ductal tissue and stroma.
- Other cause of breast enlargement such as fat accumulation or tumors is known as pseudogynecomastia.

www.damsdelhi.com Email: info@damsdelhi.com


DAMS CBT 2015 Test -1
Answer. c

176. Nissens fundoplication is a first line treatment for?

a. GERD

b. Hiatus hernia

c. Esophageal atresia

d. Congenital hypertrophic pyloric stenosis

Solution. Ans-176: (b) Hiatus hernia


Ref.: Read the text below
Sol :
- Nissen fundoplication is a surgical procedure to treat gastroesophageal reflux disease (GERD) and hiatus
hernia.
- In GERD it is usually performed when medical therapy has failed, but with paraesophageal hiatus hernia, it
is the first-line procedure.
- In a fundoplication, the gastric fundus (upper part) of the stomach is wrapped, or plicated, around the lower
end of the esophagus and stitched in place, reinforcing the closing function of the lower esophageal sphincter.
- The esophageal hiatus is also narrowed down by sutures to prevent or treat concurrent hiatal hernia, in
which the fundus slides up through the enlarged esophageal hiatus of the
diaphragm.

Answer. b

177. Concentration of Na (meq/1) in normal saline is?

a. 77

b. 109

c. 130

d. 154

Solution. Ans-177: (d) 154


Ref.: Read the text below
Sol :

www.damsdelhi.com Email: info@damsdelhi.com


DAMS CBT 2015 Test -1

Answer. d

178. Most common site of metastasis for breast carcinoma is?

a. Thoracic vertebra

b. Pelvis

c. Femur

d. Lumbar vertebra

Solution. Ans-178: (d) Lumbar vertebra


Ref.: Read the text below
Sol :
- Vertebra is the most common site of metastatic breast carcinoma.
- “In order of frequency the lumbar vertebrae, femur, thoracic vertebrae, rib and skull are affected and these
deposits are generally osteolytic”
- Metastases may also commonly occur in the liver, lungs and brain and, occasionally, the adrenal glands and
ovaries.

Answer. d

179. Sign seen in “Large duct papilloma” is?

a. Nipple discharge

b. Breast mass

c. Skin excoriation

d. Lymph node involvement

Solution. Ans-179: (a) Nipple discharge


Ref.: Read the text below
Sol :
www.damsdelhi.com Email: info@damsdelhi.com
DAMS CBT 2015 Test -1
Large duct papilloma (intraductal papilloma) :
- Papilloma is a benign lesion that arises from the proliferation of ductal epithelium that projects into the
lumen of the duct.
- Benign papilloma is the single most common cause of serous or bloody discharge from the nipple
- More than 80% of large duct papilloma produces nipple discharge
- Often the lesion is only few millimeters in size and mammography is normal.
- Occasionally mulberry like calcification is seen.

Answer. a

180. Hernia with highest rate of strangulation is?

a. Direct inguinal hernia

b. Indirect inguinal hernia

c. Femoral hernia

d. Incisional hernia

Solution. Ans-180: (c) Femoral hernia


Ref.: Read the text below
Sol :

- A strangulated femoral hernia occurs when a femoral hernia blocks blood supply to part of the bowel - the
loop of bowel loses its blood supply.
- Strangulation can happen in all hernias, but is more common in femoral hernias due to their narrow "necks".
Nausea, vomiting, and severe abdominal pain may occur with a strangulated hernia. This is a medical
emergency.
- A strangulated intestine can result in necrosis (tissue death) followed by gangrene (tissue decay). This is a
life-threatening condition requiring immediate surgery

Answer. c

181. An 18 yr old boy presents with melaena, abdominal pain, numbness of both lower limbs and right upper
limb. O/E: pulse: 88/min,BP : 170/110 mm Hg, and all peripheral pulses palpable. Renal function is normal.
Urine examination shows proteinuria and RBC with no casts. What is the most probable diagnosis?

a. Classical PAN

b. Microscopic polyangitis

c. HSP

d. Wegener’s granulomatosis

Solution. Ans 181: (a) Classical PAN

www.damsdelhi.com Email: info@damsdelhi.com


DAMS CBT 2015 Test -1
Ref– Read the text below
Sol:
- Digital gangrene is usually associated with vasculitis of the medium sized vessels. Among the
options PAN is medium vessel vasculitis.
- Mononeuritis multiplex : which is presenting as numbness of different neuroanatomical
regions: is a characteristic feature of PAN and Microscopic polyangitis (MPA,) less characteristic
of Wegner’s (WG)
- No RBC casts are seen in urine and Hypertension is present: so there is no glomerulonephritis
with arteritis, PAN is arteritis with no glomerulonephritis.
- MPA and WG: renal involvement is Glomerulonephritis. Hence diagnosis is PAN.

Answer. a

182. True statement regarding nausea and vomiting during pregnancy include all of the following except:

a. Nausea and vomiting during pregnancy are more common in nonsmokers

b. They are more common in primigravida

c. They are more common in those who have taken oral contraceptives

d. They are more common in woman with less than a tenth-grade education

Solution. Ans 182: (c) They are more common in those who have taken oral contraceptives
Ref– Read the text below
Sol:
- Prior use of oral contraceptives does not alter the incidence of nausea and vomiting during pregnancy.
- However, nausea during the first trimester of pregnancy is more common in non-smokers, primigravidas
obese women and woman with lower levels of education.

Answer. c

183. The following persons are at increased risk for gastrointestinal tuberculosis except:

a. Persons from endemic areas

b. Persons infected with human immunodeficiency

c. Person with a history of alcohol abuse

d. Persons who have splenectomy

Solution. Ans 183: (d) Persons who have splenectomy


Ref– Read the text below
Sol:
- Gastrointestinal tuberculosis is caused by Myco-bacterium tuberculosis and Mycobacterium
bovis.

www.damsdelhi.com Email: info@damsdelhi.com


DAMS CBT 2015 Test -1
- Persons at increased risk for tuberculosis infection include those infected with human
immunodeficiency virus, older people, persons from endemic areas, persons who abuse
intravenous drugs.
- The most common presenting symptoms is abdominal pain.
- The differential diagnosis includes Crohn’s disease, lymphoma and infections including
mycobacterium avium complex enteritis,histoplasmosis and cryptospo-ridiasis.

Answer. d

184. What is the most frequent site of gastrointestinal tuberculosis infection?

a. Stomach

b. Duodenum

c. Jejunum

d. Terminal ileum and Cecum

Solution. Ans 184: (d) Terminal ileum and Cecum


Ref– Read the text below
Sol:
- Mycotic infection of gastrointestinal tract can result from the dissemination of pulmonary infections.
- Histoplasmosis can disseminate in immunocompetent as well as immune compromised persons.
- Intestinal aspergillosis, candidiasis and mucormycosis affect only severly immunocompromised persons.

Answer. d

185. Which of the following mycotic infection of the small intestine affects immunocompetent as well as
immunocompromised persons?

a. Histoplasmosis

b. Aspergillosis

c. Candidiasis

d. Mucormycosis

Solution. Ans 185: (a) Histoplasmosis


Ref– Read the text below

Sol:
- Mycotic infection of gastrointestinal tract can result from dissemination of pulmonary
infections.
- Histoplasmosis can disseminate in immunocompetent as well as immune compromised
persons.
- Intestinal aspergillosis, candidiasis and mucormycosis affect only severely

www.damsdelhi.com Email: info@damsdelhi.com


DAMS CBT 2015 Test -1
immunocompromised persons.

Answer. a

186. Recto-anal inhibitory reflex includes all except:

a. Relaxation of EAS

b. Contraction of EAS

c. Relaxation of IAS

d. Contraction of rectal Muscles

Solution. Ans 186: (a) Relaxation of EAS


Ref– Read the text below
Sol:
- Recto-anal inhibitory reflex occurs in response to rectal distension.
- Relaxation of EAS not occur as a part of recto-anal inhibitory reflex.

Answer. a

187. Any lung cavity with a wall thickness more than or equal to 4 mm should raise suspicion of Ca Lung.
Squamous-cell carcinoma is the most common histological type of lung cancer to cavitate (82%). Other causes
of thick walled cavity include all except:

a. Lung abscess

b. Wegeners granulomatosis

c. Emphysema

d. Blastomycosis

Solution. Ans 187: (c) Emphysema


Ref– Read the text below
Sol:
Thick walled cavities are seen in:
- Lung abscess
- Necrotizing squamous cell lung cancer
- Wegners granulomatosis
- Blastomycosis
Thin walled cavities are seen in:
- Coccidiomycosis
- Metastatic cavitating squamous cell carcinoma from Cervix
- M. Kansasii infection
- Congenital or acquired bullae (Emphysema)
www.damsdelhi.com Email: info@damsdelhi.com
DAMS CBT 2015 Test -1
- Post- traumatic cysts
- Open negative TB

Answer. c

188. Each of the following statement regarding diverticular bleed is true except:

a. Occurs in 3%-5% of diverticular cases

b. Right sided colon is the most common source of bleed in Asian patient

c. Diverticulitis increases the risk of diverticular bleed

d. Surgery is not required in all cases

Solution. Ans 188: (c) Diverticulitis increases the risk of diverticular bleed
Ref– Read the text below
Sol:
- Diverticular bleed occurs in only 3-5% of diverticulosis patient.
- Diverticulitis never increases risk of diverticular bleed.
- Surgery is not indicated in all cases.
- Right sided colon is the most common source of diverticular bleed in Asian patient.

Answer. c

189. Rectosigmoid junction from anal verge:

a. 12 cm

b. 18 cm

c. 24 cm

d. 30 cm

Solution. Ans 189: (a) 12 cm


Ref– Read the text below
Sol:
- Rectosigmoid junction lies 12-16 cm above anal verge.

Answer. a

190. Which of the following haematological conditions predispose to renal papillary necrosis?

a. Hereditary Spherocytosis
www.damsdelhi.com Email: info@damsdelhi.com
DAMS CBT 2015 Test -1

b. Acute Myeloid Leukemia

c. Sickle cell anaemia

d. Hemophilia

Solution. Ans 190: (c) Sickle cell anaemia


Ref– Read the text below
Sol:
The major causes of Renal Papillary necrosis are:
a. Analgesic nephropathy
- b. Sickle cell nephropathy
- Diabetes with urinary tract infection

Answer. c

191. Prevalence of portal vein thrombosis in cirrhosis is:

a. 6%

b. 10%

c. 16%

d. 25%

Solution. Ans 191: (b) 10%


Ref– Read the text below
Sol:
- The disorder occurs in at least 10% of patients with cirrhosis, presumably as a result of sluggish portal vein
blood flow, but acquired and inherited hypercoagulable states can be indentified in many patients with
cirrhosis and portal vein thrombosis.

Answer. b

192. The minimum level of AFP which is strongly suggestive of HCC is:

a. 500 ng/ml

b. 500 ng/l

c. 1000 ng/ml

d. 1000 mg/ml

Solution. Ans 192: (a) 500 ng/ml


Ref– Read the text below
Sol:
- A value of 500 ng/Ml (500 mg/L) is used as a diagnostic level because levels below this value may be found
in patients with a variety of acute and chronic benign liver diseases, such as acute and chronic hepatitis and
www.damsdelhi.com Email: info@damsdelhi.com
DAMS CBT 2015 Test -1
cirrhosis.
- False- Positive results also may occurs in patients with tumors of endodermal origin and nonseminomatous
germ cell tumors.

Answer. a

193. The level of which one of the following hormones is likely to increase after hypothalamic ablation?

a. Growth hormone

b. Prolactin

c. FSH

d. ACTH

Solution. Ans-193: (b) Prolactin


Ref.: Read the text below
Sol:

www.damsdelhi.com Email: info@damsdelhi.com


DAMS CBT 2015 Test -1

Answer. b

194. Which one of the following does not cause pulsus paradoxus ?

a. Severe aortic regurgitation

b. Cardiac tamponade

c. Constrictive pericarditis

d. Acute severe bronchial asthma

Solution. Ans-194: (a) Severe aortic regurgitation


Sol :
Causes of pulsus parodoxus :
- Cardiac – common with cardiac temponade, infrequent with constrictive pericarditis
www.damsdelhi.com Email: info@damsdelhi.com
DAMS CBT 2015 Test -1

- Pulmonary – bronchospasm as in severe asthma or emphysema, pulmonary embolism


- Miscellaneous – obesity possibility due to excessive compression of IVC at thoracic inlet, after
thoracotomy
- Hypovolemic shock
- SVC obstruction
- Reversed pulsus paradoxus – inspiratory rise in arterial systolic and diastolic pressure.

- Seen in HOCM, during intermittent positive pressure breathing (IPPB), isometric ventricular
rhythm.

Answer. a

195. Which of the following test of peritoneal fluid has the greatest sensitivity for detecting spontaneous
bacterial peritonitis?

a. White blood cell count

b. Gram’s stain

c. pH

d. Lactate

Solution. Ans-195: (a) White blood cell count


Ref.: Read the text below
Sol :
- There is not true gold standard in the diagnosis of spontaneous bacterial peritonitis.
- If the white blood cell count is more than 250 calls/mm3, there is a greater than 95% chance that bacterial
peritonitis is present.
- The sensitivities of ascetic fluid lactate and pH are less than 50%, whereas that of the Gram stain is less
than 10%.

Answer. a

196. Which of the following represents a contraindication to abdominal paracentesis?

a. Obesity

b. Extensive prior abdominal surgery

c. Abdominal aortic aneurysm

d. Disseminated intravascular coagulation.

Solution. Ans-196: (d) Disseminated intravascular coagulation.


Ref.: Read the text below
www.damsdelhi.com Email: info@damsdelhi.com
DAMS CBT 2015 Test -1
Sol :
- There are few contraindications to abdominal paracentesis. The most important is the presence of severe
uncountrollable coagulopathy.
- If there are concerns about localizing site for aspiration of peritoneal fluid in obese patients or those with a
history of extensive abdominal surgery, ascetic fluid collections can be localized by ultrasonography.

Answer. d

197. A 45-year-old man presents with sudden onset breathlessness and chest pain. Examination revealed
tachypnoea, cyanosis, hypotension, raised JVP, widely split second heart sound heart sound, and right
ventricular gallop. What is the most likely diagnosis?

a. Acute myocardial infarction with cardiogenic shock

b. Acute pericarditis

c. Massive pulmonary embolism

d. Pneumothorax

Solution. Ans-197: (c) Massive pulmonary embolism


Ref.: Harrison’s Principles of internal medicine
Sol :
- Pneumothorax – may present with breathlessness, cyanosis, hypotension, but widely split second heart
sound and right ventricular gallop not found in this disease.
- Acute pericarditis – usually present with chest pain with pericardial click. Breathlessness is not a common
symptom. Widely split second heart sound not found in this disease.
- Breathlessness, chest pain, hypotension, raised JVP, right ventricular gallop can be present in booth AMI
and massive pulmonary embolism. Widely split second heart sound, tachypnoea is in favour of massive
pulmonary embolism.
- Most common cause of acute right ventricular failure is pulmonary embolism.

Answer. c

198. In which of the following is differential cyanosis found?

a. VSD with reversal of shunt

b. PDA with reversal of shunt

c. ASD with reversal of shunt

d. Tetralogy of Fallot

Solution. Ans-198: (b) PDA with reversal of shunt


Ref.: Gupte S. -240
Sol :
www.damsdelhi.com Email: info@damsdelhi.com
DAMS CBT 2015 Test -1
Some important feature of patent ductus arteriosus :
- It is most often occur as isolated defect unlike other congenital cardiac disease.
- Symptomatic patient may have growth retardation, exertion dyspnoea, left ventricular failure
and congestive cardiac failure.
- Precordial pain, hoarseness may present.
- Pulse pressure is wide.
- Water hammer pulse and Corrigan pulse may present.
- Differential cyanosis may present in both legs and left arm (since the aortic attachment of ductus
arteriosus is just distal to left subclavian artery) there may be paradoxical splitting of P2.
- Continuous murmur (machinery murmur) best heard at 2nd intercostals space.

Answer. b

199. A patient complains of intermittent claudication, dizziness and headache. Which one of the following is
the likely diagnosis?

a. Coarctation of aorta

b. Patent ductus arteriosus

c. Tetralogy of fallot

d. Atrial septal defect

Solution. Ans-199: (a) Coarctation of aorta


Ref.: CSDT -426
Sol :

- Most patients are asymptomatic.


- Headache, epistaxis, cold extremities and claudication with exercise may occur

Answer. a

200. What are exacerbations of bronchial asthma, that occur with little or no warning, called?

a. Brittle asthma

b. Acute severe asthma

c. Poorly controlled asthma

d. Nocturnal asthma

Solution. Ans-200: (a) Brittle asthma


Ref.: Harrisons’ - 1606.
Sol :
There is chaotic variations in lung function despite talking appropriate therapy.

www.damsdelhi.com Email: info@damsdelhi.com


DAMS CBT 2015 Test -1
- Type – 1 brittle asthma – there is persistent pattern of variability. Oral steroid and continuous infusion of β2
agonist is required.
- Type -2 brittle asthma – generally normal or near normal lung function. But precipitous, unpredictable fall in
lung function that may result in death.
- It is difficult to manage because these do not respond to steroid and worsening of asthma do not reverse with
bronchodilator.
- Most effective therapy is subcutaneous epinephrine

Answer. a

201. A 60-year-old chronic smoker presents with retrosternal chest pain and diaphoresis. The ECG shows
ST elevation in V2-V4 leads for which primary PTCA and stent placement have been done. Which of the
following drugs is not to be advised in the first 48 hours of intervention?

a. Abciximab

b. Aspirin

c. Enalapril

d. Dexamethasone

Solution. Ans-201: (d) Dexamethasone


Ref.: CMDT 2007, 367
Sol :
- In any type of acute coronary syndrome, STEMI, INSTEMI, angina-aspirin blocker and ACE inhibitor is

www.damsdelhi.com Email: info@damsdelhi.com


DAMS CBT 2015 Test -1
given in any cause.
- Abciximab is given in conjunction with PCI stenting – this procedure is now widely used.
- According to CMDT, dexametha-sone is option of exclusion.

Answer. d

202. Palpable purpura may be seen in the following, except :-

a. Scurvy

b. Acute meningococcemia

c. Vasculitis

d. Essential mixed cryoglobulinemia

Solution. Ans-202: (a) Scurvy


Ref: Read the text below
Sol :

CAUSES OF PURPURA
I. Primary cutaneous disorders
A. Nonpalpable
1. Trauma
2. Solar pupura
3. Steroid purpura
4. Capillaritis
5. Livedoid vasculitis
II. Systemic diseases
A. Nonpalpable
1. Clotting disturbances
a. Thrombocytopenia (including ITP)
b. Abnormal platelet function
c. Clotting factor defects
2. Vascular fragility
a. Amyloidosis b. Ehlers-Danlos syndrome c. Scurvy
3. Thrombi
a. Disseminated intravascular coagulation
b. Monoclonal cryoglobulinemia
c. Thrombotic thrombocytopenic purpura
d. Warfarin reaction
4. Emboli
a. Cholesterol b. Fat
5. Possible immune complex
a. Gardner-Diamond syndrome (autoerythrocyte sensitivity)
b. Waldenstrom's hypergammaglobulinemic purpura
B. Palpable
1. Vasculitis
a. Leukocytoclastic vasculitis
b. Polyarteritis nodosa

www.damsdelhi.com Email: info@damsdelhi.com


DAMS CBT 2015 Test -1
2. Emboli
a. Acute meningococcemia
b. Disseminated gonococcal infection
c. Rocky mountain spotted fever
d. Ecthyma gangrenosum

Answer. a

203. Typical features of tabes dorsalis include all, except :

a. Paroxysmal abdominal and girdle pains

b. Urinary incontinence with absent ankle and plantar reflexes

c. High stepping, stamping gait with muscle hypotonia

d. None of the above

Solution. Ans-203: (d) None of the above


Ref: Read the text below

Sol :
- Tabes dorsalis is a late manifestation of syphilis that presents as symptoms and signs of demyelination of
posterior columns, dorsal roots & dorsal root ganglia.
- The most common symptoms of tabes are characteristic fleeting and repetitive lancinating pains, which occur
primarily in the legs and less commonly in the back, thorax, abdomen, arms, and face. Ataxia of the legs due to
loss of position sense (leading to stamping and high stepping gate) occurs in half of patients. Paresthesias,
bladder disturbances, and acute abdominal pain with vomiting (visceral crisis) occur in 15 to 30% of patients.
- The cardinal signs of tabes are loss of reflexes in the legs, impaired position and vibratory sense,
Romberg's sign, and bilateral Argyll Robertson pupils, which fail to constrict to light but react with
accommodation.

Answer. d

204. A young male presented with hematuria after two days of diarrhea. The blood chemistry shows more
than 80% dysmorphic RBCs. The most likely diagnosis is:

a. Disseminated intravascular coagulation

b. Hemolytic uremic syndrome

c. Post streptococcal glomerulonephritis

d. Acute pyelonephritis

Solution. Ans-204: (b) Hemolytic uremic syndrome


Ref: Read the text below

www.damsdelhi.com Email: info@damsdelhi.com


DAMS CBT 2015 Test -1
Sol :
- Hemolytic-uremic syndrome is a disease of infancy and early childhood.
- Patients present with fever, thrombocytopenia, microangiopathic hemolytic anemia (leading to dysmorphic
RBCs), hypertension, and varying degrees of acute renal failure.
- Epidemics related to infection with a specific strain of Escherichia coli (O157:H7) have been documemted.
As in TTP, DIC is not found & neurological symptoms other than uraemia are uncommon.

Answer. b

205. Each of the following conditions may explain the presence of abnormal liver chemistries in patients
receiving a total parenteral nutrition (TPN) except :

a. Hepatic steatosis

b. Acalculous cholecystitis

c. Calculous cholecystitis

d. TPN – associated hemolysis

Solution. Ans-205: (d) TPN – associated hemolysis


Ref.: Read the text below
Sol :
- There is no increased risk of hemolysis in patients receiving total parenteral nutrition.
- Elevated liver chemistries in patients receiving total parenteral nutrition are likely to be multifactorial,
including hepatic steatosis and calculous as well as acalculous cholecystitis.

Answer. d

206. The retractor shown below is used to retract

www.damsdelhi.com Email: info@damsdelhi.com


DAMS CBT 2015 Test -1

a. Bladder

b. Uterus

c. Abdominal wall

d. Vagina

Solution. Ans 206: (a) Bladder


Ref:Read the text below
Sol:
Doyen Retractor is a handheld retractor available in two distinct styles. They are each primarily used in

abdominal OB/GYN procedures such as abdominal hysterectomies, cesarean section deliveries, and
procedures for ectopic pregnancies.

Answer. a

207. Shortest Diameter of fetal head

a. Bitemporal
www.damsdelhi.com Email: info@damsdelhi.com
DAMS CBT 2015 Test -1

b. Biparietal dia

c. Suboccipetofrontal

d. Occipetofrontal

Solution. Ans 207: (a) Bitemporal


Ref:Read the text below

Sol:
- Shortest fetal head diameter is bimastoid 7.5cm and then bitemporal which is 8cm
- Longest fetal head Bitrochanteric is engaging dia in breech and measures 10cm diameter is
mentovertical which is 14 cm and then comes occipetomental which is 13.5cm
- Occipetofrontal is the engaging diameter in occipeto posterior and measures 11.5cm(OP can
also have suboccipetofrontal as engaging is the engaging diameter in vertex presentation and
measures 9.5
- Submentobregmatic is engaging diameter in face presentation and measures 9.5
Mentovertical is engaging dia in brow presentation

Answer. a

208. Most effective method of Birth control

a. POP

b. Condom

c. Vasectomy

d. Diaphragm

Solution. Ans 208: (C) Vasectomy


Ref:Read the text below
Sol:
- Vasectomy has failure rate of 0.1%
- Female sterilization has a filure rate of 0.5%
And is effective after 3 months or when azoospermia is documented in the client(better
answer)
- The eligibility criteria for male sterilization is that male client should be below 60 years of age
- Implants have the least failure rate among all contraceptive agents which is 0.05% while for
implanon it is 0.01%

Answer. c

209. Treatmentl of stage 2A.Ca.Cx. is

www.damsdelhi.com Email: info@damsdelhi.com


DAMS CBT 2015 Test -1
a. Wertheims hysterectomy

b. Meigs Gysterectomy

c. Chemotharapy

d. EBRT + Brachytherapy

Solution. Ans 209: (d) EBRT + Brachytherapy


Ref:Read the text below
Sol:
- Management of stages beyond and equal to 1b2 is chemoradiation
- Management of stage 1b1 is radical hysterectomy while that of stage 1a2 is modified radical
and 1a1 is simple hysterectomy if family iscomplete
- In chemoradiation both should be given concurrently. External beam radiotherapy is followed
by brachytherapy.
- The chemotherapy used is weekly cisplatin 40mg/m2during radiothetherapy
- Most women receive EBRT to pelvis while brachytherapy is local application of radiation to vagina and
allows higher dose to cervix while sparing normal tissue

- Brachytherapy can be given as low dose rate therapy which is 0.4-2gy per hour or high dose rate therapy
which is >12 gy per hour

Answer. d

210. SRY gene is located on

a. Long arm of Y – chromosome

b. Short arm of Y – chromosome

c. Long arm of X – chromosome

d. Short arm of X – chromosome

Solution. Ans-210: (b) Short arm of Y – chromosome


Ref:Read the text below
Sol:
- SRY (which stands for sex-determining region Y gene) is found on the Y chromosome. In the
cell, it binds to other DNA and in doing so distorts it dramatically out of shape.
- This alters the properties of the DNA and likely alters the expression of a number of genes,
leading to testis formation.
- Most XX men who lack a Y chromosome do still have a copy of the SRY region on one of their X
chromosomes. This copy accounts for their maleness.
- However, because the remainder of the Y chromosome is missing they frequently do not
develop secondary sexual characteristics in the usual way.

Answer. b

211. Which one of the following biochemical parameters is the most sensitive to detect open spina bifida?

www.damsdelhi.com Email: info@damsdelhi.com


DAMS CBT 2015 Test -1

a. Matermal serum alpha fetoprotein

b. Amniotic fluid alpha fetoprotein

c. Amniotic fluid acetyl cholinesterase

d. Amniotic fluid glucohexaminase

Solution. Ans-211:(c) Amniotic fluid acetyl cholinesterase


Ref: Williams - 322
Sol:
- Spina Bifida, anencephaly and encephalocele are collectively included under neural tube defect.
- Amniotic fluid cholinesterase level is elevated in most cases of neural tube defects and has got
better diagnostic value than AFP.
Ultrasonographic signs indicating Spina Bifida.
- Small biparietal diameter.
- Ventriculomegaly.
- Frontal bone scalloping (lemon sign).
- Elongation and downward displacement of the cerebellum (banana sign)
- Effacement or obliteration of the cistern magna.

Answer. c

212. Genital tuberculosis primarily affects :

a. Uterus

b. Ovary

c. Fallopian tubes

d. Cervix

Solution. Ans-212: (c) Fallopian tubes


Ref.: Read the text below
Sol :
- The incidence of genital tuberculosis is about 1 per cent amongst the gynecological patients
attending the outpatient department in the developing countries.
- The causative organism is Mycobacterium tuberculosis of human type. Very rarely the bovine
type may affect the vulva.
- Genital tuberculosis is almost always secondary to primary infection elsewhere in the extra
genital sites as lungs as lungs lymph nodes, urinary tract bones and joints.
- The fallopian tubes are invariably the primary site of pelvic tuberculosis from where secondary
spread occur other genital organs.

Answer. c
www.damsdelhi.com Email: info@damsdelhi.com
DAMS CBT 2015 Test -1

213. Macafee treatment in placenta praevia is contraindicated in all except :-

a. The pregnancy has reached 38 weeks

b. If the fetus is dead

c. If the fetus is IUGR

d. If profuse bleeding starts

Solution. Ans-213: (c) If the fetus is IUGR


Ref:Read the text below
Sol:
- If the fetus is IUGR, Macafee treatment is applicable. In others, termination of pregnancy is
desirable.

Answer. c

214. A 24 years old female has severe cervical erosion,pap smear shows inflammation. Medical management
of choice is :-

a. Hysterectomy

b. Combination of azithromycin, secnidazole with cryo surgery

c. Cervical caps

d. Vaginal pessaries.

Solution. Ans-214: (b) Combination of azithromycin, secnidazole with cryo surgery


Ref:Read the text below
Sol:
 It is PID, usually M/M by triad of fluconazole (150 gm) azithromycin (1 gm) secnidazole (1 gm)
cryosurgery to freeze the erosion should be done in postmenstrual phase (on 6th day of menses)
- The refrigents used in cryo are :--
Feron (-60 degrees)
- Carbondioxide (-60°C)
- Nitrous oxide (-80°C)

Answer. b

215. The most specific assessment of foetal well being is by :

a. Biophysical profile (BPP)

www.damsdelhi.com Email: info@damsdelhi.com


DAMS CBT 2015 Test -1
b. Oxytocin challenge test

c. Non – stress test

d. Urinary estradiol estimation

Solution. Ans-215: (a) Biophysical profile (BPP)


Ref.: Read the text below
Sol :
The BPP looks at five categories and gives a score of either 0 or 2 for each.
- The categories include :
- Amniotic fluid volume.
- Fetal tone, fetal activity.
- Fetal breathing movements.
- NST

- A BPP o 8/10 or better is reassuring.


- Ultrasound is used to perform a BPP.
- Ultrasound can also be used to assess the blood flow velocity in the umbilical cord and the
presence or absence of diastolic flow.

Answer. a

216. Longest diameter of fetal skull is:

a. Biparietal

b. Bitemporal

c. Occipito temporal

d. Submentovertical

Solution. Ans 216: (d) Submentovertical


Reference: Dutta Obs. 6/e, p 85
Sol :
Diameter Value
Subocipito bregmatic 9.5 cms
Sub mento bregmatic
Suboccipito frontal 10 cms
Occipito frontal 11.5 cms
Sub mento vertical
Mento vertical 14 cms

Answer. d

217. Not a function of amniotic fluid :

www.damsdelhi.com Email: info@damsdelhi.com


DAMS CBT 2015 Test -1
a. Decreased fetal weight

b. Reduce trauma to fetus

c. Maintain even temperature

d. Provide nutrition in the form of proteins

Solution. Ans-217: (d) Provide nutrition in the form of proteins.


Ref.: Read the text below
Sol :
The main function of placenta is protective to the fetus.
- During pregnancy :

- It acts as a shock absorber protecting the fetus from possible extraneous injury.
- Maintains an even temperature.
- The fluid distends the amniotic sac and thereby all for growth and free movement of the fetus
and prevents adhesion between the fetal parts and amniotic sac.
- Its nutritive value is negligible because of small amount of protein and salt content, however,
water supply to the fetus is quite adequate.
- During labor :
- The amnion and chorion are combined to form a hydrostatic wedge which helps in dilatation of

the cervix.
- During uterine contraction, it prevents marked interference with the placental circulation so
long as the membranes remain intact.
- It flushes the birth canal at the end of first stage of labor and by its aseptic and bactericidal action protects the
fetus and prevents ascending infection to the uterine cavity.

Answer. d

218. CA 125 is increased in all Except.

a. TB of genital tract

b. endometrioma

c. PCOS

d. Menstruation

Solution. Ans 218: (c) PCOS


Ref:Read the text below
Sol:
The normal value of CA 125 is <=35U/ml. CA 125 antigen is a transmembrane glycoprotein and is
raised in the following benign conditions
Menstruation
Pregnancy
PID
Fibroid
Adenomyosis
www.damsdelhi.com Email: info@damsdelhi.com
DAMS CBT 2015 Test -1
Endometriosis
Ovarian hyperstimulation
Meigs syndrome
Functional ovarian cyst
Inflammatory conditions of abdomen
Genital tuberculosis
Malignancies in which it is raised
Ovarian
Endometrial
Fallopian tube
Lung
Breast
Liver
Gall bladder

Colon
Pancreas
Hematologic malignancies

Answer. c

219. Morning after pill is:

a. Mifepristone

b. Levonorgestrel
c. Norethisterone

d. Medroxyprogesterone

Solution. Ans 219: (b) Levonorgestrel


Ref:Read the text below
Sol:
- Morning after pill is the levonorgestrel emergency contraceptive pill
- Candidates for emergency contraceptives are women who have had recent unprotected
intercourse or those who have had a failure of another method of contraception
The pack has 2 tablets each of strength 0.75mg. the pill can be tajen as single dose of 1.5 mg or
as divided doses 12 hrs apart.
- The pill is most eefctive if used within 72 hrs of unprotected intercourse but is effective even
when taken upto 120hr
- Main mechanism of action is delaying ovulation followed by inhibition of implantation
- Levonorgestrel is less effective than ulipristal and mifepristone and is more effective than
yuzpe regimen

Answer. b

220. Contraception of choice in woman with HIV

a. Condom
www.damsdelhi.com Email: info@damsdelhi.com
DAMS CBT 2015 Test -1

b. OCP

c. Condom +OCP

d. Mirena

Solution. Ans 220: (c) Condom +OCP


Ref:Read the text below
Sol:
- Contraception of choice for HIV positive is permanent methods preferred over non
permamnent
- Among non permanent methods the choice is IUD PLUS CONDOMS
- Then OCP PLUS CONDOMS then IUD alone then OCP alone
- Contraception of choice for lactating women is POP then IUD
- Contraception for newly married couple is OCP
- Contraception of choice for couple living far away meeting occasionally is BARRIER
- Contraception with least risk of ectopic is OCP
Contraception with least risk of PID IS BARRIER

Answer. c

221. Amniocentesis is done between

a. 12-14 weeks

b. 14-16weeks

c. 16-18 weeks

d. 18-20 weeks

Solution. Ans 221: (c) 16-18 weeks


Ref:Read the text below
Sol:
Amniocentesis can be done anytime at and beyond 15 weeks but most commonly done at 16-18 weeks
- When amniocentesis was done at 11-14 weeks it was called as early amniocentesis but had a
high risk of pregnancy loss so not done now
- The cells used are amniocytes or fibroblast or epitheleoid
- The result of fetal karyotyping is obtained after 7-10 days after the produre About 20 -30 ml of
fluid is aspirated
- Procedure related fetal loss rate is 1/300 to 1/500

Answer. c

222. Sperm count parameters by WHO requires minimum of

www.damsdelhi.com Email: info@damsdelhi.com


DAMS CBT 2015 Test -1
a. 10 million/ml

b. 15 million/ml

c. 20 million/ml

d. All of above

Solution. Ans 222: (b) 15 million/ml


Ref:Read the text below
Sol:
The WHO sperm count parameters require the following
- Total count >39 million per ejaculate
- Sperm concentration more than 15 million per ml
- Total motility is more than 40%
- Progressive motility more than 32%
- Sperm morphology more than 4%
- Sperm vitality more than 58%
- Ph>7.2
- Volume >1.5 ml
- Wbc count<1million per ml

Answer. b

223. Ventouse in second stage of labor is contraindicated in :

a. Persistent O.P

b. Heart disease.

c. uterine Inertia

d. Preterm labor.

Solution. Ans 223: (d) Preterm labor.


Ref:Read the text below

Sol: Vaccum delivery should not be used to assist delivery prior to 34 weeks other contraindication to
instrumental delivery are Fetal demineralization disease like osteogenesis imperfect, Connective tissue
disorders,Fetal bleeding diathesis
- The most common cups used are soft cups which are bell shaped
- Most common cup sizes are 50 -70 mm
- It is applied at flexion point which is 6cm posterior to anterior fontanelle
- The suction pressure is 0.8kg/cm2 or 600mmhg

Answer. d

224. Active management of 3 rd stage g labor includes all except


www.damsdelhi.com Email: info@damsdelhi.com
DAMS CBT 2015 Test -1

a. 1/m oxytocin after delivery

b. Controlled cord traction

c. Delayed cord clamping

d. Routine uterine massage

Solution. Ans 224: (d) Routine uterine massage


Ref:Read the text below
Sol:
- Injection oxytocin immediately after delivery of baby is preffered.
- 10iu i/m is the dose
- Delayed cord clamping for atleast 1minute is recommended
- Controlled cord traction is recommended for delivery of placenta intermittent uterine tone
asessment.

Answer. d

225. The device shown below is used to sample

www.damsdelhi.com Email: info@damsdelhi.com


DAMS CBT 2015 Test -1

a. Vaginal walls

b. Endometrium

c. Ectocervix

d. Endocervix

Solution. Ans 225: (d) Endocervix


Ref:Read the text below
Sol:
Cyto Brush
- Dual sampling of endo and ectocervix utilizing one device
- Separation of endo and ectocervical cells in a one-slide technique
- Minimizes false negatives
- Eliminates need for deep endocervical cell harvesting
- Minimizes red blood cell and blood artifact

Answer. d

226. Which of the following is seen in the infant of a diabetic mother :

www.damsdelhi.com Email: info@damsdelhi.com


DAMS CBT 2015 Test -1

a. Hyperkalemia

b. Hypercalcemia

c. Macrocytic anemia

d. Polycythemia

Solution. Ans-226 (d) Polycythemia


Ref: Dutta Obs—287

Sol:
Effect of diabetes on :

Answer. d

227. Maximum level of alpha feto protein is seen in :

a. Fetal serum

b. Placenta

c. Amniotic fluid

d. Maternal serum

Solution. Ans-227: (a) Fetal serum


Ref: Williams Obs- 319;
Sol:
www.damsdelhi.com Email: info@damsdelhi.com
DAMS CBT 2015 Test -1
- “Ordinarily high levels of feto – protein are found in developing fetus and low levels exist in maternal
serum and amniotic fluid.”

Answer. a

228. Drug useful in last stage of endometrial carcinoma is :

a. Danazol

b. OC Pills

c. Cisplatin

d. Levonorgestrol

Solution. Ans-228: (c) Cisplatin


Ref.: Read the text below
Sol :
- Chemotherapy is used in advanced and recurrent cases or in metastatic lesions.
- Progestogens are widely used.
- The response is good in well differentiated carcinoma with adequate oestrogen and
progesterone receptors.
- Cytotoxic drugs are being tried either singly or in combination but without any superiority over
the hormonal therapy.
- The drugs commonly used are adriamycin cisplatin and cyclophosphamide.

Answer. c

229. True regarding normal sperm count :

a. 60-100 lakh/mm3

b. 4-5 million/mm3

c. 60-120 lakh/mm3

d. 60-120 million/ml

Solution. Ans-229: (d) 60-120 million/ml


Ref.: Read the text below
Sol :
- Standard for semen analysis within 2 hours of ejaculation (Average)

www.damsdelhi.com Email: info@damsdelhi.com


DAMS CBT 2015 Test -1

Answer. d

230. All are true about cephalhematoma except

a. Foundoutside the pericranium

b. Expectant management only is needed

c. It appears few hrs after delivery

d. Pitting on pressure is seen

Solution. Ans-230: (a) Foundoutside the pericranium


Ref:Read the text below
Sol:
Causes
A. Prolonged labor
B. Instrumented delivery (e.g. forceps)
II. Pathophysiology
A. Rupture of blood vessels between skull and periosteum
B. Results in subperiosteal blood collection
C. Bleeding limited by Suture lines
III. Signs
A. Cephalhematoma does not cross Suture lines
B. Well-demarcated, fluctuant swelling
C. Most commonly occurs over parietal bone
D. No overlying Skin Discoloration
E. Appears by day 2-3 of life

Answer. a

231. Which among the following causes of neonatal sepsis have a high propensity to produce biliary stasis?

a. E.coli

b. Klebsiella

c. Streptococcus agalactiae

d. Acinetobacter
www.damsdelhi.com Email: info@damsdelhi.com
DAMS CBT 2015 Test -1
Solution. Ans 231: (a) E.coli
Ref: Read the text below
Sol:
- Sepsis is known to produce cholestasis in a neonate, particularly that caused by E.coli.
- The endotoxin produced by E.coli induces biliary stasis & favours lithogenic action of bacterial
glycoproteins.

Answer. a

232. A child with Kwashiorkar will have all of the following except:

a. Flaky paint dermatosis

b. Hepatomegaly

c. Voracious appetite

d. Muscle wasting

Solution. Ans 232: (c) Voracious appetite


Ref: Read the text below
Sol:
The cardinal features of Kwashiorkar include PEM-G
- P: Psychomotor changes
- E: Edema
- M: Muscle wasting (which may be masked by edema, but always present)
- G: Growth retardation
In addition, children with Kwashiorkar have poor appetite, Hepatomegaly, Hypoalbuminemia, Hair
changes (e.g. flag sign) & Skin changes (e.g. Flaky paint dermatosis or Crazy pavement dermatosis).

Answer. c

233. Normal urine output in a 1 year old child:

a. 1 ml/kg/hr

b. 1.5 to 2 ml/kg/hr

c. 2 to 2.5 ml/kg/hr

d. 3 to 3.5 ml/kg/hr

Solution. Ans 233: (b) 1.5 to 2 ml/kg/hr


Ref: Read the text below
Sol:

www.damsdelhi.com Email: info@damsdelhi.com


DAMS CBT 2015 Test -1
- As per PALS-AHA guidelines, the normal urine output in infants is 1.5 to 2 ml/kg/hr. For older children &
adolescents, its 1 ml/kg/hr.
- Oliguria is defined as urine output less than 0.5 ml/kg/hr for children as per the same guidelines.

Answer. b

234. All of the following predict a poor outcome in HIE except:

a. Oliguria In the first 36 hours of life

b. Burst suppression pattern on EEG

c. Hypodensities on CT after 1 month of life

d. Hyperalert status persisting for 18 hrs after birth

Solution. Ans 234: (d) Hyperalert status persisting for 18 hrs after birth
Ref: Read the text below
Sol:
Poor prognostic factors for Hypoxic Ischemic Encephalopathy (HIE) include:
- Sarnat&Sarnat stage 3
- Burst suppression pattern/Isoelectric pattern on EEG
- Urine output <1 ml/kg/hr during first 36 hr of life
- Lack of spontaneous breathing efforts within 20 to 30 min of life
- Persistent hypodensities even 1 month after insult.
- Apgar score 0-3 at 5 min of life
- Decerebrate posturing
- Umbilical cord pH <6.7 or base deficit > 20 mmol/L

Answer. d

235. True statements regarding Late-onset hemorrhagic disease of newborn include the following except:

a. Onset between 21 to 28 days of life

b. Intracranial haemorrhage is most common manifestation

c. Cholestasis is a major risk factor

d. Associated with abetalipoproteinemia deficiency

Solution. Ans 235: (a) Onset between 21 to 28 days of life


Ref: Read the text below
Sol:
Explanation:HEMORRHAGIC DISEASE OF NEWBORN (Ref: Nelson 19th Ed)

www.damsdelhi.com Email: info@damsdelhi.com


DAMS CBT 2015 Test -1

Answer. a

236. Which is true regarding Obstructive TAPVC?

a. Prostaglandin E1 is life-saving

b. Surgical correction to be done around 3 months of age

c. ECMO is life-saving in neonatal period

d. Snowman appearance is seen on CXR

Solution. Ans 236: (c) ECMO is life-saving in neonatal period


Ref: Read the text below
Sol:
TAPVC (Total anomalous pulmonary venous connection) is a cyanotic heart disease characterised by
abnormal connections of pulmonary veins. The pulmonary veins open into right side of heart.All forms
of TAPVC involve mixing of oxygenated & deoxygenated blood. They can be Supra-cardiac, Cardiac,
Infra-cardiac or Mixed.
- The clinical manifestations depend on the presence or absence of obstruction of venous
return.In Obstructive variety(seen commonly in Infra-cardiac variety), severe pulmonary
hypertension develops. This variety commonly manifests as severe cyanosis in neonatal
period & is commonly worsened on starting prostaglandin E1 infusion.
- Diagnosis of Obstructive variety is mainly clinical, confirmed by ECHO.
- Snowman appearance is seen on CXR of non-obstructive TAPVC; not in obstructive type.
- Rx is Emergency surgery in neonatal period. ECMO is life-saving in patients who cannot
undergo immediate surgery.
(Ref: Nelson 19th Ed, Park’s Pediatric Cardiology)

Answer. c

www.damsdelhi.com Email: info@damsdelhi.com


DAMS CBT 2015 Test -1
237. Drug of choice in Diamond-Blackfan anemia is:

a. Androgens

b. Prednisolone

c. ATG

d. G-CSF

Solution. Ans 237: (b) Prednisolone


Ref: Read the text below
Sol:
- Diamond -BlackfanAnemia is also known as Congenital Hypoplastic Anemia.
- Becomes manifest in neonatal period or early infancy as progressive pallor. There is
reticulocytopenia with absent RBC precursors in bone marrow, which is otherwise cellular.
- 25% cases associated with RPS-19 gene mutation on chromosome 19q13.2.
- Corticosteroid are drug of choice, mechanism is unknown.
- In refractory cases, periodic transfusions with chelation therapy is needed. Hematopoietic
stem cell transplantation (HSCT) is curative. Indication of HSCT in this condition is transfusion
dependence.

Answer. b

238. Syndromes associated with neuroblastoma in a child include:

a. Pepper Syndrome

b. Hutchinson Syndrome

c. Neurocristopathy Syndrome

d. All of the above

Solution. Ans 238: (d) All of the above


Ref: Read the text below

Sol:
The syndromes associated with Neuroblastoma include:
- Pepper syndrome: Massive hepatic involvement with or without respiratory distress
- Horner syndrome: Unilateral; associated with a thoracic or cervical location of tumour
- Hutchinson syndrome: Limping & irritability due to bony metastasis
- Opsoclonus-Myoclonus-Ataxia syndrome: Immune mediated paraneoplastic syndrome
- Kerner-Morrison syndrome: Intractable secretory diarrhea due to VIP production
- Neurocristopathy syndrome: Neuroblastoma associated with other neural crest disorders
like Hirschsprung disease or congenital hypoventilation syndrome.
(Ref: Nelson 19th Ed)

www.damsdelhi.com Email: info@damsdelhi.com


DAMS CBT 2015 Test -1
Answer. d

239. Drugs having a role in management of Tuberous Sclerosis include all of the following except:

a. Vigabatrin

b. Sirolimus

c. Everolimus

d. Rapamycin

Solution. Ans 239: (b) Sirolimus


Ref: Read the text below
Sol:
- Tuberous sclerosis is a neurocutaneous syndrome, associated with benign tumours
(rarely malignant) in various organ systems. Treatment is mainly supportive.
- Vigabatrin is drug of choice for infantile spasms seen in tuberous sclerosis.
- Topical Rapamycin is useful in benign skin tumours
- SEGA (Subependymal giant cell astrocytomas) are generally resected surgically or
managed with V-P shunt to treat associated obstructive hydrocephalus but many patients who
cannot undergo surgery, respond to Everolimus.

Answer. b

240. Highest mortality rates in childhood meningitis are observed with which of the following agents?

a. E.coli

b. Pneumococcus

c. H. influenzae

d. N. Meningitides

Solution. Ans 240: (b) Pneumococcus


Ref: Read the text below
Sol:
- As per Nelson 19thed, the mortality rates in childhood meningitis beyond neonatal
period is less than 10%.
- The highest mortality rates are observed for pneumococcal meningitis.
- Among those who survive, 10-20% develop neurodevelopmental sequelae in childhood
pyogenic meningitis.

Answer. b

241. A 2-yr-old boy with the spastic diplegia form of cerebral palsy is being evaluated. MRI of his brain is
www.damsdelhi.com Email: info@damsdelhi.com
DAMS CBT 2015 Test -1
most likely to show:

a. Multicystic encephalomalacia

b. Periventricular leukomalacia

c. Basal ganglia abnormalities

d. Agenesis of the corpus callosum

Solution. Ans 241:(b) Periventricular leukomalacia


Ref: Read the text below
Sol:
- PVL is a common observation in children with CP. It is first observed in the neonatal period in
both term and preterm infants.
- If identified in the neonatal period and extensive, it is a very strong predictor of CP.

Answer. b

242. Which of the following is most appropriate method for obtaining a Urine specimen for culture in a 8
month old girl :

a. Suprapubic aspiration

b. Indwelling catheter sample

c. Clean catch void

d. Urinary bag sample

Solution. Ans-242: (a) Suprapubic aspiration


Ref: Read the text below
Sol :
- The diagnosis of UTI is confirmed by Urine culture.
- Urine is obtained by Suprapubic aspiration or urethral catheterization in children below 2 year
and careful clean catch procedure in older children.
- Sensitivity of suprapubic aspiration is more (99%) than catheterization (95%).

Answer. a

243. One year old male child presented with poor urinary stream since birth. The investigation of choice for
evaluation is:

a. Voiding cystourethrogram

b. USG bladder

www.damsdelhi.com Email: info@damsdelhi.com


DAMS CBT 2015 Test -1
c. Intravenous urography

d. Uroflometry

Solution. Ans-243: (a) Voiding cystourethrogram


Ref: Nelson 17th edition/1802
Sol :
- Poor urinary stream in a male child since birth most probably suggest the diagnosis of Posterior
Urethral value.
- In PUV – Dribbling, abnormal urinary stream, Palpable bladder, and recurrent UTI are usual
presenting Features.
- Diagnosis can be made prenatally by – Maternal Ultrasound which reveals bilateral
hydronephrosis, a distended bladder and Oligohydroamnios.
- Postnatal diagnosis is made by – Voiding cystourethrogram and Endoscopy.

Answer. a

244. Which of the following social skills would be expected to be accomplished by age 3 years ?

a. Engages in parallel play

b. Engages in interactive play

c. Dresses self completely

d. Bathes self skillfully

Solution. Ans-244: (a) Engages in parallel play


Ref.: Read the text Below
Sol :
- Engaging in interactive play is not accomplished until age 4.
- Choice C is incorrect; this is not accomplished until age 5.
- Choice D is incorrect; this is not accomplished until after age 5.

Answer. a

245. Characteristic presenting symptoms and signs of childhood leukemia include:

a. Pallor

b. Lymphadenopathy

c. Bone or joint pain

d. All of the above

Solution. Ans 245:(d) All of the above


Ref: Read the text below

www.damsdelhi.com Email: info@damsdelhi.com


DAMS CBT 2015 Test -1
Sol:
- Most children with leukemia present with symptoms of less than 4 weeks' duration. Most children with ALL
have pallor, 50% have petechiae or mucous membrane bleeding, 60% have lymphadenopathy, 25% have fever,
and about 25% have bone pain and arthralgias caused by leukemic infiltration of the perichondral bone or joint
or by leukemic expansion of the marrow cavity.

Answer. d

246. A5-year-old girl diagnosed with pauciarticular juvenile idiopathic arthritis has a positive antinuclear
antibody test. Which of the following would most likely be found in this patient?

a. Pericarditis

b. Nephritis

c. Uveitis

d. Splenomegaly

Solution. Ans 246:(c) Uveitis


Ref: Read the text below
Sol:
- There are three classic clinical presentations of juvenile idiopathic arthritis (JIA)—acute
systemic, pauciarticular, and polyarticular.
- The frequency of ANAseropositivity is highest in girls with younger age at onset of disease.
- Among children who have oligoarthritis and uveitis there is highest prevalence of positive ANAs.
- Determination of ANAseropositivity is therefore helpful in identifying children most at risk for chronic
uveitis.

Answer. c

247. Defective hepatic conjugation is seen in all the following except

a. Neonatal jaundice

b. Gilbert syndrome

c. Crigler najjar syndrome

d. Novobiocin therapy

Solution. Ans-247: (d) Novobiocin therapy


Ref.: Read the text below
Sol :
- Gilbert syndrome is due to decrease activity of uridyl glucuronyl transferase which causes
impaired bilirubin conjugation.
- In crigler-najjar syndrome there is decrease activity of UDP-Glucuronyl transferase which also
causes impaired conjugation.

www.damsdelhi.com Email: info@damsdelhi.com


DAMS CBT 2015 Test -1
- Neonatal jaundice is a physiological jaundice in which conjugation of bilirubin is impaired.
- All these three condition leads to unconjugated hyperbilirubinemia.

Answer. d

248. True about Criggler najjar type-II syndrome is

a. Complete diglucuronide deficiency

b. Recessive trait

c. Kernicterus is seen

d. Phenobarbitone not useful

Solution. Ans-248: (b) Recessive trait


Ref.: Read the text below
Sol :
Crigler–Najjar syndrome or CNS is a rare disorder affecting the metabolismof bilirubin, a chemical formed
from the breakdown of red blood cells. The disorder results in an inherited form of nonhemolytic jaundice,
which results in high levels of unconjugated bilirubin and often leads to brain
damage in infants.
This syndrome is divided into type I and type II, with the latter sometimes calledArias syndrome.

Type II differs from type I in several aspects:


- Bilirubin levels are generally below 345 μmol/L (100–430; thus, there is overlap), and some cases
are only detected later in life.
- Because of lower serum bilirubin, kernicterus is rare in type II.
- Bile is pigmented, instead of pale in type I or dark as normal, and monoconjugates constitute the
largest fraction of bile conjugates.
- UGT1A1 is present at reduced but detectable levels (typically <10% of normal), because of
single base pair mutations.
- Therefore, treatment with phenobarbital is effective, generally with a decrease of at least 25% in
serum bilirubin. In fact, this can be used, along with these other factors, to differentiate type I and II.
- The inheritance pattern of Crigler–Najjar syndrome type II has been difficult to determine but is
generally considered to be autosomal recessive

Answer. b

249. A term baby of an uncomplicated pregnancy is born limp, cyanotic, and apneic after a difficult vaginal
delivery. Possible considerations in the differential diagnosis include all of the following except:

a. Prolapsed umbilical cord

b. Central nervous system trauma

c. Administration of morphine to the mother


www.damsdelhi.com Email: info@damsdelhi.com
DAMS CBT 2015 Test -1

d. Klumpke paralysis

Solution. Ans 249:(d) Klumpke paralysis


Ref: Read the text below
Sol:
- Klumpke paralysis involves injury to the 7th and 8th cervical nerves and the 1st thoracic nerve.
- It is usually unilateral, due to traction injury of the brachial plexus.

Answer. d

250. A35-week-gestation infant is delivered weighing 3.9 kg, with an omphalocele and a large tongue.No
other abnormalities are detected. Which of the following is the most likely diagnosis?

a. congenital hypothyroidism

b. Trisomy 18

c. Trisomy 13

d. Beckwith-Wiedemann syndrome

Solution. Ans 250:(d) Beckwith-Wiedemann syndrome


Ref: Read the text below
Sol:
- Beckwith-Wiedemann syndrome encompasses abdominal wall defects (especially
omphalocele), macroglossia, and macrosomia.
- Its recognition is critical in the immediate newborn period as about one-third to one-half of cases have
hypoglycemia.
- Cases are usually sporadic, the gene is located at 11p15. The characteristic phenotype occurs due to a variety
of different genetic mechanisms, which all result in a dosage imbalance of a number of genes clustered at
11p15.
- Patients with this disorder have an increased risk of abdominal tumors (ie, hepatoblastoma and Wilms tumor)
during childhood. Regular screening for these tumors during the early part of childhood is warranted.

Answer. d

251. What sign is being demonstrated in this patient?

www.damsdelhi.com Email: info@damsdelhi.com


DAMS CBT 2015 Test -1

a. Nikolsky sign

b. Asboe Hansen sign

c. Auspitz sign

d. Koebner sign

Solution. Ans 251: (b) Asboe Hansen sign


Ref:Read the text below
Sol:
- “Asboe Hansen sign”or “Bulla spread sign”means when you apply gentle pressure on an intact
bulla,the fluid spreads away under the skin away from the site of pressure.
- This spread occurs because of loss of desmosomes(acantholysis).

- This sign is positive in all varieties of pemphigus and many cases of subepidermal blisters,
including bullous pemphigoid, dermatitis herpetiformis, epidermolysisbullosaacquisita,
cicatricialpemphigoid, dystrophic epidermolysisbullosa,
- Stevens-Johnson syndrome and toxic epidermal necrolysis.
- Skin biopsy of Pemphigus vulgaris on Direct Immunofluorescence shows “Fish net pattern”

Answer. b

252. A 30 year old female presents with well defined,irregular brown red plaques on both shins(Image shown
below).What could be the possible underlying systemic disorder she may have?

www.damsdelhi.com Email: info@damsdelhi.com


DAMS CBT 2015 Test -1

a. Diabetes

b. Hypertension

c. Hypothyroidism

d. Hyperthyroidism

Solution. Ans 252: (a) Diabetes


Ref:Read the text below
Sol:
- The diagnosis is “Necrobiosislipoidicadiabeticorum”which is seen in young female diabetics.
- Treatment is with steroids,but often unsatisfactory.Chronic stage of this condition may present
as ulcers.

Answer. a

253. A 2 year old child presents with patches and crusted plaques in face,neck and extensor surfaces of the
body.Additional finding is shown in the image.What is the diagnosis?

www.damsdelhi.com Email: info@damsdelhi.com


DAMS CBT 2015 Test -1

a. Urticaria

b. Seborrheic dermatitis

c. Scabies

d. Atopic dermatitis

Solution. Ans 253: (d) Atopic dermatitis


Ref:Read the text below
Sol:
- The clinical history as well as the finding of Denny-Morgan fold(extrafold of skin beneath the
lower eyelid) is suggestive of Atopic dermatitis.

Answer. d

254. Hyper extensibility with normal elastic recoil is seen in

a. Ehlers-Danlos syndrome

b. Pseudoxanthoma elasticum

c. Cutis laxa

d. Scleroderma

Solution. Ans 254: (a) Ehlers-Danlos syndrome


Ref:Read the text below
Sol:
- Collagen defects allow the hyperextensibility of skin and loose jointedness but skin bounces
back because of normal elastin as in Ehlers Danlos syndrome.
- However, in elastin defects like cutis laxa and pseudoxanthoma elasticum skin looses its elastic
recoil and hangs in loose folds.
- Scleroderma on the other hand is characterized by excessive abnormal collagen producing
binding down of skin that is difficult to pinch and pick into folds.

Answer. a

255. A 5 yr old male child has multiple hyperpigmented macules over the trunk. On rubbing the lesion with
www.damsdelhi.com Email: info@damsdelhi.com
DAMS CBT 2015 Test -1
rounded end of pen, he developed urticarial wheal, confined to border of lesion, most likely diagnosis is-

a. Fixed drug eruption

b. Lichen planus

c. Urticaria pigmentosa

d. Urticarial vasculitis

Solution. Ans 255: (c) Urticaria pigmentosa


Ref:Read the text below
Sol:
- It is urticaria pigmentosa.
- It is cutaneous mastocytosis presenting with pigmented lesions.

- On rubbing the lesions mast cells degranulate releasing histamine.


- This produces the wheal and flare of the lesion. It is known as Darier’s sign.

Answer. c

256. Which of the following is a suprabasal split

a. Pemphigus vegetans

b. Pemphigus foliaceus

c. Fogo selvagem

d. Staphylococcal scalded skin syndrome

Solution. Ans 256: (a) Pemphigus vegetans


Ref:Read the text below
Sol:
- Pemphigus vulgaris and vegetans have suprabasal split.
- Pemphigus foliaceus and its variants like fogo selvagem, pemphigus erythematosus have
subcorneal split.
- SSSS and bullous impetigo are mediated by epidermolytic toxins that cleave the desmoglein 1
and produce a subcorneal split.

Answer. a

257. Defect in gene ATP2C1 is associated with

a. Hailey-Hailey disease

www.damsdelhi.com Email: info@damsdelhi.com


DAMS CBT 2015 Test -1
b. Darier’s disease

c. Grover’s disease

d. Dermatitis herpetiformis

Solution. Ans 257: (a) Hailey-Hailey disease


Ref:Read the text below
Sol:
Hailey-Hailey disease
- Autosomal dominant inheritance
- Defect in gene ATP2C1, (ATPase, Ca2+ transporting, type2C, member1, a regulator of
cytoplasmic calcium concentration) leads to acantholysis.
 Clinically presents in 2nd to 3rd decade with recurrent intertrigo with vesicles, erosions, crusts,
erythema, maceration and foul smell in intertriginous areas like groin, axillae and neck.
- It can be confused with intertrigo, candidiasis & tinea cruris
Histology: Extensive epidermal acantholysis in mid epidermis with “dilapidated
or crumbling brick wall” appearance.
Darier’s disease is associated with ATP2A2 defect

Answer. a

258. A female sex worker presents with vaginal discharge. In absence of speculum examination which

conditions are to be treated for in syndromic management:


a. Gonorrhea + Syphilis

b. Gonorrhea + Syphilis + Candidiasis

c. Gonorrhea + Syphilis + Candidiasis + Trichomonas vaginalis

d. Gonorrhea + Syphilis + Candidiasis + Trichomonas vaginalis + Bacterial Vaginosis

Solution. Ans 258: (d) Gonorrhea + Syphilis + Candidiasis + Trichomonas vaginalis + Bacterial Vaginosis
Ref:Read the text below
Sol:
- In Syndromic approach of vaginal discharge if patient or partner is high risk for STDs or if speculum
examination can not be done then both vaginal discharge (Trichomonas, candida, bacterial vaginosis) and
cervical discharge (gonorrhea and chlamydia) should be treated.

Answer. d

259. School of fish appearance is seen in

a. Chancroid

b. Hunterian chancre

c. Donovanosis

www.damsdelhi.com Email: info@damsdelhi.com


DAMS CBT 2015 Test -1

d. Mycobacterial Granuloma

Solution. Ans 259: (a) Chancroid


Ref:Read the text below
Sol:
- Chancroid is an acute STD characterized by a painful ulcer at the site of inoculation, usually on
the external genitalia, and the development of suppurative regional lymphadenopathy.
- Gram’s Stain Of scrapings from ulcer base or pus from bubo, show small clusters or parallel
chains of gram-negative rods (School of Fish appearance). Interpretation is difficult due to
presence of contaminating organisms in ulcers.
- Hunterian chancre is syphilitic.
- Donovanosis (Granuloma inguinale) is a mildly contagious, chronic, indolent, progressive,
autoinoculable, ulcerative disease involving the skin and lymphatics of the genital and perianal
areas.
- It is caused by Calymmatobacterium granulomatis, an encapsulated intracellular gram-negative
rod. Touch or Crush Preparation
- Of punch biopsy stained withWright’s or Giemsa’s stain shows Donovan bodies in cytoplasm of
macrophages. Mycobacterial infections do not have any school of fish appearance.

Answer. a

260. Which Vitamin deficiency can produce the typical skin change shown?

www.damsdelhi.com Email: info@damsdelhi.com


DAMS CBT 2015 Test -1

a. Vitamin B1

b. Vitamin B2

c. Vitamin B3

d. Vitamin B6

Solution. Ans 260: (c) Vitamin B3


Ref:Read the text below
Sol:
- Casal’s necklace pattern of skin lesion around the neck is a feature of Pellagra.
- The recommended daily allowance of Niacin is 6mg/1000 kcal of energy intake.

Answer. c

www.damsdelhi.com Email: info@damsdelhi.com


DAMS CBT 2015 Test -1
261. Neuroleptanalgesia has all of the following properties except:

a. Droperidol and fentanyl are commonly used

b. It can be used with nitrous oxide to provide neuroleptanesthesia

c. Hypertension is a common consequence

d. Confusion and mental depression can occur as adverse effects

Solution. Ans 261: (c) Hypertension is a common consequence

Ref– Read the text below


Sol:
- Form of analgesia achieved by the concurrent administration of a neuroleptic such as
droperidol and an analgesic such as fentanyl.
- Anxiety, motor activity, and sensitivity to painful stimuli are reduced; the person is quiet and
indifferent to surroundings and is able to respond to commands.
- If nitrous oxide with oxygen is also administered, neuroleptanalgesia can be converted to
neuroleptanesthesia.
- Neuroleptanalgesia or neuroleptanesthesia is contraindicated in patients receiving monoamine
oxidase inhibitors (MAOI), in those who abuse drugs or alcohol, or those with Parkinson disease.

Answer. c

262. Which of the following local anesthetics is preferable in patient with pseudocholinesterase deficiency?

a. Procaine

b. Ropivacaine

c. Tetracaine

d. Benzocaine

Solution. Ans 262: (b) Ropivacaine


Ref– Read the text below
Sol:
- The new long-acting amino-amide local anesthetic, ropivacaine, combines the anesthetic
potency and long duration of action of bupivacaine with a toxicity profile intermediate
between bupivacaine and lidocaine.
 Ropivacaine undergoes hepatic biotransformation and renal clearance of the intact drug
accounts for a minor proportion of total clearance. Peak plasma levels of ropivacaine following
epidural of peripheral nerve block may rise twice as high as levels of bupivacaine, likely due to
ropivacaine's decreased lipid solubility and volume of distribution
- Ropivacaine is preferable in patient with pseudocholinesterase deficiency

Answer. b

www.damsdelhi.com Email: info@damsdelhi.com


DAMS CBT 2015 Test -1

263. The maximum safe dose of lignocaine that can be used without adrenaline is

a. 1 mg/kg

b. 3 mg/kg

c. 5 mg/kg

d. 7 mg/kg

Solution. Ans-263: (b) 3 mg/kg


Ref.: KD Tripathi-351-61
Sol :
Maximum recommended dose of
- Lignocaine without adrenaline is 3 mg/kg (max 200 mg) and
- Lignocaine with adrenaline is 7 mg/kg (max 500 mg).

Answer. b

264. The ratio of chest compression to manual ventilation for a single rescuer in CPR is

a. 15 : 2

b. 10 : 2

c. 12 : 2

d. 5 : 1

Solution. Ans-264: (a) 15 : 2


Ref.: OP Ghai, - 689, 694
Sol :
- The ratio of chest compression to manual ventilation for a single rescuer in CPR is 15
(compressions) : 2 (breaths) ; for two rescuers is 5 (compressions) : 1 (breath).

Answer. a

265. Which of the following muscle relaxants is known to be associated with masseter spasm ?

a. Pancuronium

b. Succinyl-choline

c. Vecuronium

www.damsdelhi.com Email: info@damsdelhi.com


DAMS CBT 2015 Test -1
d. d-tubocurarine

Solution. Ans-265: (b) Succinyl-choline


Ref.: Lee’s Synopsis of Anesthesia, 13 th edn. Pg. 354
Sol :
- If a patient develops severe muscle spasm after suxamethonium lasting several minutes such
that it is very difficult to open the mouth to intubate there is significant possibility (20-30%) that
they may have susceptibility to malignant hyperthermia.

Answer. b

266. Which of the following local anesthetic has a vasoconstrictor action

a. Lignocaine

b. Bupivacaine

c. Cocaine

d. Procaine

Solution. Ans-266: (c) Cocaine


Ref.: KD Tripathi- 356-57
Sol :
- The sympathetic nervous system is stimulated by cocaine.
- As cocaine is a powerful vasoconstrictor, adrenaline added to it is not only unnecessary but also
increases the risk of cardiac dysrhythmias and VF. The two drugs should not be used together.

Answer. c

267. Which of the following, when given as an induction agent, causes the least amount of respiratory
depression?

a. Thiopental
b. Midazolam

c. Fentanyl

d. Ketamine

Solution. Ans-267: (d) Ketamine


Ref: Read the text below.
Sol:
- Ketamine causes the least amount of central respiratory depression.

- Although apnea has been reported after high doses of ketamine, it is extremely rare.
- In contrast, the other four agents all cause doser-related respiratory depression.

www.damsdelhi.com Email: info@damsdelhi.com


DAMS CBT 2015 Test -1
Answer. d

268. Which of the following inductions agents would be likely to cause hypotension when used in a healthy
40-year-old man who has been NPO for 8 hrs ?

a. Thiopental 4 mg/kg

b. Propofol 2.5 mg/kg

c. Ketamine 2 mg/kg

d. None of the above

Solution. Ans-268: (b) Propofol 2.5 mg/kg


Ref: Read the text below.
Sol:
- In a healthy man who is euvolemic, 4 mg/kg is an induction dose of thiopental that should cause
no significant changes in hemodynamics.
- Thiopental’s predominant cardiovascular effect is venodilation, with some myocardial
depressant effect that causes a decrease in cardiac output.
- However, the depressant effect of thiopental is minimized in a healthy person because heart rate
also increases secondary to the only slightly affected baroreflex mechanism. No change in
systemic vascular resistance occurs.
- Therefore, no significant change in blood pressure occurs. The usual induction dose of ketamine
is 2 mg/kg.
- Because of its sympathomimetic effect, ketamine increases systemic vascular resistance and
heart rate and thus causes an increase in blood pressure.
- Propofol at 2.5 mg/kg could cause 25 % to 40% decrease in blood pressure.

Answer. b

269. Factors that increase the magnitude of opioid-induced respiratory depression include all the following
except

a. Increased age

b. Concomitant use of benzodiazepines

c. Decreased hepatic blood flow


d. Increased clearance with decreased volume of distribution

Solution. Ans-269: (d) Increased clearance with decreased volume of distribution


Ref: Read the text below.
Sol:
- An increased clearance with a decreased volume of distribution will decrease the half-life of any
opioid and, therefore, all the opioid’s properties will be of shorter duration including respiratory
depression.
- When opioids are combined with any other respiratory depressant, the effects are synergistic. In
www.damsdelhi.com Email: info@damsdelhi.com
DAMS CBT 2015 Test -1
addition the magnitude of respiratory depression will increase with bolus injections of
narcotics, increased age, and decreased hepatic blood flow.

Answer. d

270. Adequate anesthesia for surgical incision is least likely to be provided by

a. 0.5% halothane in 66% nitrous oxide/33% O 2

b. 1.0% halothane in 100% O 2

c. 0.5% isoflurane in 70% nitrous oxide/30% O 2

d. 1.0% isoflurane in 100% O 2

Solution. Ans-270: (d) 1.0% isoflurane in 100% O 2


Ref: Read the text below.
Sol:
- MAC for halothane is 0.77% in 100% O2 and 0.3% in 66% nitrous oxide/33% O 2.
- MAC for isoflurane is 1.15% in 100% O2 and 0.5% in 70% nitrous oxide.

Answer. d

271. Isotope used in bone tumour:

a. I-131

b. I-125

c. Cr-51

d. Sr90

Solution. Ans-271: (d) Sr90


Ref: Read the text below
Sol:
- I-131 is used for treatment of thyroid tumours, I-125 is used for Radio-immunoassay and
brachgytherapy .
- Sr90 is a calcium analogue and Bone seeker and used for bone tumours.

Answer. d

272. Unilateral smooth enlarged kidney with striated nephrogram is seen in

a. Acute pyelonephritis

www.damsdelhi.com Email: info@damsdelhi.com


DAMS CBT 2015 Test -1
b. Chronic pyelopnephritis

c. Autosomal recessive polycystic kidney disease

d. Multicystic dysplastic kidney

Solution. Ans-272: (a) Acute pyelonephritis


Ref: Read the text below
Sol:
- Striated nephrogram is seen in Autosomal recessive PCKD and in acute pyelonphritis.
- Other findings of Acute pyelopnephritis are on ultrasound: normal or enlarged kindey, with
altered echogenicity areas showing areas of reduced vascularity on Doppler.

Answer. a

273. What is meant by Doppler effect?

a. Change in frequency

b. Change in Direction

c. Change in Velocity

d. Change in amplitude

Solution. Ans-273: (a) Change in frequency


Ref: Read the text below
Sol:
- Doppler effect means : moving objects show frequency shift and is utilized for assessing for
blood flow.

Answer. a

274. Identify the finding shown in the radiological Image

www.damsdelhi.com Email: info@damsdelhi.com


DAMS CBT 2015 Test -1

www.damsdelhi.com Email: info@damsdelhi.com


DAMS CBT 2015 Test -1
a. Acute infarct

b. Acute Hemorrhage

c. Chronic infarct

d. Chronic Hemorrhage

Solution. Ans-274: (a) Acute infarct


Ref: Read the text below
Sol:
- This is diffusion weighted image, which is special MRI image based on Brownian motion.
- This image shows a left sided left MCA territory infarct.

Answer. a

275. Roentgen used x-ray for the first time in which year?

a. 1880

b. 1885

c. 1890

d. 1895

Solution. Ans-275: (d) 1895


Ref: Read the text below
Sol:
- Xrays were discovered by German, WC Roengten, 8 November 1895 and Nov 8 is international
day of radiology.
- Theme for 2015 is Pediatric imaging.

Answer. d

276. Which cardiac chamber is enlarged in this CXR?

www.damsdelhi.com Email: info@damsdelhi.com


DAMS CBT 2015 Test -1

www.damsdelhi.com Email: info@damsdelhi.com


DAMS CBT 2015 Test -1
a. LA

b. LV

c. RA

d. RV

Solution. Ans-276: (a) LA


Ref: Read the text below
Sol:
LA enlargement
- This image shows carinal widening and double atrial shadow, seen in RHD.

Answer. a

277. Which of the following is not based on ionizing radiation ?

a. X-ray

b. CT scan

c. PET scan

d. USG

Solution. Ans-277: (d) USG


Ref: Read the text below
Sol:
- Xrays, alpha , beta and gamma rays are ionizing radiation.
- USG is based on sound which is non-ionizing radiation.

Answer. d

278. On Chest Xray right heart border is mainly formed by

a. RA

b. RV

c. Ascending aorta

d. SVC

Solution. Ans-278: (a) RA

Ref: Read the text below


Sol:
- RA Right atrium is major part of right cardiac boder on CXR.
www.damsdelhi.com Email: info@damsdelhi.com
DAMS CBT 2015 Test -1

Answer. a

279. Investigation of choice for urinary calculi

a. NCCT

b. CECT

c. USG

d. IVP

Solution. Ans-279: (a) NCCT


Ref: Read the text below
Sol:
- NCCT is considered the IOC for urinary stones.
- IVP is not done in most of places today.

Answer. a

280. Miliary lesions on CXR in lung are not seen in

a. Pneumoconiosis

b. Sarcoidosis

c. Staphylococcal pneumonia

d. Fat embolism

Solution. Ans-280: (c) Staphylococcal pneumonia


Ref: Read the text below
Sol:
Staphyloccal pneumonia.
- Rest all are causes of military mottling.
- Other causes of military mottling are tuberculosis, metastases, alveolar microlithiasis

Answer. c

281. Contraindications to open reduction and internal fixation (ORIF) are all except

a. Acute osteomyelitis

b. Osteoporotic bone
www.damsdelhi.com Email: info@damsdelhi.com
DAMS CBT 2015 Test -1

c. Soft tissues overlying the fracture or surgical approach that are poor in quality due to burns

d. Old myocardial infarction

Solution. Ans-281: (d) Old myocardial infarction


Ref.: Read the text below
Sol :
Contraindications to surgical reconstruction (open reduction and internal fixation (ORIF) are as
follows:
- Active infection (local or systemic) or osteomyelitis
- Cases in which amputation would better serve the limb and the patient.
- Medical conditions that contraindicate surgery of anesthesia (eg, recent myocardial infarction)
- Osteoporotic bone that is too weak to sustain internal or external fixation.
- Soft tissues overlying the fracture or surgical approach that are poor in quality due to burns,
surgical scars, or infection (In such scenarios, soft tissue coverage is recommended.)

Answer. d

282. Swan-neck deformities, boutonniere deformities, ulnar deviation of fingers at MCP joints (late findings)
are seen in :

a. Osteoarthritis

b. Rheumatoid arthritis

c. Septic arthritis

d. Gout

Solution. Ans-282: (b) Rheumatoid arthritis


Ref.: Read the text below
Sol :
- With progression of arthritis, characteristic deformities of RA become apparent.
- These include ulnar deviation of the finger at the MCP joints, subluxation of the MCP joints
with the proximal phalanx slipping to the volar side of the metacarpal heads, hyperextension of
the PIP joint with flexion of the DIP joint (Swan-neck deformity), flexion of the PIP joint ith
hyperextension of the DIP joint (boutonniere/buttion-hole deformity), z-shaped deformity of the
thumb from subluxation of the first MCP joint and compensatory hyperextension of the IP joint,
and drooping of the ring and little fingers resulting from rupture of the extensor tendons at the
point of crossing the inflamed eroded ulnar styloid

Answer. b

283. Stage 2 of Destruction in rheumatoid arthritis are characterized by :

a. Persistent chronic inflammation and pannus formation a peri articular erosions secondary to release of
proteolytic enzymes
www.damsdelhi.com Email: info@damsdelhi.com
DAMS CBT 2015 Test -1

b. Pannus and osteoclastic resorption of bone

c. Pannus and osteoclastic resorption around tendons.

d. All are true.

Solution. Ans-283: (d) All are true.


Ref.: Read the text below
Sol :
Stage 2 of Destruction in rheumatoid arthritis
- Persistent chronic inflammation and pannus formation – peri articular erosions secondary to
release of proteolytic enzymes, pannus and osteoclastic resorption of bone, similar changes
occur around tendons.
- X-Ray : Marginal bony erosions, reduced joint space, especially proximal joints of the hands,
feet, wrists and AC joints

Answer. d

284. Little finger of the hand corresponds to which dermatome –

a. C6 dermatome

b. C7 dermatome

c. C8 dermatome

d. T1 dermatome

Solution. Ans-284: (c) C8 dermatome


Ref: Apley’s system of orthopaedics & fracture,9th ed.page,229,272
Sol :

Clinically important dermatomes


Upper extremity
- C6 - Thumb
- C7 - Middle finger
- C8 - Little finger
- T1 - Inner forearm
- T2 - Upper inner arm
Lower extremity
- L3 - Knee
- L4 - Medial malleolus
- L5 - Dorsum of foot
- L5 - Toes 1-3
- S1 - Toes 4 and 5; lateral malleolus
Other
- C2 and C3 - Posterior head and neck
- T4 - Nipple
- T10 - Umbilicus
www.damsdelhi.com Email: info@damsdelhi.com
DAMS CBT 2015 Test -1

Answer. c

285. After lifting something heavy from ground,patient complaints of backpain, which is radiating to lateral leg
& great toe of lower limb. Most probable diagnosis would be-

a. L5-S1 disc prolapse

b. L4-L5 disc prolapse

c. L3-L4 disc prolapse

d. L5 fracture

Solution. Ans-285: (b) L4-L5 disc prolapse


Ref: Essential orthopaedics,Maheshwari,3rd edition page230,231
Sol :

The Extensor Hallucis longus muscle is exclusively supplied by L5 root & its weakness is easily detected by
asking the patient to dorsiflex the Big toe against resistance.Sensory loss may be merely the blunting of
sensation or hypoesthesia in the dermatome of the affected root.

Answer. b

286. All of the following can occur as complications of fracture of neck of femur except –

www.damsdelhi.com Email: info@damsdelhi.com


DAMS CBT 2015 Test -1

a. Shortening

b. Non-union

c. Mal-union

d. Avascular necrosis

Solution. Ans-286: (c) Mal-union


Ref: Apley’s system of orthopaedics & fracture,9th ed.page 852-855
Sol :
Current Essentials Orthopaedics.
Malunion(Varus & external rotation deformities are common) is a complication of inter trochanteric fractures
of femur while rest are known complications of fracture neck of femur. Avascular Necrosis in about 30% with
displaced fracture & 10% in those with undisplaced fracture.Non union in more than 30% of all fracture neck
of femur.
Fracture Neck of femur-
Most often due to falls; risk of injury is related to conditions that increase the probability of falls and those
that decrease a person’s intrinsic ability to withstand trauma
Patients present with severe hip pain, inability to ambulate, and a shortened limb on the fracture side, when
displaced
Tenderness to palpation over the injured hip
Hip is externally rotated and abducted when displaced
Range of motion (ROM) is limited by pain at extremes when undisplaced
Radiography is the first-line modality for imaging and classifying femoral neck fractures
CT is the most useful test for evaluating bony injury, but sometimes fractures in the axial plane can be
missed
MRI is both sensitive and specific in detecting undisplaced femoral neck fractures, because it can show both
the actual fracture line and the resulting bone marrow edema
Garden classification: nondisplaced fractures (20%)—
- type 1: stressfracture, valgus impaction of head
- type 2: complete, nondisplacedfracture; displaced fractures

- type 3: varus displacement of the femoral head


- type 4: fracture fragments, completely displaced
- Differential Diagnosis
Pathologic fracture or impending fracture
Intertrochanteric fractures
Iliopsoas bursitis
Lumbar radicular pain
Psoas abscess
- Treatment

Treatment is almost always by surgical intervention; this allows patients to ambulate with minimum delay
Nondisplaced fractures and femoral neck fractures in young patients are usually treated with closed reduction
and percutaneous pinning; elderly patients or those with Garden type 3 or 4 fractures are usually treated with
hemiarthroplasty

Answer. c

287. Following an accident a person is not able to abduct his shoulder & Flexion movement of elbow joint is
www.damsdelhi.com Email: info@damsdelhi.com
DAMS CBT 2015 Test -1
also not possible,rest all movements are normal. Most likely site of injury?

a. Upper trunk

b. Shoulder dislocation

c. Medial cord

d. Lateral cord

Solution. Ans-287: (a) Upper trunk


Ref: Apley’s system of orthopaedics & fracture,9th ed.page 277
Sol :
- The upper plexus injuries ( C5 & C6) the shoulder abductors and external rotators and the
forearm supinators are paralysed.sensory loss involves the outer aspect of arm & fore-arm.
- Abduction of shoulder is attributed to supraspinatus , deltoid , serratus anterior and trapezius .
- Flexion of elbow is mainly attributed to biceps brachii.
- So the injury must be to the upper trunk of brachial plexus involving the nerve roots C5 and C6

Answer. a

288. A patient presents with recurrent posterior dislocation of shoulder.Most probable site for Hill- Sach’s
lesion on xray would be-

a. Anteromedial

b. Anterior

c. Posteromedial

d. Posterior

Solution. Ans-288: (a) Anteromedial


Ref:www.learningradiology.com/archives04/COW%20105Posterior
%20dislocation/postdislocationcorrect.htm
Sol :

o Posterior shoulder dislocation (2-4%)


- Causes
- Traumatic
o Convulsive disorders or electroshock therapy
- Nontraumatic
o Congenital or developmental
o May be done voluntarily, especially in children
- Usually due to axial loading of an adducted and internally rotated arm
- In >50% unrecognized initially and subsequently misdiagnosed as frozen
shoulder

- May be difficult to see on AP radiograph


www.damsdelhi.com Email: info@damsdelhi.com
DAMS CBT 2015 Test -1
- Typically, a scapular Y view or transthoracic lateral of the humeral head
demonstrate a posterior dislocation better
- Imaging signs of posterior dislocation
- Rim sign (66%) = distance between medial border of humeral head and
anterior glenoid rim >6 mm
- Humeral head is fixed in internal rotation no matter how forearm is
turned – “lightbulb sign”
- May be associated with:
o Trough sign (75%) = "reverse Hill-Sachs" = compression
fracture of anteromedial humeral head
o Fracture of posterior glenoid rim
o Avulsion fracture of lesser tuberosity
- Isolated fractures of the lesser tuberosity should raise
suspicion of an associated posterior dislocation

www.damsdelhi.com Email: info@damsdelhi.com


DAMS CBT 2015 Test -1

Frontal radiograph of shoulder shows a humeral head in internal rotation (lightbulb sign), decreased distance
between humeral head and glenoid (red arrow) and a 'trough fracture" or "reverse Hill-Sachs deformity of the
medial humeral head (blue arrow). The scapular Y view, shown below, demonstrates the humeral head
(yellow arrow) lies posterior to the glenoid and beneath the acromion (green arrow).

Answer. a

www.damsdelhi.com Email: info@damsdelhi.com


DAMS CBT 2015 Test -1

289. Avascular necrosis in fractured proximal fragment of scaphoid bone is due to -

a. Retrograde blood flow in scaphoid

b. The proximal fragment is articulating with radius

c. The proximal fragment is comminuted

d. Immobilization of proximal fragment is difficult.

Solution. Ans-289: (a) Retrograde blood flow in scaphoid


Ref. Apley’s system of orthopaedics & fracture,9th ed.page 780-81
Sol :
- Scaphoid fractures are among the most common hand fractures in adults. The geometry of the
scaphoid as it relates to its retrograde blood supply renders it particularly prone to avascular
necrosis and other fracture complications.
- Fractures should be followed to monitor healing progress in order to ensure the eventual
development of bridging bone across the fracture line, usually best appreciated on CT. Proper
treatment of scaphoid fractures and assessment of fracture healing can minimize the occurrence
of non-unions and associated arthritic changes.
- Scaphoid’s major blood supply is through radial artery. perforating branches from the radial
artery pass through thr dorsal ridge and supply approx 75% of the intra osseous blood. Through
retrograde blood flow dorsal branches also supply the entire proximal pole . distal pole has its
own abundant blood supply from volar branches of the radial artery that constitutes to 25 % of
intraosseous blood flow

Answer. a

290. Most common cause of bone disease(especially in women) in India is–

a. Steroid-induced osteoporosis

b. Nutritional deficiency

c. Paget’s disease

d. Sarcoidosis

Solution. Ans-290: (b) Nutritional deficiency


Ref: Read the text below
Sol :
Nutritional bone disease is defined as a syndrome of bone disease and deformities in which the bone is
affected as a tissue, primarily as a consequence to deficiencies of vitamin D and calcium; or imbalances of the
nutrients which are critically important for the growth and development of the bone; its mineralization and
maintenance of calcium homeostasis and the structural integrity and health of the skeleton.

www.damsdelhi.com Email: info@damsdelhi.com


DAMS CBT 2015 Test -1
Answer. b

291. Which one of the following is not a feature of catatonic schizophrenia

a. Cataplexy

b. Catalepsy

c. Flexibilitas cereas

d. Command automatism

Solution. Ans-291: (a) Cataplexy

Ref:Read the text below


Sol:
- Cataplexy is a sudden and transient episode of loss of muscle tone, often triggered by emotions.

- It is a rare disease (prevalence of fewer than 5 per 10,000 in the community), but frequently
affects people who have narcolepsy, a disorder whose principal signs are EDS (Excessive
Daytime Sleepiness), sleep attacks, sleep paralysis,hypnagogic hallucinations and disturbed
night-time sleep.
- Cataplexy is sometimes confused with epilepsy.It is seen in catatonic schizophrenia.

Answer. a

292. Defence mechanism not seen in OCD

a. Isolation

b. Projection

c. Reaction formation

d. Undoing

Solution. Ans-292: (b) Projection


Ref:Read the text below
Sol:
Projection
- In Freudian psychology, Psychological projection or projection bias is a psychological defense
mechanism where a person unconsciously denies their own attributes, thoughts, and
emotions, which are then ascribed to the outside world, such as to the weather, a tool, or to
other people.
- Thus, it involves imagining or projecting that others have those feelings.It is not a part of OCD.

Answer. b

www.damsdelhi.com Email: info@damsdelhi.com


DAMS CBT 2015 Test -1

293. A Patient with pneumonia for 5 days is admitted to the hospital. He suddenly ceases to recognize the
doctor and staff, thinks that he is in jail and complains of scorpions attacking him. He is in altered sensorium;
this condition is

a. Acute delirium

b. Acute dementia

c. Acute schizophrenia

d. Acute paranoia

Solution. Ans-293: (a) Acute delirium


Ref:Read the text below
Sol:
The patient in question has an acute confusional disorder with:
- Disturbance in consciousness (altered sensorium)
- Disturbance in orientation – to place (thinks, he is in jail)
- To person (ceases to recognize the doctor and staff)
- Disturbance in perception (complains of scorpions attacking him)

All these features are consistant with the diagnosis of delirium

Answer. a

294. A person missing from home, is found wandering purposefully. He is well groomed, and denies of
having any amnesia. Most likely diagnosis is:

a. Dissociative fugue

b. Dissociative amnesia

c. Schizophrenia

d. Dementia

Solution. Ans-294: (a) Dissociative fugue


Ref:Read the text below
Sol:
Dissociative fugue is characterized by
- sudden onset of complete amnesia for his earlier life
o Patient usually wander’s away from home
o Adapts new purposeful identity
o Absence of awareness of amnesia
- All features in the questions are consistend with the diagnosis of dissociative fugue.
- In dissociative amnesia, memory loss is patchy and not complete.
- It is characterized by sudden inability to recall important personal information, particularly
concerning stressful or traumatic life events. Patients does not adapt, any new identity and
www.damsdelhi.com Email: info@damsdelhi.com
DAMS CBT 2015 Test -1
awareness of amnesia be present in the post amnestic period.

Answer. a

295. Babu, a 40 years aged male complains of sudden onset palpitations and apprehension. He is sweating for
the last 10 minutes and fears of impending death. Diagnosis is

a. Hysteria

b. Cystic fibrosis

c. Panic attack

d. Generalized anxiety disorder

Solution. Ans-295: (c) Panic attack


Ref:Read the text below
Sol:
A panic attack is a discrete episode of acute anxiety
The episode is usually:
- Sudden in onset
- Lasts a few minutes
- Characterized by severe anxiety (indicated

Answer. c

296. Drug of choice for panic disorder is :

a. Clonazepam

b. Fluoxetine

c. Chlorpromazine

d. Lithium

Solution. Ans-296: (b) Fluoxetine


Ref: Harrison’s--2548
Sol :
- Drug of choice for generalized anxiety – benzodiazepines.
- Drug of choice for panic disorder – antidepressants.

Answer. b

297. ECT is most useful in which type of schizophrenia :

www.damsdelhi.com Email: info@damsdelhi.com


DAMS CBT 2015 Test -1

a. Simple

b. Hebephrenic

c. Catatonic

d. Paranoid

Solution. Ans-297: (c) Catatonic


Ref: Ahuja - 184
Sol :

Answer. c

298. The treatment of choice in the management of acute manic illness is :

a. ECT

b. Amitryptiline

c. Diazepam

www.damsdelhi.com Email: info@damsdelhi.com


DAMS CBT 2015 Test -1
d. Lithium

Solution. Ans-298: (d) Lithium


Ref: Ahuja - 184
Sol :
- Drug of choice for acute mania – Li.
- Main stay of treatment in bipolar disorder – lithium carbonate.

Answer. d

299. The following modes of therapy may be useful for treatment of obsessive compulsive disorder except :

a. Fluoxetine

b. Clomipramine

c. Behaviour therapy

d. Electroconvulsive therapy

Solution. Ans-299: (d) Electroconvulsive therapy


Ref: Ahuja -100
Sol :
Treatment of obsessive compulsive disorders is :

- Serotonine reuptake inhibitor like flluoxetine, fluvoxamine, sertraline,


- Compipramine
- Behavior therapy
- Psychotherapy.
For extreme cases, that are treatment resistant and chronically debilitating, electroconvulsive
therapy and psychosurgery are considerations.

Answer. d

300. Which one of the following is not correct in cases of bulimia nervosa ?

a. Recurrent bouts of binge eating

b. Lack of self-control over eating during binges

c. Self-induced vomiting or dieting after binge

d. Continuous weight gain

Solution. Ans-300: (d) Continuous weight gain


Ref: Read the text below
Sol :
Bulimia nervosa is an eating disorder characterized by binge eating, or consuming a large amount of
food in a short amount of time, followed by an attempt to rid oneself of the calories consumed, usually
www.damsdelhi.com Email: info@damsdelhi.com
DAMS CBT 2015 Test -1
by purging (vomiting) and/or by laxative, diuretics or excessive exercise
These cycles often involve rapid and out-of-control eating, which may stop when the bulimic is
interrupted by another person or the stomach hurts from overextension, followed by self-induced

vomiting or other forms of purging. This cycle may be repeated several times a week or, in more serious
cases, several times a day, and may directly cause:
- Chronic gastric reflux after eating
- Dehydration and hypokalemia caused by frequent vomiting
- Electrolyte imbalance, which can lead to cardiac arrhythmia, cardiac arrest, and even death
- Esophagitis, or inflammation of the esophagus
- Boerhaave syndrome, a rupture in the esophageal wall due to vomiting
- Oral trauma, in which repetitive insertion of fingers or other objects causes lacerations to the
lining of the mouth or throat
- Gastroparesis or delayed emptying
- Constipation
- Infertility
- Enlarged glands in the neck, under the jaw line
- Peptic ulcers
- Calluses or scars on back of hands due to repeated trauma from incisors
- Constant weight fluctuations are common
The frequent contact between teeth and gastric acid, in particular, may cause:
- Severe dental erosion
- Perimolysis, or the erosion of tooth enamel
- Swollen salivary glands

Answer. d

www.damsdelhi.com Email: info@damsdelhi.com

You might also like